You are on page 1of 287

UNDER THE GUIDANCE OF DR.

Anurudhdha Padeniya
Done by Deepaani & Nilmini 07/08, Sanooshiya 08/09, Rizky & In-aam 09/10

TOF-Murmur, cyanosis, squatting on exercise,


Congenital Heart Diseases hypercyanotic spell(Rapid in cyanosis, irritability,
inconsolable crying, breathlessness, pallor, precipitated with
Introduction : dehydration, infection, anaemia)
Age TGA
Sex-male: coarctation of aorta, TGA RS- surfactant deficiency, meconeum aspiration, pulmonary
hypoplasia, LRTI
-Female: ASD, PDA
PERSISTANCE PULMONARY HTN OF THE NEWBORN
Presentations:
INFECTION-septicaemia, GBS etc
Antenatal USS diagnosis
INBORN ERROR OF METABOLISM-metabolic acidosis,
Detection of a heart murmur
shock
Cyanosis
POLYCYTHEMIA
HF-(Breathlessness specially on feeding and exertion or
CNS OR PNS HYPOVENTILATION (irregular shallow
rest, scalp sweating on sucking, poor feeding, recurrent chest
weak respiration associated with CNS syn & sym)-birth
infection, N & V, loose stools)
asphyxia, ICH, diaphragmatic palsy
shock

DD’s:

According to the P/C


History of presenting complain
Ex-: Cyanosis-
. What age they notice the problem first?
CHD:
. How did they present / diagnosed where?

Page | 1
FMAS, RUSL
UNDER THE GUIDANCE OF DR. Anurudhdha Padeniya
Done by Deepaani & Nilmini 07/08, Sanooshiya 08/09, Rizky & In-aam 09/10

Ex: cyanosis, respiratory distress, scalp sweating, difficulty in . Prolonged fever (rare in infancy – can follow start Sx-VSD,
breathing, inadequate weight gain, fever (IE) TOF,PDA, TGA), high intermittent fever, fatigue,myalgia,
arthralgia, headache.
.Recurrent RTI –how often? Severity?
. When was the child Rx? Where? By whom? What was done? How
. SOB-Infant pants for breath while crying / feeding often is the child hospitalised?
. Feeding difficulty-

How long on each breast? Does mother feel that breast has
Antenatal history
emptied? SOB & scalp sweating
during feeds? How long does the child sleep after feeds? (Normally Antenatal uss- any abnormalities detected
sleeps for 3 – 4 hrs) inadequate will get up soon – Does the child
get up irritated & hungry? Regurgitate Could be inherent (Downs, At pregnancy – mother’s age(down’s?)GDM, (2% Of CHD-HOCM,
Rubella) Is mother’s let down reflex good- (check techniques in VSD) SLE(35% CHD- congenital heart block)
examination) Rubella – vaccination, fever , rash –specially in T1( 5% CHD-PDA
. Vomiting / LOA ,peripheral pulmonary stenosis, characteristic)

. Respiratory distress – rapid breathing, nasal flaring, chest Alcohol abuse during pregnancy- foetal alcohol Xn(Receding chin)
retraction der children – poor exercise tolerance, increased naps, –ASD/VSD/TOF
poor growth, what they do to relieve ex: squatting (TOF) Drugs taken in antennal period- phenytoin-coarctation of aorta ,
. Inadequate weight gain – who noticed? When? What was done? semilunar vale stenosis

.Cyanosis – at rest, on crying /exertion / feeding? Developed at what Warfarin- PDA, pulmonary valve stenosis
age? If present, look for-headache and vomiting (cerebral abscess), Litium- ebstain anomaly
convulsions, LOC, stroke

Page | 2
FMAS, RUSL
UNDER THE GUIDANCE OF DR. Anurudhdha Padeniya
Done by Deepaani & Nilmini 07/08, Sanooshiya 08/09, Rizky & In-aam 09/10

BIRTH Hx Clinic visits (regular? noticed anything)

Prematurity, cyanosis , respiratory distress Family Hx


Birth weight – IUGR (rubella) o Consanguinity
TORCH screen done ( in mother/ in baby)- blood sent to MRI o FHx of CHD
FAMILY Hx
o Generalized muscle disease
Consanguinity
o Inherited/ congenital abnormality among 1st degree
FHx of CHD (very rare)- holt oram Xn (autosomal dominant ) ASD relatives

Generalized muscle disease Social Hx→


Inherited/ congenital abnormalities in immediate family o Socio economic back ground
NUTRITIONAL Hx o Recurrent hospitalization- burden on economy
FTT- very detailed
o Occupation
How the food is prepared
o Transport
Complimentary feeding?
o How the other children are affected
24hr dietary recall
o Family support, extended family
DEVELOPMENT Hx
o Compliance
Milestones- recognize mother, social smile ect.
o Knowledge of the Mx of hypercyanotic spell
IMMUNIZATION Hx

Page | 3
FMAS, RUSL
UNDER THE GUIDANCE OF DR. Anurudhdha Padeniya
Done by Deepaani & Nilmini 07/08, Sanooshiya 08/09, Rizky & In-aam 09/10

Examination: Terner’s- AS, coarctation

General→ Marfans- MVP, AR, MR

o Ht, length, Wt, OFC Foetal alcohol syn- recending chin

o ill or well looking Noonas syn- right hrt dx, PS, ASD

o active or not Rubella-

o pallor, anemic features o dental carries

o plethora o cold periphery

o cyanosis-central(reduced Hb >59/dl) or peripheral o oedema

(Persistent cyanosis in otherwise well infant is nearly o clubbing- seen after 1 yr


always a sign of structural hrt dx)
o features of IE
Cyanosis more in LL than UL(coarctation of aorta,
PDA with R to L shunt)

o iv cannula CVS→

o res distress- increased HR, suprasternal recession, o full examination


SCR, ICR
o PDA
o dysmorphic-
bounding peripheral pulses
down’s- VSD, ASD, TOF
lower limb pulses are extremely prominent

Page | 4
FMAS, RUSL
UNDER THE GUIDANCE OF DR. Anurudhdha Padeniya
Done by Deepaani & Nilmini 07/08, Sanooshiya 08/09, Rizky & In-aam 09/10

left para-sternal heaves loud pan systolic murmur at lower left sternal edge
(in small VSD loud but in large soft)and quite
loud P2 if in pulmonary hypertension pulmonary second sound
continuous mechanical murmur in the left o Transposition of great vessels
infraclavicular area
Second heart sound loud and single
o TOF
no murmur
Central and peripheral cyanosis
o Aortic stenosis
Child may be small for age
Small volume slow rising pulse, carotid thrill,
Left para sterna heave ejection systolic murmur at upper right sterna edge
radiating to neck
A loud harsh ejection systolic murmur at the
L sterna edge from D1(during the Delayed and soft aortic second sound
hyypercyanotic spell very short or inaudible)
Apical ejection click
Single S2
Pulmonary stenosis
o ASD
Ejection systolic murmur at upper left sterna edge,
Ejection systolic murmur at upper left sterna edge
and Fixed split s2 Thrill may be present,

o VSD ejection clicK at upper left sterna edge

prominence right ventricular impulse-heave

Page | 5
FMAS, RUSL
UNDER THE GUIDANCE OF DR. Anurudhdha Padeniya
Done by Deepaani & Nilmini 07/08, Sanooshiya 08/09, Rizky & In-aam 09/10

RS→ Boot shape- TOF

o RR Egg on side- TGA

o Look for RTI- not in TOF (decreased blood to lungs) Snow man –TAPVD

o Pulmonary edema- crepts, grunting respiration Lungs –LRTI, pulmonary oedema-Bat’s wing, curly B
lines
Abd→
ASD Cardiomegal , enlarged pulmonary arteries and increased
pulmonary vascular marking
o Tender hepatomegaly (RVF due to increased RA Large VSD Cardiomegaly ,enlarged pulmonary arteries and
pressure) increased pulmonary vascular marking pulmonary
CNS→ edema

Developmental Assessment→ PDA Normal or enlarged heart

Investigations: Enlarged pulmonary arteries and increased


pulmonary

vascular marking
Radiological- CXR
TOF small heart, boot shaped ape ,pulmonary artery bay
Cardiac size- PA telechest in mid inspiration and decreased vascular marking

Maximum cardiac width (enlarged)maximum cardiac Transposition of great vessels Narrow upper mediastinum
width >50% (in infants not useful) with egg on side appearance of cardiac
shadow and increased pulmonary
Enlargement of cardiac chambers
vascular marking
Pulmonary vascularity

Page | 6
FMAS, RUSL
UNDER THE GUIDANCE OF DR. Anurudhdha Padeniya
Done by Deepaani & Nilmini 07/08, Sanooshiya 08/09, Rizky & In-aam 09/10

Aortic stenosis Normal or prominent left ventricle post Intra cardiac pressure
stenotic dilatation of ascending aorta
Qualitation of cardiac contractile function
Pulmonary stenosis Norma or post stenotic dilatation of
pulmonary artery Direction of flow across the defect

ECG Intergrity of coronary arteries

R axis deviation-order rhythm strips look for arrthmia Presence of vegitations due to IE

L axis deviation-tricuspic atresia unless proven otherwise Presence of pericardial fluid/ tumors/chamber thrombi

Haematological Assistance Tx- balloon arterial septostomy,


pericardiocentesis, biopsy, etc
FBC- infection, PCV, HB%
Blood gasses
Polycythemia- PCV> 65%
RBS
In cyanosis (R-L shunts-if Sx evaluvate for abnormal
coagulation) MRI

Risk of vascular thrombosis-cerebral Cardiac catheterization


veins(dehydration/Fe deficiency-increase risk,diuretics Indications
use with causion)therefore if severely cyanotic-periodic
Hb% & PCV o With echo incomplete pre surgical evaluation

Rx-correct hydration,Fe Rx, phlebotomies(replace blood o Evaluation of pulmonary vascular resistance


with FFP/albumin)
o Follow up after complete surgical repair
Echo
o Myocardial biopsy- cardiomyopathy, after
Cardiac structure transplantation

Page | 7
FMAS, RUSL
UNDER THE GUIDANCE OF DR. Anurudhdha Padeniya
Done by Deepaani & Nilmini 07/08, Sanooshiya 08/09, Rizky & In-aam 09/10

o Interventriculer cardiac catheterization rejection ex: If needed muscle paralysis and artificial ventilation
in PS/ AS closure of PDA
Manage predicating factors- Control infections, correct
anaemia, correct dehydration

Management: Educate parents regarding the initial mx of hypercyanotic


spell (Knee chest,squatting position and take the baby
Initial management –Medical immediately to the hospital)
Palliative shunt- Modified Ballock- Taussing shunt Prevent recurrent Ix. IE prophylaxis
(artificial tube anastomosing the subclavian artery and the
pulmonary artery- it increase pulmonary blood flow Target Questions

Definitive management- surgery at around 6 months (close 1. congenital heart disease


VSD and relieve RV out flow tract obstruction with artificial
patch)
Cyanotic –TOF, TGA, Tricuspid atresia, truncusarteriosus, Ebstein
‘sanomaly, complete AVSD
Management of hyper cyanotic spell (if >15 min)

Child in the knee position


Acyanotic
100% O2
Left to right shunt (ASD, VSD, PDA,
IV fluid bolus
Outflow obstruction in well child- AS, PS, coarctation of aorta
IV morphine- sedation and pain relief
Out flow obstruction in sick infant- interruption of the aortic arch,
IV beta blockers- propranolol hyperplastic left heart sx

Bicarbonate to correct acidosis TOF

Page | 8
FMAS, RUSL
UNDER THE GUIDANCE OF DR. Anurudhdha Padeniya
Done by Deepaani & Nilmini 07/08, Sanooshiya 08/09, Rizky & In-aam 09/10

2. What are the cardinal anatomical features of TOF?4 Infective endocarditis

HF (very rare)

Large VSD 5. What are the duct dependent lesions?

Over riding of aorta Coarctation of aorta and interruption of aortic arch

Sub pulmonary stenosis causing RV out flow tract obstruction Hypo plastic left heart Sx

RVH

3. Associated syndromes? 6. Heart murmur


Down syndrome Physiological
Di George anormaly Asymptomatic patient
Velocardiofacial syndrome Soft blowing murmur

Systolic murmur only

4. Complications? At Left sterna edge


Cerebral thrombosis (common in <2yrs, due to dehydration and Also
polycythemia)
Normal heart sound without added sound
Brain Abscess (> 2 yrs)
No parasternal thrills
Hyper cyanotic spells
No radiation

Page | 9
FMAS, RUSL
UNDER THE GUIDANCE OF DR. Anurudhdha Padeniya
Done by Deepaani & Nilmini 07/08, Sanooshiya 08/09, Rizky & In-aam 09/10

Ex Febrile illness Mitral stenosis and tricuspid stenosis

Anaemia Continuous

Pregnancy PDA

Bradycardia (Athletes) 7. What are the causes for outflow obstruction in a well child?

Aortic stenosis

Pathological murmur Pulmonary stenosis

Systolic Adult type coarctation of the aorta

Ejection systolic 8. What are the causes for outflow obstruction in the sick
infant?
ASD, Aortic stenosis, Pumonary stenosis and hypertrophic
obstructive cardiomyopathy and aortic regurgitation Coarctation of the aorta

Pansystolic Interruption of the aortic arch

Mitral regurgitation, tricuspid regurgitation,VSD and leaking mitral .hypoplastic left heart syndrome
or tricuspid prosthesis
9. what are the signs of heart failure?

Tender hepatomegaly
Diastolic murmur
Tachycardia
Early
Tachypnea
Aortic regurgitation and pulmonary regurgitation
Enlarged heart
Mid

Page | 10
FMAS, RUSL
UNDER THE GUIDANCE OF DR. Anurudhdha Padeniya
Done by Deepaani & Nilmini 07/08, Sanooshiya 08/09, Rizky & In-aam 09/10

Gallop rhythm

Poor weight gain

Cool peripharies

10. what are the clinical features of ASD?

Commonly asymptomatic

Symptoms

Recurrent chest infections

Arrhythmias

Signs

Ejection systolic murmur best heard in upper left sternal


edge

Fixed and widely splitting 1st heart sound.

Page | 11
FMAS, RUSL
UNDER THE GUIDANCE OF DR. Anurudhdha Padeniya
Done by Chandi Prasangika07/08,Thushan08/09,Nimanthika09/10

Serum sickness
Rheumatic Heart Disease
Sickle cell disease
Introduction-
SLE
It’s an inflammatory disorder. Most important cause for acquired
heart disease in children worldwide.Incidence can be minimized by Malignancies
the improvement of sanitation,social factors and the more liberal use
Lyme disease
of antibiotics. In susceptible individuals there is an abnormal immune
response to a preceding infection with group A beta haemolytic Gonococcal infection
streptococcus . Usually 14-28 days after infection.
JIA
Sex- both sexes
Carditis
Presentation-
Viral myocarditis
Fever with joint involment
Viral pancarditis
Fever with carditis
IE
Chorea
Kawasaki disease

CHD
DDs
Mitral valve prolapse
Joint involvement
Innocent murmurs
Rheumatoid arthritis
Chorea
Reactive arthritis
Hungtingtons chorea
Page | 12
FMAS, RUSL
UNDER THE GUIDANCE OF DR. Anurudhdha Padeniya
Done by Chandi Prasangika07/08,Thushan08/09,Nimanthika09/10

Wilson’s disease Begin in large joint in lower extremities

SLE Migrate to other large joint (lower or upper


extremities)
Cerebral palsy
One or more joint are involve
Tics
Uncommon in small joint
Hyperactivty
Respond to aspirin

Assosiated with any skin rash or nodules or


HISTORY abnormal movements

Presenting complain Recent past history of sore throat /recurrent


tonsillitis
1. Fever with multiple joint
pain/swelling (acute onset) Past history of rheumatic fever

History of presenting complain

Fever - Onset,duration,character,response to Other clinical manifestation – abdomen pain


antipyretics
Bleeding from the nose
mild fever
Weight loss
No characterized variation
Malaise
Decrease without PCM
Fatigue
Persist for 2-3 weeks

Joint pain – multiple joint pain, swelling

Page | 13
FMAS, RUSL
UNDER THE GUIDANCE OF DR. Anurudhdha Padeniya
Done by Chandi Prasangika07/08,Thushan08/09,Nimanthika09/10

Exclude differential diagnosis Abnormal movement associated with difficulty swallowing &speech
and convulsion
Rheumatoid arthritis – most commonly small joint affected
JIA - small joint persistent swelling
Associate with loss function & deformity
Associated with high grade fever & malaise
No migratory involvement
Salmon colored rash with fever
Morning stiffness
Malignancy-LOA,LOW
Reactive arthritis-Preceding history of gastroenteritis, urethritis,
conjunctivitis Serum sickness-Recent history of vaccination(7-12)

Sickle cell disease-Dx pt Urticarial rash

Fx of anaemia Pruritis

Recurrent respiratory tract


infections
2. Fever with carditis
Kawasaki disease - high & persistent fever

Fever not response to PCM


Fever as above
Symmetrical joint involvement
Carditis was detected
Redness of mucous membrane, eye, sole, palm
Recent history of sore throat
SLE – malar rash
Past history of rheumatic fever
More female than male (9:1)
Viral myocarditis/pancarditis-recent history of viral
Huntington chorea - common 35 – 44 age group infection(mumps,measles,viralhepatitis)

Page | 14
FMAS, RUSL
UNDER THE GUIDANCE OF DR. Anurudhdha Padeniya
Done by Chandi Prasangika07/08,Thushan08/09,Nimanthika09/10

IE, CHD,Fx ofanaemia, haematuria Chorea - after long period from sore throat (1-6 months)

Kawasaki disease-Conjunctival njestions ,red mucous Common in female than male


membrane,rash,red & oedematous palms &soles
Uncontrolled unilateral movement
Fx of heart failure- Orthopnoea, PND, Exetional dyspnea
Associated with emotional lability
Complications
In coordination
Pancarditis – (myocarditis, pericarditis,
endocarditis, congestive heart failure) Poor school performance

Chest pain Movement disappearing with sleep

Mild to moderate chest discomfort

Pleuratic chest pain

Cough

Difficulty breathing Erythema marginatum – rashes over the trunk & proximal limbs

Orthopnoea No on face

Associated with fever Rash may fed & reappear

Poly arthritis-Knee,Ankle, Elbow, Wrist Exacerbated by heat

Inflammation begins within few days to week. Disapear Nonpruritic, Painless, Serpiginous
within2-4 weeks. Classicaly migrating Subcutaneous nodule – over the bonny prominences

Associated with carditis

Page | 15
FMAS, RUSL
UNDER THE GUIDANCE OF DR. Anurudhdha Padeniya
Done by Chandi Prasangika07/08,Thushan08/09,Nimanthika09/10

Past medical history - Resent history of pharyngitis or skin infection Ill looking

History of rheumatic fever Febrile

Past surgery history - no Pallor, icteric, dental carries

Birth history – no Sydenham’s chorea – milkmaids grip

Immunization history – no Spooning & pronation of hand

Development history – no Fine tremor involve face ,


hands, feet
Family history – hx of rheumatic fever
Wormian darting movements
Hx of SLE of tongue( Jack in the box)
Hx of Huntington chorea Dysarthric speech
Dietary history – no Gait problems
Allergic history – no Poor fine motor skills
Drug history- Benzathine peniciline injection Erythema marginatum – erythematous rise margins
Social history – poor sanitation Central clear
Overcrowding Size 1 -3 cm

Subcutaneous nodule – firm


Examination Non tender
General examination Freely attach to overlying skin

Page | 16
FMAS, RUSL
UNDER THE GUIDANCE OF DR. Anurudhdha Padeniya
Done by Chandi Prasangika07/08,Thushan08/09,Nimanthika09/10

Large joint – swollen, erythematous, warm, tender, New murmur- high pitched, blowing, holosystolic,
apical murmur of MR
Ankle oedema (CHF)
Low pitched, apical, middiastolic, flow
murmur &high pitched, dececendo , diastolic murmur of AR at
Respiratory system aortic area

-According to complication

Bilateral crepitation (CHF) Abdomen examination

Heoatomegally(CHF)

CVS examination

CHF - Tachycardia CNS examination – chorea, milk maid grip, jack in the
bo,dancing pupils, spooning & pronation of the palms when the
3rd heart sound hands are extended,hand writing for fine motor involvement

Distention JVP How you are going to diagnose Rheumatic fever?

Gallop rhythm Using the Jones criteria

Cardiomegally - displace apex 2 major/ 1major & 2 minor criteria + supportive evidence of
preceding group A streptococcal infection (markedly ASOT/ other
Pericarditis - friction rub
streptococcal Abs / group A streptococcus in throat culture)
Pericardial effusion – cardiac dullness
MAJOR MINOR
Muffled heart sound
1.Pancarditis (50%) 1.Fever
Changing murmur 2. Poly arthritis(80%) 2.Polyartherlgia
3.Sydenham chorea(10%) 3. History of rheumatic fever
Page | 17
FMAS, RUSL
UNDER THE GUIDANCE OF DR. Anurudhdha Padeniya
Done by Chandi Prasangika07/08,Thushan08/09,Nimanthika09/10

4.Erythema marginatum(less 4. High acute phase proteins- Chest X ray – heart failure feature
than 5%) ESR,CRP, Leukocytosis
5.Subcutaneous nodules(rare) 5. Prolong PR interval on ECG Cardiomyopathy

Pericardial effusion

Echocardiography – new valvular regurgitation /


valvular dysfunction
Investigation
Other-
To confirm
ECG – prolonged P-R interval
FBC (Neutrophil leucocytosis)
Rarely 2nd &3rd degree heart block
Throat swab culture & ABST
Biopsy from subcutaneous nodule – aschoff bodies
ESR (high)
Rheumatoid factor
C reactive protein (high

Streptococcal antibody test(ASOT)


Management

Medical care-

Main 4 aspects
To assess complications
1. Treatment for group A streptococci
Blood culture & ABST- IE, bacteremia,
disseminated gonococcal infection Antibiotic therapy –( according to throat culture report)

Imaging- Oral penicillin or erythromycin – 10 days

Page | 18
FMAS, RUSL
UNDER THE GUIDANCE OF DR. Anurudhdha Padeniya
Done by Chandi Prasangika07/08,Thushan08/09,Nimanthika09/10

Or Single IM benzathine penicillin 3.Supportive therapy

2.General treatment of acute episode Heart failure- Digoxine, diuretics, ACE inhibitors

Anti inflammatory therapy – Controle arthritis, srttle fever & Pericardial effusion – pericardiocenteces
other acute symptoms
If Sydenham’s chorea
Polyathritis+carditis without cardiomegally or CHF
Sedatives – Phenobarbital it ineffective use haloperidol or
Aspirin 100mg/kg/daily in 4 divided doses 3 -4days orally, after that chlorpromazine
aspirin 75mg/kg /daily in 4 divided doses orally for 4 weeks ( se-
gastritis, increase bleeding tendency) 4. Prophylaxis

IM benzathine penicillin every 3weekS (high risk) or


Polyarthritis+ carditis+cardiomegally or CHF
4weeks up to age 18 or 21 year or 5years duration
Corticosteroid (prednisalon) 2mg/kg/day in 4 divided doses alternatively oral penicillin ever day
for 2-3 weeks, after tapering the dose by reducing the dose by
5mg/day every 2-3 days.

At the beginning of tapering ,Start aspirin 75mg/kg/day in 4 divided


doses for 6 weeks . As alternative digoxin, fluid & water
restriction,oxygen )

Bed rest

Chorea- usually manage conservatively / phenobarbital 16-


32 mg 6-8 hrs. If ineffective haloperidol 0.01-0.03mg/kg/day or
chlorpromazine (0.5mg/kg 4-6 hrs)

Page | 19
FMAS, RUSL
UNDER THE GUIDANCE OF DR. Anurudhdha Padeniya
Done by Mubashshira07/08.,Zamara08/09, Indeewarie09/10

Hx/P/C→
Infective Endocarditis
o Fever- can be,

1. Acute onset sever high grade intermittent fever or,


Presentations:
2. Mild prolonged fever (which could be the only
Prolong fever feature) –in 45%
Newly diagnosed murmur
Associated with,
Highly suggestive of IE
Weight loss, loss of appetite, arthragia, myalgia, malaise
Newly diagnosed of heart murmur due to anemia

Prolonged fever Child may present with a newly diagnosed or


changing cardiac murmur – 90% if so by
DD’s: hematuria
whom
Prolonged fever-mentioned in long case PUO
Exclude other causes of POU
Newly diagnosed murmur-rheumatic fever

History:
Complications of IE
P/C→
1. Heart failure 50%-(due to myocardial abscesses
o Fever for __ duration which can damage the cardiac conduction system
& cause heart block)
o Newly diagnosed of cardiac murmur
Infants- scalp sweating, less active, poor feeding,
FTT

Page | 20
FMAS, RUSL
UNDER THE GUIDANCE OF DR. Anurudhdha Padeniya
Done by Mubashshira07/08.,Zamara08/09, Indeewarie09/10

Older children: poor exercise tolerance, fatigue, orthopnea, PND, Osteomyelitis- limb pain, limp/ restricted
tachypnea, facial puffiness, tenderness in RHC movements, back pain, groin pain.

4. Renal involvement

2. embolic events(infrequent) Haematuria-70%

Pulmonary embolism- acute breathlessness, pluratic type chest pain, Glomerulonephritis- frothy urine, anuria
hemoptysis, dizziness, syncope.

Intracranial- embolic stroke 20%

-intra cranial hemorrhages 10% Hx of taking any antibiotics- culture negative IE

These can give rise to increased intracranial pressure-


features- early morning headache worsening with sneezing, bending
forward’, vomiting. P/M/Hx→

o Congenital heart disease

3. Metastatic infections o History of rheumatic fever

Meningitis- headache, vomiting, features of meningial irritation. o IV drug use

Arthritis 25%- joint pain, warmth, restricted movements. o Central venous catheter / Hx of prolonged hospitali-
zation( hospital acquired bacteremia)
Mycotic aneurysms

Pericarditis
Page | 21
FMAS, RUSL
UNDER THE GUIDANCE OF DR. Anurudhdha Padeniya
Done by Mubashshira07/08.,Zamara08/09, Indeewarie09/10

o Prosthetic heart valve/ VSD patches / shunts (ask o Ill looking


about the date as the organism change according to
duration) o Febrile

o Preceding dental, urinary or intestinal procedure o Pallor

Birth Hx→ o Cyanosis

o Normal paediatricHx o Oral hygiene

ImmHx→ o Embolic phenomena-Roth spots, petechiae, splinter


nail bed hemorrhages (linear lesions beneath nails) ,
o Normal paediatricHx Osler nodes, CNS or ocular lesions

Diet Hx→ o Peripheral stigmata of IE

o Normal paediatricHx Clubbing

Social Hx→ Janeway lesion-painless small erythematous


or hemorrhagic lesions on the palms or soles
o IV drug abuse
Osler nodes-tender pea sized intra dermal
nodules in the pads of palms & soles

Examination: o arthritis

General→ o Ankle edema/ sacral edema (HF)

o Weight loss o Cold peripheries


o IV cannula

Page | 22
FMAS, RUSL
UNDER THE GUIDANCE OF DR. Anurudhdha Padeniya
Done by Mubashshira07/08.,Zamara08/09, Indeewarie09/10

Cardinal features of HF

CVS→ Tachypnea

o Pulse-tachycardia Tachycardia
o BP-narrow PP,↓SBP
o JVP Cardiomegaly
o Surgical scars-prosthetic valve
o Visible, displaced apex Hepatomegaly
o RV heaving Investigations:
o Thrills
o New or changing murmur FBC→
o Gallop rhythm
o WBC↑
o Arrhythmias
o Hb (Anemia)
RS→
UFR: haematuria
o tachypnoea
o Pleural effusion
o Pleural rub
o Bibasal crepitation Blood Culture-take 3 samples from different 2 sites in 24hr
Abd→

o Hepatosplenomegaly If a patient has sub-acute IE collect blood during fever spike


Soft spleen is very significant, as at that time no. of bacteria in blood is high (high
Pulsatile tender hepatomegaly in HF bactereamia)

CNS→
2D echo→(TOE is the best)
Page | 23
FMAS, RUSL
UNDER THE GUIDANCE OF DR. Anurudhdha Padeniya
Done by Mubashshira07/08.,Zamara08/09, Indeewarie09/10

Vegetation o Fever,>38 c

Valve dysfunction o Embolic-vascular signs-major arterial emboli, septic pulmo-


nary infarcts, mycotic aneurysms’ ICH,
Embolic complication
o Immune complex phenomena (glomerulonephritis, arthritis,
Reduced ejection fraction rheumatoid factor, Osler nodes, Roth spots),

o A single positive blood culture or serologic evidence of in-


↑ESR fection

↑CRP(monitoring the treatment response) o Echocardiographic signs not meeting the major criteria.

o The presence of newly diagnosed clubbing, splenomegaly,


splinter hemorrhages, and petechiae;
Diagnosis
o A high erythrocyte sedimentation rate;
By Duke’s criteria
o A high C-reactive protein level; and
Major criteria include
o The presence of central nonfeeding lines, peripheral lines,
(1) Positive blood cultures (two separate cultures for a usual and microscopic hematuria.
pathogen, two or more for less typical pathogens)

(2) Evidence of endocarditis on echocardiography (intracardiac mass


on a valve or other site, regurgitant flow near a prosthesis, abscess, Two major criteria, one major and three minor, or five minor
partial dehiscence of prosthetic valves, or new valve regurgitant criteria suggest definite endocarditis.
flow).

Minor criteria include


Management:
o Predisposing conditions, ( heart conditions, IV drug use)
Page | 24
FMAS, RUSL
UNDER THE GUIDANCE OF DR. Anurudhdha Padeniya
Done by Mubashshira07/08.,Zamara08/09, Indeewarie09/10

IV antibiotics-for 4-6 weeks Target Questions

Salt restriction 1) What are the common organisms found in IE

Diuretic therapy 1) Viridians type streptococcus*-dental procedure

Surgical intervention-for severe aortic, mitral valve involve- 2) Staphylococcus*


ment with intractable HF, mycotic aneurysm, rupture of aor-
tic sinus, failure to sterilize vegetation with antibiotics, myo- 3) Staphylococcus Endocarditis-common in normal heart valve
cardial abscess, recurrent emboli, increasing size of vegeta- 4) Group D enterococci-lower bowel or GU manipulation
tion
5) Pseudomonas aeruginosa-IV drug uses
Fungal endocarditis-(DOC)Amphotericin B &Flurocytosine
6) Serratia--IV drug uses

7) Fungi-open heart surgery


Investigations
Blood cultures 3sets, different venepuncture sites 8) Coagulase (-) staphylococcus-indwelling CV catheter
Serological tests-for culture negative cases
*leading organisms in paediatrics
FBC-↓hb, ↑WBC, ↑or↓ PLT
BU & Sr E-↑
Liver biochemistry-↑ALP
Inflammatory markers-↑ESR, CRP 2) Complications of IE
Urine-proteinuria & hematuria
ECG- prolonged PR, heart block HF
CXR-pulmonary edema (left sided IE) pulmonary emboli, abscess
CNS involvement due to embolization(ICH due to rupture of
(right sided disease)
vessels)
Echo-transthorasic (1st line, non invasive), transoesophagial (2nd
Pulmonary embolization
line, invasive, higher sensitivity)

Page | 25
FMAS, RUSL
UNDER THE GUIDANCE OF DR. Anurudhdha Padeniya
Done by Mubashshira07/08.,Zamara08/09, Indeewarie09/10

Mycotic aneurysm Vigorous Tx of sepsis &local infection

Rupture of a sinus valsalva Careful asepsis during heart surgery& catheterization

Obstruction of valve

Acquired VSD 4) Indications for infective endocarditis prophylaxis in dental pro-


cedures
Meningitis
-6 2007 STATEMENT OF THE AE
Osteomyelitis Prosthetic cardiac valve or prosthetic material used for
cardiac valve repair
Arthritis Previous infective endocarditis
CONGENITAL HEART DISEASE (CHD)*
Renal abscess Unrepaired cyanotic CHD, including palliative shunts
and conduits
Immune complex mediated glomerulonephritis Completely repaired CHD with prosthetic material or de-
vice, whether placed by
ARF surgery or
catheter intervention, during the first 6 mo after the pro-
cedure†
Repaired CHD with residual defects at the site or adja-
3) What precautions would you take to prevent bacterial endo- cent to the site of a
carditis prosthetic patch, or
prosthetic device (which inhibit endothelization)
Antimicrobial prophylaxis before any dental, GIT, GU local Cardiac transplantation recipients who develop cardiac
procedures in susceptible patients valulopathy

Continuing education regarding prophylaxis

Proper general care & oral hygiene

Page | 26
FMAS, RUSL
UNDER THE GUIDANCE OF DR. Anurudhdha Padeniya
Done by Mubashshira07/08.,Zamara08/09, Indeewarie09/10

P/C→

o cough, wheezing, fever with duration


Bronchiolitis
Hx/P/C→

o describe the cough (dry or productive with what col-


Presentations: our of sputum, persist for how long, worse at what
time, associations, end up with, progression, contact
No/Low grade fever
hx, etc)
Rhinorrhoea (coryzal symptoms)
o description of fever
dry cough&increasing breathlessness
o up to now what was done at home & hospital(in a
wheezing often not always
chronological order)
Post tussive vomiting
o feeding difficulties, sleep disturbances
feeding difficulty associated with dyspnea
o DDs
recurrent apnoea (in sever bronchiolitis)
-Pneumonia (high grade fever, poor feeding, cough,
lethargy, cyanosis,
DD’s: tachypnoea, wheeze)
-heart failure (scalp sweating
Bronchiolitis while breast feeding, recurrent
Pneumonia Chest infection, lethargy, FTT)
Bronchial asthma -bronchial asthma(eczema,
Transient early wheezing Recurrent wheeze & family hx
Heart failure Of rhinitis,BA or atopy)
Foreign body inhalation -foreign body inhalation
Aspiration
History: -hx of milk or food aspiration
Age: bronchiolitis(1-9mths)

Page | 27
FMAS, RUSL
UNDER THE GUIDANCE OF DR. Anurudhdha Padeniya
Done by Mubashshira07/08.,Zamara08/09, Indeewarie09/10

P/M/Hx→ Examination:

o congenital heart disease, immunedeficiency(HIV), General→


malnutrition, cystic fibrosis or any other lung dxs, re-
current chest infection o Ill/well looking
D/Hx→
o Steroids inhalation or oral use
o Wt, ht and OFC with centiles

o Nasal flaring ,dyspnic tachypnoea, head bobbing,


Birth Hx→

o Preterm (due to bronchopulmonary dysplasia), low o fontanelles, febrile, pale, cyanosis


birth wt
o mucous membranes& skin turgor for hydration
ImmHx→
RS→
Diet Hx→ o RR,
F/Hx→ o Sharp, dry cough
o parity, BA, atopy, or rhinitis o SCR,ICR, suprasternal recessions, prominent ster-
num, chest expansion, accessory muscle involve-
Social Hx→
ment,
o overcrowding,
o Percussion
o passive smoking
o air entry,VB,rhonchi,crepitations
o Fine-end inspiratory crackles

o High pitched wheezes expiratory>inspiratory

Page | 28
FMAS, RUSL
UNDER THE GUIDANCE OF DR. Anurudhdha Padeniya
Done by Mubashshira07/08.,Zamara08/09, Indeewarie09/10

CVS→ o mild – Mx at home

o pulse volume, PR- Tachycardia o moderate to severe- Mx at hospital

o CRFT, BP, Mainly supportive


Mist, antibiotics, steroids are not helpful
o Heart sounds, murmur
Monitor for apneoa
Abd→
Monitoring (hrly if severe & 2 hrly if moderate).
o liver displaced downwards (percuss for upper border) If SpO2 low(<92%), give O2{humidified O2 via face
CNS→ mask(4L/min)or nasal prongs(2L/min)
Hydration: Give oral fluids if can take oral. If can’t take oral
give NG. If NG is irritable, give IV. If very severe respira-
tory distress, keep NBM.
Investigations:
Antibiotics-not have been shown to reduce the severity & the
o not need in mild duration of the illness.
o if high fever, lethargy do FBC - WBC/DC (exclude second-
Give Antibiotics if secondary bacterial infection is suspected
ary bacterial infection)
Nebulization
o if fever persists , CXR(because bronchiolitis perse not give
< 3mths: ipratropium 4H
this)
>3mths: salbutamol (back to back/hrly/etc)
o CXR - hyperinflation of the lungs due to small airways
&Ipratropium (4H)(add
Obstruction, air trapping, focal atelectasis
Ipratropium because of
o if fever, search for the cause Synergistic effect)
Management: If >1 mths, oral salbuamol
No place for steroids
depend on severity of the disease Mild
• Normal ability to feed
after hx and ex determine the severity and arrange Mx plan

Page | 29
FMAS, RUSL
UNDER THE GUIDANCE OF DR. Anurudhdha Padeniya
Done by Mubashshira07/08.,Zamara08/09, Indeewarie09/10

• Little or no respiratory distress (Resp rate less than 50 o Give oxygen to maintain saturation >92%
breaths/ min) o Give intravenous fluids
• No requirement for oxygen (Oxygen saturation more than o Observe closely anticipating the possible need for
95%) intubation and positive pressure ventilation
* no risk factors o If available monitor arterial blood gases
o No investigation o Consider providing Intensive Care
o Can be treated at home Mechanical ventilation is required in about 2% of infants ad-
o Review in 2-3 days mitted to the hospital
Moderate
• Moderate respiratory distress with intercostal and subcostal
recession (RR 50 -70 breaths /min) Discharge
• Nasal flaring • Feeding re-established
• Mild hypoxaemia (SpO2 92-95%) • SpO2>92 % without administered O2
• Difficulty in feeding • Review after one week
• Brief episodes of apnoea Target Questions
* no risk factors
o Admit to hospital 5) What is respiraory distress?
o Give oxygen / maintain SpO2> 92%
o Consider giving intravenous fluids 1) Tachypnoea
o Adrenaline 1: 1000 3ml , two doses nebulization, 30 min
apart 2) Laboured breathing with chest wall recession & nasal flaring
o Chest x- ray / reassess
3) Expiratory grunting
Severe
• Unable to feed 4) Cyanosis if severe
• Severe respiratory distress with marked chest wall
indrawing (Respiratory rate more than 70 breaths /min) 6) Normal RR according to age& when consider as tachyp-
• Increasingly tired
noea?
• Prolonged apnoeic episodes
• Hypoxaemia not corrected with extra
O2(Oxygen saturation less than 92%)
o Admit to hospital

Page | 30
FMAS, RUSL
UNDER THE GUIDANCE OF DR. Anurudhdha Padeniya
Done by Mubashshira07/08.,Zamara08/09, Indeewarie09/10

Tachypnoea 9) How this dx progress


Age Normal
Peak around 4th,5th days
Neonate 30-50 more than 60
usually lasts for 14days
Infants 20-30 more than 50
10) When to suspect pneumonia?
Young children 20-30 more than 40
High fever
Elder children 15-20 more than 30
Very Ill

7) What are the signs in bronchiolitis? Increased respiratory distress

hyper inflation of chest (prominent sternum, liver dis- Bronchial breathing


placeddownwards)
Crepts prominent in both pneumonia &bronchiolitis.But
prolong expiration, fine end-inspiratory crepitations, high rhonchi &prolong expiration are less likely in pneumonia
pitched wheezes
11) Not respond to treatment? Recurrence?
hypoxia
80% of bronchiolitis occur due to RSV.So after an episode there
tachycardia is development of immunity against RSV.So recurrence is
unlikely.
8) Why develop prolong expiration?
If not respond to treatment, consider atypical organisms &also
Virus causing epithelial necrosis & initiating an inflammatory secondary bacterial infection.
response. The developing oedema& exudate result in partial
obstruction which is most pronounced in expiration If recurrence, consider

-early wheeze which can result asthma.,So


Page | 31
FMAS, RUSL
UNDER THE GUIDANCE OF DR. Anurudhdha Padeniya
Done by Mubashshira07/08.,Zamara08/09, Indeewarie09/10

Giving steroids such as budesonide & hydrocortisone is RSV- highly infectious, infection control measures are needed to
helpful prevent cross infection.

-atypical organisms Prognosis- most infants recover from acute infection within 2 weeks

-bronchiolitis obliterans Rarely following adenovirus infection, the illness result in


permanent damage to the airways
8.Risk factors for severe bronchiolitis
-Host factors 11.wt are the complications of bronchiolitis?
Pre term
Parity Cyanosis, apnoea,dehydration, fatigue & respiratory failuer
Age < 6 weeks
Congenital heart disease
Immune deficiency
- Environment
Passive smoking
Overcrowding
Malnutrition
9. wt is the common age group affect by bronchiolitis?
1M -9M
10.wt are the causative pathogens of bronchiolitis?
RSV- 80% of all cases
human metapneumovirus ,parainfluenza virus , rhinovirus ,
adenovirus , influenza virus

Bronchiolitis is commonest serious respiratory infection of infancy.

Pathogen: RSV 80% of the case, also human metapneumo virus.

Page | 32
FMAS, RUSL
UNDER THE GUIDANCE OF DR. Anurudhdha Padeniya
Done by Kumudu Kumari and Akila Rathnayaka 07/08 Edittion by Muditha 08/09, 3rd Darshanee and Ridma
09/10
07. Heart Failure -Poor Exercise tolerance/ lethargy/Malaise/Orthopnea.
Bronchial asthma Tachypnea,tachycardia,hepatomegaly
scalp sweating in feeding(younger)
08. Structural anomaly/Congenital lesions of the respiratory tract

Presentations: Tracheo-esophagial fistula


Cough Persistent of symptoms
Bronchial/Tracheal stenosis
Wheezing(Major Clinical Expression) Tracheomalesia since birth
DIB Laryngomalecia
Chest tightness 09. Foreign Body inhalation (Intra bronchial) -Previously well child
DD/ 10. Upper Respiratory tract Infections -Allergic Rhinitis / Adenotonciler hy-
For Cough & Wheeze in infants & Children pertrophy/Sinusitis.
01. Bronchial Asthma - symptoms are frequent and recurrent.child is Well In- 11. Tumors, mediastinal masses
between episode/ worse at night and in the early morning/ Triggering fac- 12. Loffler’sSyndrome
tors for episode like exposure to cold,pets,damp air emmortions,laughter/ 13. Rarecases
family history of atopy, asthma -cystic fibrosis/
02. Bronchiectasis - cough with foul smelling sputum production/FTT/ Club- -cilliary dyskinesia - Recurrent Lower Respiratory tract infections/failure
bing to thrive/chronic sinus infections
03. GORD - Symptoms associated with meals, associatedregurgitation, burn- -Immuno deficiency
ing epigastric pain, Abdominal Discomfort.
04. Recurrent aspirations -NeurologicallyImpaired Children, Oropharyngeal
History:
incoordination or associated GORD
05. TB -Cough,Contact History of TB/ Cough with sputum/haemoptysis,
evening pyrexia, Night Sweating /FTT. P/C Diagnosed patient with asthma with an exacerbation
06. Interstitial lungdisease – Long standing history Recurrent episode of Cough / Wheeze

Page | 33
FMAS, RUSL
UNDER THE GUIDANCE OF DR. Anurudhdha Padeniya
Done by Kumudu Kumari and Akila Rathnayaka 07/08 Edittion by Muditha 08/09, 3rd Darshanee and Ridma
09/10
- Day symptoms? Night Symptoms? Their frequency
H/P/C - Acute exacerbations/OPD nebulization/ Hospital admissions/ICU admis-
Key points in History. sions.
i. Describe present episode in details. - Given Treatment/Compliance
Ex: Child was apparently well five days back acute onset–after Expose to - Side effect of Steroids
allergen – Cough & Wheeze. - Past history of heart Disease.
Gradual onset –Associate with respiratory tract infection - Any Congenital Anomalies.
ii. Progression - Allergic rhinitis /eczema/hay fever/sinusitis

iii. PrecedingTriggering Factors/preceding respiratory tract infection/ag-


gravating factors(cold air,airpollutant,RTI,smoke,stress, inhalation, Drug History
exercise,some drugs-b blockers,aspirin)
iv. Associated with atopic - What drug Child is on – Long term prophylaxis or only during attacks,.
v. Diurnal variation-worse at night & early hrs of morning/reduce at day How long
time Blue – How many puffs
vi. What child’s mother done at home –if a diagnosed patient inhaler Brown - How many puffs
used/what type of Inhaler How many puff/ Respond or not. How many days used blue one(salbutamal) with in last
vii. Describe the presentation on admission month
viii. In the Hospital what has done up to now? and describe the present
state of the patient( Current situation-activity,feeding,speech) - Use of Spacer, Face Mask
. - Nebulize treatment at home
01. Exclude DD &Establish the probable diagnosis of asthma. - Oral Drugs (Montelukast in Exercise induce asthma)
- Compliance
P/M/Hx - Check the technic (Never forget)

- Previous attacks -Details,What age/how often/symptoms/severity/how - Improvement/whether spirometry done


many episodes,last hospital addmition - Side effect of drugs- Salbutamol – Tremors, Palpitation

Page | 34
FMAS, RUSL
UNDER THE GUIDANCE OF DR. Anurudhdha Padeniya
Done by Kumudu Kumari and Akila Rathnayaka 07/08 Edittion by Muditha 08/09, 3rd Darshanee and Ridma
09/10
Steroids –Oropharyngel Candidiasis/weight gain/ - General Introductions of Family/Parental Education
Cataract/Cushinoid features. - Impact on the Child -School Missing (Most Common Course for school ab-
- Use of any other drugs that can precipitate asthma, NSAIDS, beta block- sence)
ers. -Playing is reduced in-Exercise Induce Asthma,
- Specially Aminophilline -Foods- Ice Cream, Yogurt, cool drinks
-Bathing (Days per week),
Birth History -sleeping
- Impact on parents
- Maternal Antenatal Smoking/ Preterm Delivery (asthma is common - Hospital state,
among preterm) - other Siblings, Normal Family Activities, effects on income

ImmHx - Home environment-nearby gravel road,mills ,wodden work shop,


- Up to date - Housing Condition–Wall, Floor, Roof, Sealing, Kitchen, and Floor – How
often Swept/Mopped, Covering of windows

Growth History - Bed rooms – Numbers, Ventilation, How many sleep in Single room.
- Cleaning of bed sheets/pillow cases /mattresses, how often they changed.

- Chronic disease, if FTT present think of DD- Bronchiectasis, CCF, - Environment


o -Main roads (carpet or gravel) near to the house/ Dust exposer.
o -Factories & Bakeries
Family History
o -School Environments
- Cooking fumes ,mosquito coils, pets, soft toys, smoking
- Atopy- eczema, Allergic Rhinitis, Allergic Conjunctivitis
- Parental Education regarding
(Childhood asthma is more common among children who are having
- drugs types and when to use them/Different between
atopic siblings & parents)
preventers&relievers/
Secondary smoking
Social History
Recognition of acute exacerbation,& what to do?

Page | 35
FMAS, RUSL
UNDER THE GUIDANCE OF DR. Anurudhdha Padeniya
Done by Kumudu Kumari and Akila Rathnayaka 07/08 Edittion by Muditha 08/09, 3rd Darshanee and Ridma
09/10
- Intercostal / Sub Costalrecession/chest windrowing
- Evidence of Chronichyperinflation of the lungs
Examinations
- Increased AP Diameter (Barrel Shape)
General
-Harrison’s sulcus
- Position of child(popup) -severity
-Impaired liver& cardiac dullness
- Appearance
Palpation
- Drowsy/confused or not
- Respiratory rate
- Anthropometry
- Trachea
Centiles according to CHDR / Failure to thrive – alternative di-
Percussion
agnosis to bronchial asthma
- hyper resonant
- Respiratory distress
Auscultate
Dyspnea, Cyanosis, Use of Accessory respiratory mussels, nasal
- Breath Sounds(VB)
flaring,audible wheezing
- AE=
Can the child talk.(well/halting)
- Added sounds(Wheezing /ronchy)
- Temperaturechart (Fever), Cannula, Warm to touch, Nasalcongestion,
Polyps,
Cardio Vascular System
- Listen to the cough& examine Throat, Ear, Cervical LN, and BCG Scar.
- PR
- Finger clubbing.
- Pulses paradox
- Tremors due to salbutamol retainsion
- murmur
- Ankle / Sacraledema
- Chronic steroid use- striae, truncal obesity, moon face
Abdomen
- Skin – Atopic eczema
- Upper border of the liver

Respiratory Systems
Inspection

Page | 36
FMAS, RUSL
UNDER THE GUIDANCE OF DR. Anurudhdha Padeniya
Done by Kumudu Kumari and Akila Rathnayaka 07/08 Edittion by Muditha 08/09, 3rd Darshanee and Ridma
09/10
Target Questions Chest x-ray- the diagnosis is uncertain,Severe/life threaten episode
responding poorly (like pneumothorax & pneumonia)
01. What is your diagnosis?
Mildpersistent asthma presented with acute server exacerbation
Hypersensitivity test (not available in SL)

Assessment of Severity of asthma. 02. How you are going to manage acute exacerbation.

Day symptoms Night symptoms Recognition of acute server asthma &life-threatening asthma is the most

Mild 2 or less per wk <2 per month important as a HO

intermittent Refer national guidelines for acute management. You should thorough

Mild >2 per wk but >2 per month with this.

persistent <1 per day


Moderate Daily >1 per month 03 Patient Stabilization & subsequent management

persistent - Consider maintenance fluid if oral intake is poor

Sever Continuous Frequent - Antibiotics if fever is there,

persistent - Then think about,


1. Control of Factors contributing 3. Patient educations
Objective Tests: to asthma
•> 5 years of age. 2. Asthma Pharmacotherapy 4. Regular assessment & follow
• Peak expiratory flow (PEF)
up.
Forcedexpiratory volume in the first second (FEV1).
• Variability of PEF and FEV1, either spontaneously over time or in
Response to bronchodilator therapy is a characteristic feature of Goals of asthma therapy in children
Asthma.
• The percentage PEF variability - 20% or more is Minimal, ideally no, symptoms during the day or at night
Highly suggestive of asthma Minimal, ideally no exacerbations
Minimal use or no necessity for the use of reliever short acting β2 agonist
PEF variability (Amplitude % best) = (Highest-lowest)x 100
Highest FEV1 and/or PEF over 80% of personal best or predicted normal
Minimal, ideally no adverse effects from medications

Page | 37
FMAS, RUSL
UNDER THE GUIDANCE OF DR. Anurudhdha Padeniya
Done by Kumudu Kumari and Akila Rathnayaka 07/08 Edittion by Muditha 08/09, 3rd Darshanee and Ridma
09/10
Normal activities and rare school absences Poor compliance
Optimum growth of the child Inadequate dose
Minimal effects on other family members Poor environmental control
Inappropriate dose
Poor environmental control
Discharge
Improper technique
- Sustained improvement in symptoms
Inappropriate pharmacological management
- Stable on oral/inhaled bronchodilator ± steroid therapy,
Wrong diagnosis
- SpO2 > 92% in room air for 4 hours Regular assessment and follow up
- PEF more than 75% of predicted normal.
Sign & symptoms of asthma
Long turn management
- Persistent asthma is most effectively controlled with dailyanti-inflammatory Pulmonary functions
therapy.
Quality of life / functional status
- A stepwise approach to pharmacological therapy is recommended to gain and
maintain control of asthma. Symptom free days
- The amount and frequency of medication is dictated by asthmaseverity. Cool drinks/ice cream
- Long term treatment should be continued at least for a periodof 6/12.
- There are two appropriate approaches to gain control ofasthma with inhaled
School attendance

steroids Playing
1. Step up method
Adequacy of management-drug step up /step down in 3 month ,envi-
2. Step down method ronmental modifications
- Regular follow up visits at 1 to 3 monthly intervals are necessary
(Refer- GUIDELINES FOR MANAGEMENT OF ASTHMA p121)
SE of drugs

If control is not achieved with in 1 month consider stepping up Asses weight / height

1st rule out followings BP

Page | 38
FMAS, RUSL
UNDER THE GUIDANCE OF DR. Anurudhdha Padeniya
Done by Kumudu Kumari and Akila Rathnayaka 07/08 Edittion by Muditha 08/09, 3rd Darshanee and Ridma
09/10
Encourage exercecise > 5 years - MDI + Spacer device / dry powder inhaler (DPI)
Adequate diietary Ca2+ > 8 years - MDI alone may be possible
Ophthalmological assessment

Indicators that asthma can continue to adult life

Stronger predictors-atopy Advices


Onset <3 yrs Good health measures
Increased frequency & severity of childhood wheezing Nourishing, non allergic diet
- Indications for regular medications for regular medications in bron- No restriction on food intake
chial asthma
Adequate rest & sleep
- Persistent asthma
Reasonable physical activity &exercise(no restriction on play-
- After episode of life threatening asthma
ing/Swimming increase inhalation &help liquefy secretions
- Resent increase in severity or frequency of acute exacerbations &enhance vital capacity)

- Nocturnal asthma during sleep Environmental modification

- Frequent episodes of asthma Wet mopping of the floor

- Sever exercise induce asthma Change/wash bed linen regularly

- Incurability of medical care Cover pillow ,mattress with polythene or use form
rubber

Removal of cobwebs
Appropriate device
< 2 years - metered dose inhaler (MDI) + Holding chamber Prevent fumes form vehicles & kitchen coming to
2-5 years - MDI + Spacer device (with a face mask up to 3 years) baby’s room

Page | 39
FMAS, RUSL
UNDER THE GUIDANCE OF DR. Anurudhdha Padeniya
Done by Kumudu Kumari and Akila Rathnayaka 07/08 Edittion by Muditha 08/09, 3rd Darshanee and Ridma
09/10
Factors of avoid Make a functional chimney to avoid gathering of cooking fumes within the
house.
Pets like cats ,dogs
Try to avoid play with cats and dogs.
Smoking in family
Inhaler usage
Food stuff with preservatives(Tipi tip)
Less side effects than using oral drugs.
Coils, mat for mosquitoes
Check wheather it is functioning before use
Extremes of temperature
Assemble the spacer and mask and fit the mask into the mouthpiece.check
Individual with RTI whether the spacer is clean .

Others Shake the inhaler well and fit the inhaor l to the inhaler slot of the spacer.

Bathing-if precipitating reduce time Position the child in standing or back strait position.do not use in supine
position.
Instruction about acute episode-give max bronchodilator
inhaler/tablets Hold the mask firmly over the mouth and nose and keep the inhaler and
spacer in horizontal position.press the inhaler and ask to take a deep bredth
Check for response-RR,IC/SC recessions,colour,difficult and hold it or take several breaths.
in speaking
After using wash the mouth thourouly to avoid oral ulcers.
Bring early to the hospital
Clean the spacer and mask with tap water and air dry.do not rub with
Counseling clothes.store it in a plastic containor and clean the mask and spacor twice
a week and change the spacer in six months time.
Identify the triggering factors and avoid them such as dust, fumes,grass pollen

Clean the house and environment frequently.


Maintain an asthma diary.document it daily.it should include symptoms of
the chiid such as fever,wheezing,cough and cold the type of inhaler
Change pillowcases,bed sheets ,nets once a week. used.(the days that blue one used)

Make adequate ventilation within the house and bedrooms.

Page | 40
FMAS, RUSL
UNDER THE GUIDANCE OF DR. Anurudhdha Padeniya
Done by Kumudu Kumari and Akila Rathnayaka 07/08 Edittion by Muditha 08/09, 3rd Darshanee and Ridma
09/10
Inhaler devices Use and care of spacers

Technique and training The spacer should be compatible with the MDI being used.
The drug should be administered by single actuation of the MDI into
Prescribe inhalers only after patients have received adequate and thespacer, each followed by inhalation.
specific
training in the use of the device and have demonstrated satisfactory
technique.
There should be minimal delay between MDI actuation and
inhalation.
< 2 years - metered dose inhaler (MDI) + Holding chamber Tidal breathing is as effective as single breaths.
2-5 years - MDI + Spacer device (with a face mask up to 3 years) Spacers should be cleaned monthly rather than weekly. They should
> 5 years - MDI + Spacer device / dry powder inhaler (DPI) be
> 8 years - MDI alone may be possible washed with liquid detergent and allowed to dry in air. The
mouthpiece
should be wiped clean before use.
Spacers are recommended to be replaced at least every 12 months
but some may need changing every 6 monthes.
Frequency of dosing of inhaled steroids

Prescribe initially twice a day. Once a day inhaled steroids at the


same total
daily dose can be considered if good control is established.

Prescribing devices

The choice of the device may be determined by the choice of the


drug.
If the patient is unable to use a device satisfactorily, an alternative
should be found.
Reassess inhaler technique as part of the clinical review.

Page | 41
FMAS, RUSL
UNDER THE GUIDANCE OF DR. Anurudhdha Padeniya
Done by Kumudu Kumari and Akila Rathnayaka 07/08 Edittion by Muditha 08/09, 3rd Darshanee and Ridma
09/10

Page | 42
FMAS, RUSL
UNDER THE GUIDANCE OF DR. Anurudhdha Padeniya
Done By Fathima Rushida 07/08, 2nd Achini 08/09, 3rd Udara 09/10

Dry/moist/productive
PNEUMONIA
Sputum colour/amount
Presentations
Haemoptysis
Fever
Throughout the day/increased at night
Difficulty in breathing
Sleep disturbance
Preceding upper respiratory tract infection
Post tussive vomiting
Cough, lethargy, poor feeding, unwell child
Pain on coughing & breathing

Hx/P/C
Lethargy, poor feeding
Fever
Ear pain or discharges
Bacterial sudden onset, temperature more in bacterial
Sore throat or tonsillitis
High grade
Abdominal, localized chest or neck pain
Shaking chills
Restless, anxiety, drowsy
Chest pain
Nausea, vomiting, diarrohea, ab.distension
Associated arthralgia, myalgia (atypical pneumonia)
Body rashes

UOP, BO
Cough
Contact hx of fever cough or TB
Onset – sudden/gradual

Page | 43
FMAS, RUSL
UNDER THE GUIDANCE OF DR. Anurudhdha Padeniya
Done By Fathima Rushida 07/08, 2nd Achini 08/09, 3rd Udara 09/10

Extra pulmonary manifestations in atypical pneumonia P/M/Hx

Hematological- Hemolytic anaemia, thrombocytopenia Recurrent attacks

Renal- Nephritic syndrome Asthma/ TB/ Rhinitis

Skin- Erythema multiforme or Steven Johnson syndrome Joints- Congenital or acquired heart diseases
Arthritis History of congenital lung cysts, chronic lung disease, immunodeficiency,
sickle –risk groups
CNS- Peripheral neuritis, CNS Infections, GBS, transverse myelitis,
acute psychoses History of hospital admission

CVS- Pericarditis, Myocarditis

Ocular- Conjunctivitis, anterior Birth Hx

Uveitis, optic atrophy Imm Hx Hib vaccine

Diet Hx

DDs Family Hx

Pneumonia Social Hx
Bronchiolitis (No/Low grade fever,
Examination
rhinorrhoea, wheezy &dry cough, post tussive vomiting, poor feeding)
General
Bronchial asthma (eczema, recurrent wheeze, & family hx of rhinitis,
BA or atopy) Built

Page | 44
FMAS, RUSL
UNDER THE GUIDANCE OF DR. Anurudhdha Padeniya
Done By Fathima Rushida 07/08, 2nd Achini 08/09, 3rd Udara 09/10

Febrile/ not

Hydration Inspection

Dyspnoea Tacypnoea- most consistent feature of pneumonia

Posture (to a side in pleuratic chest pain) Intercostal, Subcostal, Suprasternal, Supraclavicular recessions

Cannula, Nasal prongs Nasal flaring

Nose- Rhinitis, polyps Grunting

Pallor/cyanosis/Conjunctivitis Accessory muscle usage

Ear & throat

Lymph nodes Palpation

BCG scar, Mantoux scar Symmetrical movement/ not


Skin rashes ,Joint swelling Trachea

CVS Percussion

Tachycardia Dullness on percussion over affected area

BP (Later as complications develops

Exclude heart disease Eg: Pleural effusion , Empyema

Pneumothorax

RS

Page | 45
FMAS, RUSL
UNDER THE GUIDANCE OF DR. Anurudhdha Padeniya
Done By Fathima Rushida 07/08, 2nd Achini 08/09, 3rd Udara 09/10

Auscultation Sensory loss

End inspiratory coarse cackles Focal neurological signs

Wheezing

Bronchial breathing
Investigations
Diminished breath sounds
CXR

Consolidation - Lobar consolidation in bacterial pneumonia


(Dullness, Bronchial breathing, Diminished breath sounds are absent in
young children) Pleural effusion

Empyema

Abdomen

Abdominal distension May confirm the diagnosis. But can’t differentiate between bacterial &
viral pneumonia except classic lobar pneumonia
(gastric dilatation from swallowed air

& ileus)
WBC- Useful in differentiating bacterial from viral pneumonia
Enlarged liver
Viral- WBC normal/ increased
(Downward displaced diaphragm due
Not > 20000 ,Lymphocyte predominance
to hyperinflated lung or superimposed congestive cardiac Failure)
Bacterial- WBC increased 15000-40000

Granulocyte predominance
CNS

Features of GBS

Page | 46
FMAS, RUSL
UNDER THE GUIDANCE OF DR. Anurudhdha Padeniya
Done By Fathima Rushida 07/08, 2nd Achini 08/09, 3rd Udara 09/10

Acute phase reactants – CRP Obtain IV access & take blood for FBC,CRP & blood culture

Will not differentiate bacterial Consider IV fluids if child can’t take orally

from viral Correct any dehydration or deficit (But should be done cautiously, SIADH
Secretion is a possibility)
Definitive diagnosis
Manage fever & pain with PCM
Viral infection- Isolation of virus or detection of viral
genome/ antigen in respiratory tract secretion Start monitoring chart including PR, RR, BP, oxygen saturation &
respiratory symptoms & signs of the child
Bacterial infection-

Blood culture & ABST


Antibiotic management
Pleural fluid culture & ABST

Cold agglutinin test- Mycoplasma pneumonia


If the clinical diagnosis is bronchopneumonia or lobar pneumonia
empirical treatment is necessary to reduce the morbidity and mortality
since the possible organisms are difficult to predict.
Management
Intravenous antibiotics should be used in pneumonia when the child has
If child is in severe respiratory distress signs of severe pneumonia and vomiting which disturbs oral intake [D].

(Tachypnoea, intercostal & subcostal recessions, restlesness & drowsiness


may indicate severity of pneumonia) Suggested initial empirical treatment

Asses A,B,C 0- 3 months

Measure oxygen saturation

Oxygen should be administered if oxygen saturation< 92% IV Penicillin or Ampicillin and Gentamicin as first line therapy

Page | 47
FMAS, RUSL
UNDER THE GUIDANCE OF DR. Anurudhdha Padeniya
Done By Fathima Rushida 07/08, 2nd Achini 08/09, 3rd Udara 09/10

Cefotaxime or Co amoxiclav as second line therapy >5 years

3 months to 1 year If the child is not ill - oral Macrolides (Erythromycin, Azithrmycin,
Clarythromycin)

If toxic or lobar infiltrates with or without effusion – treat as for 1-5 yr old
If the child is not ill – oral Amoxycillin
No improvement or chest x- ray / clinical findings are ambiguous – add a
If ill or lobar infiltrate in chest x-ray – IV Ampicillin Oral macrolide
If no response within 48 hours use 2nd or 3rd generation Cephalosporins
(eg. Cefuroxime or Cefotaxime)

If Staphylococcal infection is suspected add IV Cloxacilin Duration of the antibiotic therapy should be 5- 7 days and in severe cases,
10 days.
If MRSA is suspected add IV Vancomycin

After starting oral therapy if the condition is not improving after 48 hours
1-5 years the child should be admitted to hospital for further management

Intravenous antibiotics could be switched to oral antibiotics, when the


temperature falls and breathing difficulty is resolving
If the child is not ill - oral Amoxycillin or penicillin

If ill or lobar infiltrate with or without effusion in chest x-ray–IV


Penicillin

If no response within 48 hours use 2nd or 3rd generation Cephalosporins Management of specific pneumonias
(eg. Cefuroxime or Cefotaxime)
Staphylococcal
If Staphylococcal infection is suspected add IV Cloxacillin

If MRSA is suspected add IV Vancomycin


Cloxacillin IV 100mg –200mg/kg/day 6 hourly

Page | 48
FMAS, RUSL
UNDER THE GUIDANCE OF DR. Anurudhdha Padeniya
Done By Fathima Rushida 07/08, 2nd Achini 08/09, 3rd Udara 09/10

Cefuroxime IV 100-150mg/kg/day 8 hourly 12) What is pneumonia?

Vancomycin IV 40mg/kg/day 6 hourly in Inflammation of he paranchyma of the lung


an infusion if MRSA is suspected One of the leading cause of morbidity & mortality in childhood

13) What are the types of pneumonia?


Haemophilus influenza Community acquired pneumonia
Ampicillin IV 100mg/kg/day 6 hourly Hospital acquired pneumonia
About 30% produce beta lactamases against Ampicillin Pneumonia in immunocompromised child
Ceftriaxone IV 100mg/kg/day
14) What are the pathogens causing pneumonia?

1) The age is a good predictor of the likely pathogen


Klebsiella pneumonia
Neonate Gp B Streptococcus, E. coli,
Cefotaxime IV 100mg/kg/day 8 hourly
Klebsiella, Enterobacteriaceae,

Listeria monocytogens
Chlamydia trachomatis
1 m to 1 year RSV, parainfluenza, Chlamydia
Erythromycin 40mg/kg/day 6 hourly
trachomatis,S.Pneumoniae,
If conjunctivitis is suspected Erythromycin or Tetracycline eye drops S. aureus, B. pertussis

1- 5 years S. pnumoniae, S. aureus,


Target Questions H. influenzae b, Influenza,

Parainfluenza , Mycoplasma, ,
M.tuberculosis RSV,

Page | 49
FMAS, RUSL
UNDER THE GUIDANCE OF DR. Anurudhdha Padeniya
Done By Fathima Rushida 07/08, 2nd Achini 08/09, 3rd Udara 09/10

Klebsiella pnemoniae Acinobacter

Less commonly E.coli Legionella


GpA Strepocooccus

Moraxella
15) Features of different aeiolotiology
>5years S. pneumoniae, Mycoplasma

pneumoniae,Chlamydia Bacterial lower respiratory tract infection

pneumoniae, M.tuberculosis Fever more than 38.5°C.


Respiratory rate above the appropriate upper limit for age
In infants and children <5years of age commonly caused by viruses. Chest recession.
Wheezing is not a sign of primary bacterial LRTI (other than My-
In children >5years of age Streptococcus pneumonia is the most common coplasma).
bacterial cause. Other viruses may be concurrent.
Clinical and radiological signs of consolidation rather than col-
lapse
Organisms causing atypical pneumonia

Mycoplasma pneumoniae Chlamydia


pneumoniae Viral lower respiratory tract infection
Legionella pneumoniae Infants and young children.
Wheezing.
Fever lesser than 38.5°C.
Marked chest wall recessions.
Organisms causing hospital acquired pneumonia Hyperinflation.
Respiratory rate normal or raised.
Radiograph shows hyperinflation and, in 25%, patchy collapse.
Lobar collapse when severe.
Common Uncommon
Pseudomonas aeruginosa Serratia

Page | 50
FMAS, RUSL
UNDER THE GUIDANCE OF DR. Anurudhdha Padeniya
Done By Fathima Rushida 07/08, 2nd Achini 08/09, 3rd Udara 09/10

ENT - bullous myringitis ( over 2 years)

Mycoplasma pneumonia
Four distinct patterns are recognized in chest x- ray of mycoplasma
Mycoplasma pneumonia is a disease of gradual and insidious onset of pneumonia
several days to weeks.
Reticulonodular opacification often involving a single lower lobe
Clinical findings that raise the suspicion of Mycoplasma pneumonia in is the
community acquired pneumonia (CAP commonest pattern (75%).

Focal or bilateral reticulonodular pattern of lower lobes in a


clinically suspected case is suggestive of the diagnosis.
Dry paroxysmal cough or wheezing in an older child
Low grade fever (less than 38.5 oC ) Lobar consolidation is rare.
Prominent headache or myalgia and arthralgia
Illness lasting more than 1 week Hilar adenopathy around 30%
Extra pulmonary manifestations Plate like atelectasis noted as thin, flat areas of collapsed lung, of-
Poor response to routine antibiotics used in CAP ten
seen in lateral films.
Nodular infiltration
Uncommon features of mycoplasma infection.

Rhinorrhoea and nasal congestion


Pleural effusion
Pulmonary abscess

Extra-pulmonory manifestations of mycoplasma

Haematological - haemolytic anaemia , thrombocytopenia 16) Tachypnoea with chest indrawing is the best predictor of pneumo-
Renal - nephritic presentation nia of children in all age groups.
Skin - erythema multiforme or Steven Johnson syndrome
Joints - Arthritis Normal RR according to age& when consider as tachypnoea
Nervous system - peripheral neuritis, CNS infections, Guillain
Barre syndrome, transverse myelitis, acute psychoses
Cardiovascular - pericarditis, myocarditis
Ocular - conjunctivitis, anterior uveitis, optic atrophy
Age Normal Tachypnoea
Page | 51
FMAS, RUSL
UNDER THE GUIDANCE OF DR. Anurudhdha Padeniya
Done By Fathima Rushida 07/08, 2nd Achini 08/09, 3rd Udara 09/10

Neonate 30-50 > 60 Look at the monitoring chart-RR, PR, BP &


Infants 20-30 >50
Young children 20-30 > 40 oxygen saturation
Elder children 15-20 > 30
Look at the latest investigations

Features of respiratory distress 19) Poor response to treatment. Why?


Clinical response to treatment should take no more than 48-72 hrs. If
Chest wall recessions he child is still unwell after this period of time, consider the following
factors.
Use of accessary muscles
Is the child receiving the appropriate dose of appropriate antibiot-
Cyanosis ics
Assess the compliance to he medication
Grunting
Check if he child receiving the antibiotic or has he child vomited
the medication
17) Complications
Is he diognosis is correct?
Pleural effusion Has the child developed any other complications of pneumonia?
Effusion , Empyema ,Pneumothorax
Empyema Are there any hos factors predisposing to he poor response?
Other causes for continuing fever
Pneumothorax
20) How do we assess the severity of the disease?
Mild Severe

18) What would you look for in this patient on your daily ward
round? Infants Temperature<38.5c Temperature>38.5c

General condition of the child RR<50 breaths/min RR 70 breaths/min

Examine the respiratory system of th child Mild recession Moderate to severe

Page | 52
FMAS, RUSL
UNDER THE GUIDANCE OF DR. Anurudhdha Padeniya
Done By Fathima Rushida 07/08, 2nd Achini 08/09, 3rd Udara 09/10

recession Chronic lung disease.

Taking full feeds Nasal flaring Immunodeficiency.

Cyanosis Cystic fibrosis.

Intermittent apnoea Sickel cell disease.

Grunting respiration 22) What is nosocomial(hospital acquired) pneumonia?


Pulmonary infiltrates occurring in a patient who has been hospitalized
Not feeding for
1 week or more which is compatible in appearance with a bacterial
Pneumonia.
Older Temperature<38.5c Temperature>38.5c

ChildrenRR< 50 breaths/ min RR> 50 breaths/min 12)What are the risk factors for nosocomial pneumonia?

Mild breathlessness Severe difficulty in Aspiration

breathing Intubation

Nasal flaring Bacteraemia

Cayanosis 13)What are the common pathogens for nosocomial pneumonia?

Grunting respiration Pseudomonas aeruginoa


Klebsiella pneumoniae
Signs of dehydration E.coli
14) what are the indicators for admission to hospital in infants
Oxygen saturation (SaO2) less than 92%, cyanosis
21) Who are the risk groups for pneumonia? Respiratory rate over 70 breaths/min
Congenital lung cysts. Difficulty in breathing
Intermittent apnoea, grunting

Page | 53
FMAS, RUSL
UNDER THE GUIDANCE OF DR. Anurudhdha Padeniya
Done By Fathima Rushida 07/08, 2nd Achini 08/09, 3rd Udara 09/10

Poor feeding (approximately reduction by half)


Family notbeingable to provide appropriate observation or supervision.

15) what are the indicators for admission to hospital in older children
Oxygen saturation (SaO2) less than 92%, cyanosis
Respiratory rate over 50 breaths/min
Difficulty in breathing
Grunting
Signs of dehydration
Family not beingable to provide appropriate observation or supervision

Page | 54
FMAS, RUSL
UNDER THE GUIDANCE OF DR. Anurudhdha Padeniya
Done by Kasun 07/08, 2nd edition by Kumudu 08/09, 3rd edition by Irosha 09/10

Hx/P/C→
Acute Gastroenteritis
Presentations:
Symptom analysis of the presenting complain -Diarrheoa
Diarrhea usually lasting for 5 to 7 days ( can be lasted in two
weeks) o Onset

Vomiting lasting for 1 to 2 days (can be gone for 3 days ) o Duration/ frequency

Can be associated with mild fever o Watery/,mucoid(commonly viral origin)/blood and


mucous(bacillry dysentry)

DD’s: o Amount

o Medical o smell

Acute gastroenteritis o Expulsivor not

Meningitis o Associated with fever, chills ,rigors, vomiting, ab-


dominal pain, tenesmus
UTI
Vommiting
o Surgial

Intestinal obstruction o Onset

Appendicitis o Frequency

o Colour of vomitus

o Amount

Page | 55
FMAS, RUSL
UNDER THE GUIDANCE OF DR. Anurudhdha Padeniya
Done by Kasun 07/08, 2nd edition by Kumudu 08/09, 3rd edition by Irosha 09/10

o Projectile or not What mother did upto now, what was the
management at hospital upto now

o Continue breast feeding

o complementary feeding
Exclude and narrow down the diagnosis
o ORS
o Altered consciousness
o IV fuids
o Seizures
o Any other medication given,
o Crying while micturition.

o Abdominal distension
P/M/Hx→
Etiology
o Recurrent episodes
o Recent contact HX of AGE.

o Recent exposure to a known or risk source of infec-


tion (foods or water) o How many previous attacks

o If suspected food poisioning,time duration of sym- o Any chronic disease


poms since food ingestion ,what happen o the others
who ate those food Birth Hx→

Complications

o Lethargy, drowsiness, thirst (dehydration) Imm Hx→

Page | 56
FMAS, RUSL
UNDER THE GUIDANCE OF DR. Anurudhdha Padeniya
Done by Kasun 07/08, 2nd edition by Kumudu 08/09, 3rd edition by Irosha 09/10

o Childs and parents hand cleanliness.(how often nail


clipping)
Diet Hx→
o Toilet training and practices

o 24 dietary recall.
Examination:
o Palatability of the foods
General→
o Breast feeding.(try to asses wether the quality and
quantity of the foods child is taking will meet the cal- o Anthropometry
orie requirement ~mal nutrition and infection vicious
cycle) o General condition

o Well or ill looking

Social Hx→ o Active alert irritable

o Hydration ( shrunken fontanella, eyes, mouth, skin


pinch)
o Educational level of the parents
o Fever, pale, rashes
o Personal hygiene of other family members
o Signs of bleeding manifestations
o Environmental risk factors for diarrhea around house-
flies and garbage o Ground itch

o How get water CVS→

o Foods preparation methods o BP,PR,CRFT

Page | 57
FMAS, RUSL
UNDER THE GUIDANCE OF DR. Anurudhdha Padeniya
Done by Kasun 07/08, 2nd edition by Kumudu 08/09, 3rd edition by Irosha 09/10

RS→

Tachypnoic Investigations:

Laboured breathing Examination of stools

Macroscopy (colour, consistency)

Stool culture and ABST

o Severely ill and septic


Abd→
o Blood and mucus stool
o Distention
o immunocompromised
o Localization of the pain
SFR
o Visible peristalsis

o Tenderness/rebound tenderness
Reducing substance
o Anal perianal excoriation
RBC
(by doing an abdominal examination we can take
ideas mainly for surgical causes.) Amoeba, ova, cyst
CNS→ Pus cells
Bulging fontanelle in infants. Epi cells
Altered consciousness level in a child whose GCS Fat globules
level can be assessed
UFR

Page | 58
FMAS, RUSL
UNDER THE GUIDANCE OF DR. Anurudhdha Padeniya
Done by Kasun 07/08, 2nd edition by Kumudu 08/09, 3rd edition by Irosha 09/10

CRP Antibiotics

If severe dehydration PCV,SE,BU QHT

FBC and platelets (in E.coli in fection – heamoliyic ureamic Fluid balance chart
xn)
Weight chart

Target Questions
Management:
1. What is Diarrhoea?
Admit the child in diarrhea side Passage of unusually loose or watery stools (>3 times
per day)
Assess the degree of dehydration Consistency of the stool is more important than
the frequency
Hypovolemic shock/mild/moderate/severe
dehydration 2. Aeitiology
Infections
Enteric –
o bacterial
Fluid replacement accordingly o viral
o parasitic
Stools for reducing substances o fungal
Other local and systemic infections (UTI, otitis
If negative continue breast feeding
media, URTI, hepatitis A)
Replacement of ongoing loss Drugs - (antibiotics- ex:-amoxycillin)
Surgical conditions - intussusception
Zn supplement

Page | 59
FMAS, RUSL
UNDER THE GUIDANCE OF DR. Anurudhdha Padeniya
Done by Kasun 07/08, 2nd edition by Kumudu 08/09, 3rd edition by Irosha 09/10

Nutritional (overfeeding, food allergy, food poi- - Pseudomembranous colitis


soning)
5. Aeitology watery Diarrhoea
o Diarrhoea is not always due to gastro-enteritis
Viral(rota(commnest)norvo,adeno,corona)
Bacterial(salmonella, cholera, clostridium perfrnigens, bacil-
lus, e coli)
3. Clinical features of bacterial,viral,malabsorptive diar- Drugs
rhea Food poisoning
Allergic
Parenteral

6. Complications of gastroenteritis
Dehydration shock death
Seizures
Electrolyte disturbances
Septicaemia
Haemolytic uraemic syndrome
Anal excoriation
Perforation of bowel
Acute renal failure
Malnutrition
Secondary lactose intolerance
4. Aeitiology of Blood and Mucus Diarrhoea Post gastro-enteritis syndrome

Bacterial infections –(campylobacter,shigella,Enteroinva-


sive/haemorragic e coli
Protozoal infections – Amoebic dysentery 7. Assessment of a child with dehydration
Surgical causes – Intussusception
Inflammatory bowel diseases General condition – (Ex:- sensorium)
- Ulcerative colitis Mucous membranes

Page | 60
FMAS, RUSL
UNDER THE GUIDANCE OF DR. Anurudhdha Padeniya
Done by Kasun 07/08, 2nd edition by Kumudu 08/09, 3rd edition by Irosha 09/10

Urine output
Skin pinch test
Depressed fontanelle 9. Fluid therapy
Thirst 1. Correct existing water and electrolyte deficit
2. Provision of normal daily requirement
Weight reduction (if earlier weight known)
3. Replace ongoing losses
Circulatory status-
capillary refill time
pulse rate, volume, peripheral pulses Fluid Deficit
blood pressure Deficit (ml) = % Dehydration x Body weight (kg) x 1000

Fluid therapy in some dehydration


8. Oral rehydration solution and its contents
ORT can be used successfully in most instances
IV fluids if oral fluids are not tolerated
In mild-moderate dehydration requirement of ORS in the
first 04 hours = 75ml/kg

Severe Dehydration
Give IV fluids immediately
If oral fluids are tolerated give ORS until IV line is ready
100 ml /kg of ringer lactate solution is given according to the
chart

Re assess the patient every 1-2 hours and if not responding


increase the drip rate

Page | 61
FMAS, RUSL
UNDER THE GUIDANCE OF DR. Anurudhdha Padeniya
Done by Kasun 07/08, 2nd edition by Kumudu 08/09, 3rd edition by Irosha 09/10

If patient is in shock manage accordingly o children with more than 5 diarrheal stools in the pre-
In hyper-natremic dehydration correct the dehydration vious 24 hours
slowly
o children with more than 2 vomits in the previous 24
hours

Post gastro-enteritis syndrome o children who have not been offered or have not been
able to tolerate supplementary fluids prior to presen-
Certain infections may harm the brush border and thus the
tation
brush border enzymes
After a bout of diarrhoea re introduction of feeds may result
o infants in whom breastfeeding has stopped during the
in a watery diarrhoea
illness
Stools will contain reducing sugars
Switch back to ORS for 24 hours and reintroduce feeds
o children with signs of malnutrition
Usually reversible

During direct or remote assessment ask whether:

the child has seemed to the carer to be unwell


Clinical assessment
there has been excessive or unaccustomed irritability or leth-
Recognize the following as being at increased risk of
argy
dehydration:
the child has seemed unusually thirsty
o children aged less than 2 years of age, with even
greater risk for those aged less than 6 months there has been a reduction in urine output

o infants who were of low birth weight the child’s appearance has changed (e.g., sunken eyes)

the skin colour is normal

Page | 62
FMAS, RUSL
UNDER THE GUIDANCE OF DR. Anurudhdha Padeniya
Done by Kasun 07/08, 2nd edition by Kumudu 08/09, 3rd edition by Irosha 09/10

the hands and feet are warm • continue usual fluids, including breast or other milk feeds

• encourage the drinking of plenty of fluids

Assess hydration with table 1 in order to: • offer ORS as supplemental fluid for those at increased risk of
dehydration:
1 classify children as non-dehydrated, clinically dehydrated or
shocked o children less than 2 years of age, especially those aged less
than 6 months
2 use red flags as warning signs for increased risk of progres-
sion to shock. o infants who were of low birth weight

o children with more than 5 diarrhoeal stools in the previous


24 hours

o children with more than 2 vomits in the previous 24 hours

o children with signs of malnutrition.

Treatment of dehydration

Rehydrate children, including those with hypernatraemia, using


ORS unless they are in shock.

Oral fluids – optimal composition and administration


Fluid management
Use low osmolarity ORS (240–250 mOsm/l) for oral rehydration
Primary prevention of dehydration therapy.

In children with gastroenteritis but without clinical dehydration:

Page | 63
FMAS, RUSL
UNDER THE GUIDANCE OF DR. Anurudhdha Padeniya
Done by Kasun 07/08, 2nd edition by Kumudu 08/09, 3rd edition by Irosha 09/10

In children with clinical dehydration, including hypernatraemic 2 if, despite appropriate ORT, there are signs of deterioration
dehydration: with red flag symptoms or signs of dehydration.

o treat with low osmolarity ORS Treat shock with a rapid intravenous infusion of 20 ml/kg of 0.9%
sodium chloride solution.
o give 50 ml/kg of ORS over 4 hours in addition to mainte-
nance fluids

o administer the fluid frequently and in small amounts If the child remains shocked:

o consider supplementation with their usual fluids (including 1 give another rapid intravenous infusion of 20 ml/kg of 0.9%
milk feeds or water, but not fruit juices) if they refuse to take ade- sodium chloride solution
quate quantities of ORS and do not have red flag symptoms or signs
of dehydration 2 consider other possible causes of shock.

o consider administration of ORS via nasogastric tube if they If IVT is required for rehydration of non-shocked children:
are unable to drink ORS or vomit persistently 3 use 0.9% sodium chloride with 5% glucose as the initial in-
o monitor the response to ORT by regular clinical reassess- fusion fluid
ment. 4 give 50 ml/kg of intravenous fluid over 24 hours (48 hours in
hypernatraemic dehydration) in addition to maintenance fluids
Intravenous fluid therapy (IVT)

Use IVT for dehydration: 5 give an additional bolus of 5–10 ml/kg of 0.9% sodium chlo-
ride with 5% glucose for each large watery stool passed
1 if clinical assessment confirms or raises suspicion of shock
6 monitor serum electrolytes

7 consider monitoring acid/base status

8 change to 0.45% sodium chloride with 5% glucose if hyper-


1 if clinical assessment confirms or raises suspicion of shock natraemia or hyperchloraemic acidosis develop.

Page | 64
FMAS, RUSL
UNDER THE GUIDANCE OF DR. Anurudhdha Padeniya
Done by Kasun 07/08, 2nd edition by Kumudu 08/09, 3rd edition by Irosha 09/10

9 During IVT, attempt introduction of ORT and, if tolerated: Do not routinely give antibiotics to children with gastroenteritis.

10 stop IVT and complete rehydration with ORT Give appropriate antibiotic treatment to the following:

11 give 5–10 ml/kg of ORS for each large watery stool passed. those with suspected septicaemia

Preventing recurrence of dehydration those with extra-intestinal metastatic bacterial infection

Following rehydration children should be encouraged to drink infants under 6 months of age with salmonella gastroenteritis
plenty of their usual fluids or feed.
malnourished or immune deficient children (including
If dehydration recurs ORT should be recommenced. HIV/AIDS) with salmonella gastroenteritis

those with Clostridium difficile-associated pseudomembra-


nous enterocolitis, dysenteric shigellosis, dysenteric amoebiasis, or
Following rehydration, in those at increased risk of dehydration,
cholera.
give 5–10 ml/kg of ORS following the passage of each large watery
stool. These children include: Consider antibiotic therapy for those recently returned from
overseas travel.
o children less than 2 years of age, with even greater risk for
those aged less than 6 months Why do we offer zn for AGE patient?
o infants who were of low birth weight Zn reduces duration of the acute condition.
o children with more than 5 diarrhoeal stools in the previous Reduces the risk of recurrences for the next few months.
24 hours
Reduce the severity of the acute episode
o children with more than 2 vomits in the previous 24 hours.
PROBIOTICS
1
Non pathogenic bacteria for prevention and a therapy of diarrhea
What is the place for antibiotics?

Page | 65
FMAS, RUSL
UNDER THE GUIDANCE OF DR. Anurudhdha Padeniya
Done by Kasun 07/08, 2nd edition by Kumudu 08/09, 3rd edition by Irosha 09/10

PREBIOTICS Maintenance=4:2:1 regime(hourly regime)

Prebiotics are range of non-digestible dietary supplements, which So during one hour, child will have been receiving =4 x 10
modify the balance of the intestinal micro flora, stimulating the
growth and / or activity of beneficial organisms and suppressing =40ml
potentially deleterious bacteria.
So, one of the first 4 hour, child will receive =40ml +500ml/4

EXAMPLE =165ml

1 yr and 9 mo old baby boy from Isurupura has been brought to the but on the next day child has had diarrhea for 8 stools and vomited
hospital following vomiting for 30 times and diarrhea for 4 times for 10 times. On examination found child’s peripheral getting cold
over one night duration and an associated mild fever. Child was and drowsy.
diagnosed to have AGE. Child UOP became reduced to one time
Management
whole through the day. On examination child was found to have
lethargy and features of clinically dehydrated. (b.w.10 kg) Start iv fluid administration.20ml/kg 0.9% saline bolus

Management So they have given =20 x10

Since this child is only clinically dehydrated we can rehydrate the =200ml
patient orally.

Fluid deficit= 5%{5ml/100g} x10kg x 1000g

=500ml

This should be given to patient within 4 hours and this 4 hr should


be included the volume of maintenance fluid expected to be given
additionally

Page | 66
FMAS, RUSL
UNDER THE GUIDANCE OF DR. Anurudhdha Padeniya
Done by Dinusha 07/08, Harsha 08/09, Kasuni 09/10

Haemolysis, e.g. G6PD deficiency,


Neonatal Jaundice ABO incompatibility
Bruising
Presentations: Polycythaemia
Crigler–Najjar syndrome
Yellowish discoloration of the body / sclera

May associated with Jaundice at >3 weeks of age Unconjugated:


Physiological or breast milk
Dark color urine jaundice
Infection (particularly urinary tract)
Pale color stools
Hypothyroidism
Haemolytic anaemia, e.g. G6PD
SOB (symptom of anemia)
deficiency
DD’s: High gastrointestinal obstruction,
e.g. pyloric stenosis
Depend on the time of presentation
Conjugated (>20% of total
Jaundice starting at <24 h of Haemolytic disorders: bilirubin):
age Rhesus incompatibility Bile duct obstruction
ABO incompatibility Neonatal hepatitis
G6PD deficiency
Spherocytosis
Pyruvate kinase deficiency History:
Congenital infections
P/C

Jaundice at 24 h to 3 weeks Physiological jaundice o Yellowish discoloration of the body and sclera
of age Breast milk jaundice
Infection, e.g. urinary tract infection Hx/P/C

Page | 67
FMAS, RUSL
UNDER THE GUIDANCE OF DR. Anurudhdha Padeniya
Done by Dinusha 07/08, Harsha 08/09, Kasuni 09/10

At what age (because the main cause is different) wet down reflex- when BF from one breast milk comes from
the other breast)
Involved areas (sclera/ hand / feet ect..)
adequate weight gain/significant weight loss in 1st week
How they detected (by themselves or by midwife/ other health care
worker) feeding difficulties(prolong feeding, sleeping while feeding,
cyanosed/cough while feeding)
Associated other features
Bowel opening
Pale stool
(Above questions asked to exclude whether child is adequately
Dark color urine
breast fed or not)
Vomiting Fever /crying/ irritable
Feeding Altered behavior/ vomiting /convulsion- kernicterus
Frequency Features of hypothyroidism- constipation, horse cry ect…..
Duration What has been done up to now
Technique Phototherapy/ exchange transfusion
Urine output/ colure/ frequency/flow Icu/ PBU admission ect……
Crying or straining while passing urine- UTI P/M/Hx
Babies behavior after breast feeding( calm/ crying, irritable)
o Any history of previous hospital admission due to
After breast fed mother feels breast congestion (congestion- jaundice
not adequately breast fed/ empty- adequately breast fed)
o PBU/PICU care
Page | 68
FMAS, RUSL
UNDER THE GUIDANCE OF DR. Anurudhdha Padeniya
Done by Dinusha 07/08, Harsha 08/09, Kasuni 09/10

Birth Hx Blood transfusions

Antenatal history Screen for STD/hepatitis

Detailed antenatal history has to be taken Maternal pyrexia / sepsis/UTI

Maternal age Dribbling ( more than 10 hours)/ PROM/ PPROM/smelly liquor

Parity Whether mother is on any kind of drug- antithyroids

Previous abortions/ still birth Maternal alcohol drug abuse

Blood group of father/ mother Postnatal history

(OA incompatibility- IgG mediated- can cross the placenta, OB Place of delivery ( home/ hospital)
incompatibility- IgM mediated- does not cross the placenta)
Any evidence of fetal distress
Rh (-) or (+)
Delivery method- NVD or LSCS
If Rh (-) rhogum given or not
If NVD instrumentation, forceps, vacuum
Intranatal history
Evidence of cephalhaematoma
Planned pregnancy or not
Syntocinon given or not (SE – neonatal jaundice)
Any history of pv bleeding, MC, invasive procedures, ECV
If LSCS- EM-LSCS or EL-LSCS
USS- any abnormality detected
Term or not
GDM / pre existing DM
Birth weight and general condition of the new born (meconium
Investigations performed aspiration, jaundice at birth, hypoglycemia, anemia

Page | 69
FMAS, RUSL
UNDER THE GUIDANCE OF DR. Anurudhdha Padeniya
Done by Dinusha 07/08, Harsha 08/09, Kasuni 09/10

PBU admission (what happen; ventilated, LP, blood culture, blood


transfusion)
Social Hx
Features of birth asphyxia
o Parental education level
Fetal blood group checked or not
o Economy/ hygiene
o Other investigations perform to newborn (cord blood)
Examination:
o previous jaundice (how Tx, PT complications:-eye
damage, dehydration, diarrhea, bronzed baby, what General
happen after discharge
o Have to do complete neonatal examination
ImmHx
o Weight/length /OFC

o General appearance
Family Hx
o Evidence of sepsis, fever

o Obvious dysmorphic features


known blood related inherited dx
o Fontanels(bulge/not, tense/not)
Repeated blood transfusion
o cephalheamatoma
Early neonatal death
o extend of the jaundice(head and neck-100 mi-
Dx that affect commonly males, result in acute episode of passing cromoles/L, hand and feet- 300 micromoles/L
red color urine
o features of hypothyroidism

Page | 70
FMAS, RUSL
UNDER THE GUIDANCE OF DR. Anurudhdha Padeniya
Done by Dinusha 07/08, Harsha 08/09, Kasuni 09/10

o pallor Investigations:

o cataract & rash (rubella) FBC→

o cleft lip cleft palate o WBC (indicate infection)

o Hb (anemia)

o RBC/HCT (>65 in polycytheamia)

o Retic count (reduced in hemolytic dx)


CVS
Other investigations

Serum bilirubin ( total/ direct /indirect)


RS
Blood picture

o Hereditary spherocytosis (microspherocytosis)


Abd
o G6PD deficiency (bite &blister cells)
o hepatospleenomegaly
o Neutrophil leukocytosis in infection
CNS
Blood grouping /cross match
o Hypotonia
Liver function tests- SGOT/PT, albumin gamma GT
o Planter reflex
Direct comb test (to differentiate the autoimmune conditions)

Septic screen (UFR & Culture, blood culture, LP if indi-


cated, CRP)

Page | 71
FMAS, RUSL
UNDER THE GUIDANCE OF DR. Anurudhdha Padeniya
Done by Dinusha 07/08, Harsha 08/09, Kasuni 09/10

G6pd Essay Anticipated

Serum electrolytes Iv cannula

If Rh (-) mother cord blood for Billirubin, Hb, Direct comb Correct hydration , hypoxia, acidosis and electrolytes imbal-
test, blood group. ance- because it can aggravate the condition

Treatment for infection/sepsis- start appropriate antibiotics

INVESTIGATIONS FOR PROLONG JAUNDICE Phototherapy

Exchange transfusion

SBR- total direct and inderect

FBC- WBC/DC/RBC/Hb Target Questions

Retic count 1) What are the causes for neonatal jaundice?

Blood picture

TFT( TSH/T4) Jaundice starting at <24 Haemolytic disorders:


h of age Rhesus incompatibility
Urine for reducing substances ABO incompatibility
G6PD deficiency
RBS Spherocytosis, pyruvate
kinase deficiency
UFR and urine culture Congenital infection

Management:

Page | 72
FMAS, RUSL
UNDER THE GUIDANCE OF DR. Anurudhdha Padeniya
Done by Dinusha 07/08, Harsha 08/09, Kasuni 09/10

Jaundice at 24 h to 3 Physiological jaundice a. Sign of other disorder; infection, Hemolytic anemia,


weeks of age Breast milk jaundice hypothyroidism, or other metabolic disorder
Infection, e.g. urinary tract infection
Haemolysis, e.g. G6PD deficiency, ABO b. Cause to severe complications; kernicterus
incompatibility
Bruising
Polycythaemia
Crigler-Najjar syndrome 3) Why neonates are more prone to get jaundice

1) RBC life span is markedly shorter than that of udults.


Jaundice at >3 weeks of Unconjugated:
age Physiological or breast milk jaundice 2) There is marked physiological release of Hb from the break-
Infection (particularly urinary tract) down of red cells, because of the high Hb concentration at
Hypothyroidism birth.
Haemolytic anaemia, e.g. G6PD
deficiency 3) Other heam components also high ( metalo enzymes, metalo-
High gastrointestinal obstruction ps, proteins)
hirshprungs
sepsis 4) Liver enzymes are immature
Conjugated (>20% of total bilirubin):
Bile duct obstruction 5) Immature BBB
Neonatal hepatitis
(Depend on the time of presentation) 6) ↑enterohepatic circulation

(↓gut flora/ ↑ gut glucuronidase- 10

times)

2) Why neonatal jaundice is so impotent?


Page | 73
FMAS, RUSL
UNDER THE GUIDANCE OF DR. Anurudhdha Padeniya
Done by Dinusha 07/08, Harsha 08/09, Kasuni 09/10

4) How we manage it convenient, safe, reduce the need of exchange transfusion

Supportive management

Adequate hydration –correct poor milk intake Disadvantages-( mainly side effects)
and dehydration
Diarrhea- isomers-osmoticaly active- drag water
Correct hypoxia, hypothermia, hypoglycemia,
Fluid imbalance
acidosis
Temperature instability (hypo/hyperthermia)
Management of sepsis
Retinal damage (animal studies shows)
Phototherapy
Skin rashes
Exchange transfusion
Tanning of skin

Bronze baby syndrome

7).What are the precautions we have take when giving


5) What are the modes of phototherapy ?
phototheraphy?
single phototherapy & double phototherapy (phototherapu
Add additional fluid
system + Bilirubine blankets)
Input output, stool frequency charts
6).What are the advantages and disadvantages of
phototheraphy? Monitor temperature

Advantages Daily weight

Page | 74
FMAS, RUSL
UNDER THE GUIDANCE OF DR. Anurudhdha Padeniya
Done by Dinusha 07/08, Harsha 08/09, Kasuni 09/10

Cover genitalia(males), umbilical stump and eyes Keep NBM throughout procedure

Make the distance to the baby and beam- 18” Comfortable position

Phototherapy Light (wavelength 450 nm) from the blue- Monitor vital parameters
green band
Fresh screened blood is taken(<5 days)
Maximum effect achieved 24-48 h
Blood have to go through warming cassette
8).What is BIND?
First blood withdraw from the child have to send for basic IX
It is clinical manifestation of of kernicterus, called as bil-
irubin induced neurological dysfunction. Withdraw about10- 20 ml of blood per once

9).What are the indications of exchange transfusion? It depend on child’s wt

Severe inutero haemolysis 1000g- 5ml

Despite of DPT rising SBR 10 mmol/l/h 1000-1500g -10ml

Signs and symptoms suggestive of BIND More than 1500g- 15 ml ect……

10).What do you know about exchange transfusion? Remove blood slowly over 2 min

Two methods Infuse slight faster

Two catheter push pull/ one catheter push pull technique Ultimately twice a volume of infant blood have to be ex-
change transfused
Use umbilical artery/ vein or peripheral artery vein
In term it is 2 (70-90) ml/kg

Page | 75
FMAS, RUSL
UNDER THE GUIDANCE OF DR. Anurudhdha Padeniya
Done by Dinusha 07/08, Harsha 08/09, Kasuni 09/10

Preterm it is 2 (85-110) ml/kg Breast milk jaundice- prolong jaundice due to inhibitors in
breast milk.

13)What are the risk factors for neonatal hyperbilirubinemia


11).What are the complications of exchange transfusion?

Blood born infections


J-Jaundice visible on the 1st day of life
Thromboembolism
A-A sibling with neonatal jaundice or anaemia
Biochemical changes
U –Unrecognised haemolysis (ABO ,Rh or other blood
Haemodynamic changes group incompatibility

Hypo/hyperthermia N-Nonoptimal feeding

Transfusion reaction D-Deficiency of G6PD


In a case of prolong jaundice after acute management patient I-Infection
should follow up by the clinic
Infant of diabetic mother
Mx plan depend on the causative disease
Immaturity(prematurity)

C-Cephalhaematoma
12).What is breast milk jaundice and breast feeding
jaundice? E-East, Mediterranean, Native American heritage

Breast feeding jaundice - ↓ hydration- jaundice

Page | 76
FMAS, RUSL
UNDER THE GUIDANCE OF DR. Anurudhdha Padeniya
Done by Dinusha 07/08, Harsha 08/09, Kasuni 09/10

14)What are the feature of kernicterus? Educate mother that prevention is better than cure and tell how to
prevent it.
Lethargy
o Give breast milk adequately
Poor feeding
o Expose the child to morning sunlight
Irritability
o Protect chid from infections by maintaining good hygiene,
Increased muscle tone good umbilical care and limiting visitors.

Seizures

Coma

15)What ate the advices that you would give to mother?

Informed that they can have jaundice due to many reasons and give
examples like inadequate breast feeding, infections and Rh
incompatibility

Educate mother regarding complications like child can have brain


damage and anaemia.

Inform about the treatment

o Give adequate breast milk

o Give phototherapy

Page | 77
FMAS, RUSL
UNDER THE GUIDANCE OF DR. Anurudhdha Padeniya
Done by N.P.Meegaskanda, 07/08, 2nd edition by Namal 08/09, 3rd edition by lakmali Anandha

Pallor
Biliary Atresia
Bleeding manifestations
Presentation
Features of CLD
• Prolonged jaundice
CVS →
(at 2wks of age, 3wks if preterm)
PR
• Pale stools
BP
• Dark urine
Murmurs
• Bleeding tendency
RS →
• FTT
RR
DD’s
Added sounds
Causes for prolonged jaundice
Abdominal Ex. →
History
Scar over the abdomen if underwent Kasai procedure
Go through the jaundice history
Hepatosplenomegaly
On Examination
CNS →
General →
Features of hepatic encephalopathy
Growth
Investigations
Jaundice
Serum bilirubin
Page | 78
FMAS, RUSL
UNDER THE GUIDANCE OF DR. Anurudhdha Padeniya
Done by N.P.Meegaskanda, 07/08, 2nd edition by Namal 08/09, 3rd edition by lakmali Anandha

Initially conjugated bilirubin ↑ To identify histological changes consistence with


obstruction to differentiate BA from other causes of
Mild or moderate ↑in serum aminotransferases intrahepatic cholestasis
PT/INR Intra operative cholangiogram – gold standard
Disproportionate ↑ in GGT
Management
USS abdomen (fasting ) Surgical bypass of the fibrotic duct
Evaluation of biliary anatomy Hepatoportoenterostomy (Kasaiprocedure)
Exclude other causes of cholestasis Liver transplantation
Ex. choledocal cyst (Biliary atresia is the single most common indication for liver
transplantation in paediatric)
In biliary atresia,
Target Questions
Contracted or absent gallbladder,
though it may be normal What is biliary atresia?
Triangular cord sign Progressive disease, in which there is destruction or absence of the
extrahepatic biliary tree or intrahepatic biliary ducts
Absence of CBG
Incident 1 in 14000 live births
Radioisotope scan with TIBIDA
What are the causes of biliary atresia
Shows good uptake by the liver, but no excretion in
to the bowel at 24 hours Viral – CMV

Liver biopsy Toxic

Page | 79
FMAS, RUSL
UNDER THE GUIDANCE OF DR. Anurudhdha Padeniya
Done by N.P.Meegaskanda, 07/08, 2nd edition by Namal 08/09, 3rd edition by lakmali Anandha

Genetic

Immunologic

What is kasai procedure

Is to restore bile flow from liver to proximal small bowel

A loop of jejunum is anastomosed to the hilum of the liver

If surgery is performed before the age of 60 days, 80% of children


achieved bile drainage

Post-operative complications

Cholangitis

Malabsorption of fat & fat soluble vitamins

Progression to cirrhosis & portal hypertension

Normal anatomy of biliary tract.

Page | 80
FMAS, RUSL
UNDER THE GUIDANCE OF DR. Anurudhdha Padeniya
Done by Udeshika Pinnagoda 07/08, Charith 08/09, Tishni 09/10

o With above presenting complaints mention the dura-


Chronic Liver Disease tion

Presentations: Hx/P/C→

Patients may present with complications of cirrhosis most of o Etiology:


the time as they can remain asymptomatic until liver function
is adequate. 1. Chronic Hepatitis

Lethargy,malaise,poor exercise tolerance, Anemia Post viral-B,C,D(most common)-Hx of fever


with jaundice,Bloodtransfusions,Chronic
Poor feeding,LOAmallnutrition,growth faltering RHC pain,lymphadenopathy
Jaundice,Pruritus,palestools,dark urine
Autoimmune-7-10 years of mean
Abdominal distension with fever age/girls/Associated skin rashes, lupus erythe-
matosus, arthritis,nephritis,haemoyticanaemia
Hematemesis,malena
Drugs (Nitrofurantoin, NSIDS)
DD’s:

For each presentation consider about DD & proceed with Inflammatory Bowel Dx-Alteration of bowel
symptom analysis. habits,mouthulcers,incontinence,growth retar-
dation

Primary sclerosing cholangitis(+/- ulcerative


History: colitis)

P/C→ 2.Wilson’s disease->3yrs,FHx of liverDx(AR),Fx of portal HTN


delayed puberty,amenorrhea

Page | 81
FMAS, RUSL
UNDER THE GUIDANCE OF DR. Anurudhdha Padeniya
Done by Udeshika Pinnagoda 07/08, Charith 08/09, Tishni 09/10

3. Alpha 1 AT deficiency,Cystic Fibrosis, 1ry sclerosing cholangitis Anemia,hemolysis,Thrombocytopenia ,Neutropenia

4. NASH-Overweight children (60%) Mention incidents in chronological order up to admission

5.Cysstic fibrosis. P/M/Hx→

6. 2ry to o Hx of recurrent infections/Plan of liver transplanta-


tion
neonatal liver Dx(biliary atresia,congenital hepatic fibrosis)
P/S/Hx→
Bile duct lesion
o Any surgeries involving hepato-billiarytract,orAb-
dominal surgeries
Complications of CLDx Birth Hx→
Gastro-oesophagealvarices- blood vomiting, malaena o Hx of neonatal jaundice,Congenital hypothyroidism,

Portal hypertensive gastropathy-dyspeptic symptom ImmHx→

Ascites – progressive abdominal distention o Hep.B vaccination

HepatorenalSx- reductionof UOP Diet Hx→

Hepatic encephalopathy- behavioralchanges , confusion o Adequate diet Hx as they prone to develop malnutri-
tion
HepatopulmonarySx
Drug Hx→
Malnutrition,BoneDx
Anticonvulsants,methyldopa, nitrofurantoinerythromycin,
Page | 82
FMAS, RUSL
UNDER THE GUIDANCE OF DR. Anurudhdha Padeniya
Done by Udeshika Pinnagoda 07/08, Charith 08/09, Tishni 09/10

o Other complications(bony abnormalities)

FHx→ o Ulcers,Keysher-flesher rings

o Inherited liver Dx

Social history CVS→

o Discuss the impact of the disease for child, siblings& o Signs of cardiomyopathy(Willson’sDx)
parents
RS→
o Family support
o Mention any abnormality(Hepato pulmonary Sx)
Examination:
Abd→
General→ o Full abdominal examination is needed
o Specialy comment about signs of portal HTN(spider o Abdominal distension,dilatedveins,scratch marks,
navei,caputmedusa,palmarerythema,Duputeren’s
contracture) o Organomegaly

o Hyperbilirubinaemia(icterus, scaling, scratch marks) o Free fluid

o Nutritional deficiencies(leukonoichia, ,Koila- o genitalia


noichia,Pallor,kelosis, CNS→
angularstomatitis,glossitis) o Neurological fx in Willson’sDx is more prominent in
2nd decade of life & include deterioration on school
o Signs of chronic Dx(clubbing,growth)

Page | 83
FMAS, RUSL
UNDER THE GUIDANCE OF DR. Anurudhdha Padeniya
Done by Udeshika Pinnagoda 07/08, Charith 08/09, Tishni 09/10

performance,extrapyramidialsigns,incordina- o autoimmune hepatitis- ANA, anti-smoothmuscle anti-


tion,tremor, dysarthria bodies

o Signs of hepatic encephalopathy-difficulty to diag- o Willson’s Disease- serum ceruloplasmin, 24 hour uri-
nose as in children level of consciousness vary nary copper excretion
throughout the day. Infants-irritability/sleepiness
Older-mood/sleep rhythm/intellectual performance o Hemochromatosis-serum iron study
&behavioural abnormalities.

To assess severity-
Investigations: o FBC: Hb↓/Plt↓/neutropenia
To confirm the diagnosis- o Endoscopy-to detect varicies
Liver function tests-Total,direct,indirect bilirubin levels↑ o Liver Bx-may be difficult because of increased fi-
PT/INR-↑, brosis but may indicate etiology

Total Pr& albumin-↓ o Serum ammonia,EEG-to detect or confirm hepatic


encephalopathy
Liver enzymes(ALT/AST/ALP/Gamma GT)↑

USS of abdomen (shrunken liver & splenomegaly with


varicies) Management:

To find etiology- According to the aetiology-

o Viral hepatitis- hepatitis B&C serology o Viral hepatitis-Antivirals(Hep.B IFN alpha 2b)

o Autoimmune hepatitis-prednisolone &asathioipin

Page | 84
FMAS, RUSL
UNDER THE GUIDANCE OF DR. Anurudhdha Padeniya
Done by Udeshika Pinnagoda 07/08, Charith 08/09, Tishni 09/10

o Wilson’sDx- Zinc + Penicillamine (chelating How would manage an episode of hematemesis in casualty
agent/reduce Cu absorption/facilitate urinary Cu ex- ward?
cretion/detoxifying effect on Cu deposition)Pyridox-
ine Initial resuscitation( High flow O2)

o Cystic fibrosis-Standard supportive & nutritional Leftlateral position to prevent aspiration of blood
therapy with urgodoxycholic acid. For end stage-
Iv cannula
Liver transplantation
Monitor pulse & BP half hourly
o NASH-reduction of obesity most of the time im-
prove liver function Insert a NG tube

Cross match blood at least 4 units blood


Mx of complications- Start IV N saline infusion
o Esophagealvaricies-develop rapidly in children, Give PPI , Omeprezole
Acute bleeding-Blood transfusion,H2 receptor block-
ers,octreotideinfusion,Vasopressinanalogues,sclero- Consider giving FFP& packed cells
therapy or band ligation
Monitor urine out put
Permanent-porto-caval shunts
Urgent upper GI endoscopic treat ment
o Ascites-sodium & fluid restriction,diuretics,albumin
infusion &paracentesis in refractory cases Target Questions

o SBP-broad spectrum antibiotics 23) How do you define CLDx?

o Encephalopathy-avoid pptfactors, oral lactulose

Page | 85
FMAS, RUSL
UNDER THE GUIDANCE OF DR. Anurudhdha Padeniya
Done by Udeshika Pinnagoda 07/08, Charith 08/09, Tishni 09/10

1) CLDx is a Dx process of the liver that involves a process of Even thoughimproving nutrition does not improve the survival,it
progressive destruction & regeneration of liver parenchyma will improve peri operative morbidity & mortality after liver
leading to fibrosis & cirrhosis. transplantation

24) What is cirrhosis? 5) How do you provide nutritional support to malnourished


children due to liver Dx?
1) Is a consequence of CLDx characterized by replacement of
liver tissue by fibrosis scar tissue & regenerative nodules High protein
leading to loss of liver function
High carbohydrate
25) Why does malnutrition develops in CLDx?
50% more calories
Anorexia-inflammatory cytokines
20-40% long chain triglycerides
Early satiety-gastric compression,HSP,Ascites
Vitamin K,A,D,E supplements
Reduced assimilation-Bile acid deficiency-lipid,Pancreat-
icinsufficiency,Intestinal factors-all nutrients,bacterial over- Many children will require NG or parenteral nutrition
growth
6)What are the indications for liver transplantation in chronic
Increased energy consumption-Inefficient cellular oxidation, liver failure?
Vasodilation,Abdominal distension causing increased work
Severe malnutrition unresponsive to nutritional support
of breathing
Recurrent complications
4) Why does improving nutrition is important in children
with liver Dx? FTT

Poor quality of life

Page | 86
FMAS, RUSL
UNDER THE GUIDANCE OF DR. Anurudhdha Padeniya
Done by Udeshika Pinnagoda 07/08, Charith 08/09, Tishni 09/10

absolute CI are sepsis, untreatable cardiopulmonary dx or Liver transplanting plan.


cerebrovascular dx
Try to maintain good school performance.
7) What are the post transplantation complications?
Inform school teachers regarding the condition.
Primary non function of liver

Hepatic artery thrombosis

Biliary leaks & strictures

Sepsis as the main cause of death

8)How you do counseling?

Counsel the mother regarding the disease and its current sta-
tus.

Make her knowledgeable regarding the alarming signs (con-


fusion….)

Ask parents not to take child to crowded places ( infection


control)

Advice regarding importance of good drug compliance.

Ask parents not to give much PCM and NSAIDS

Do family screening.

Page | 87
FMAS, RUSL
UNDER THE GUIDANCE OF DR. Anurudhdha Padeniya
Done by Hasara 07/08 2nd Edition by Lahiru, 3rd Edition by Eranda & Dilini 09/10

UTI
Presentations

Age Most Least


common common DD’s:
group
Infants Fever Poor feeding Abdominal Vary according to P/C.
younger Vomiting Failure to pain
than 3 Lethargy thrive Jaundice Eg. Irritant urethritis
months Irritability Haematuria
Offensive Urinary calculi
urine
Urethritis secondary to STD
Infants Fever Loin Lethargy
and (preverbal) tenderness Irritability History:
children, Vomiting Haematuria
3 Poor feeding Offensive P/C→
months urine
or older Failure to o Crying during micturition
thrive
Frequency Dysfunctional Fever o fever
Dysuria voiding Malaise
(verbal) Changes to Vomiting Hx/P/C→
continence Haematuria
Abdominal Offensive o Analyze the P/C
pain urine o Prior antibiotic treatments for these symptoms?
Loin Cloudy o Exclude DD according to P/C
tenderness urine o Previous similar episodes?

Page | 88
FMAS, RUSL
UNDER THE GUIDANCE OF DR. Anurudhdha Padeniya
Done by Hasara 07/08 2nd Edition by Lahiru, 3rd Edition by Eranda & Dilini 09/10

o Predisposing factors o of VUR, renal disease


- Infrequent voiding habits
-Wiping of perineum back to front Social Hx→
-Constipation
-Poor urine flow o Hygiene
- Reduced water intake
- Impaired host defense o Drinking enough water

P/M/Hx→ o Sexual abuse


o recurrent UTI (febrile/afebrile) o Inappropriate/ prolong used of diaper.
o Recurrent PUO
o Urinary tract abnormalities/ballooning of prepuce on micturition Examination:

General→
Birth Hx→ o Dehydration
o Growth parameters – poor weight gain, failure to thrive
o antenatal diagnosis of renal abnormality o Temperature
o Features of renal failure- highBP, edema
P/S/Hx→ o Spinal defects
e.g.: sings of occult myelodysplasia
o urethral instrumentation Midline pigmentation, lipoma, sinus
Abdomen→
Growth Hx→
o Palpable bladder
o poor growth o Ballotable kidneys
o Renal angle tenderness
Family hx→

Page | 89
FMAS, RUSL
UNDER THE GUIDANCE OF DR. Anurudhdha Padeniya
Done by Hasara 07/08 2nd Edition by Lahiru, 3rd Edition by Eranda & Dilini 09/10

o External genitalia- labial adhesions,phimosis&signs of in- Voiding dysfunction and poor urine flow
flammation
Evidence of hydronephrosis

Investigations: Palpable bladder

UFR Evidance of CRF


Urine culture & ABST
FBC- neutrophil leukocytosis Should be considered to focus the exact management plan
Dipstick test for nitrites & leucocyte esterase
Ill/ septic child- BU, SCr, SE, Blood culture, LP (if indi- The next step is to confirm the diagnosis via UFR, Urine Culture &
cated) ABST
Urgent USS – in complicated UTI, neonate, children with
distended bladder and ballatable kidneys Urine Culture/ ABST is the Gold Standard Ix for Diagnosing
UTI

Ward Management

Correct dehydration
Control pain & fever
Management: Domperidone if vomiting+
The clinical assessment should also indicate any serious underline
Infants< 3/12 age
pathology which is associated with the infection. Refer immediately to paed. Specialist
Treat with parenteral antibiotics
Here,
Infants & children >3/12 age
Recurrent UTI
Page | 90
FMAS, RUSL
UNDER THE GUIDANCE OF DR. Anurudhdha Padeniya
Done by Hasara 07/08 2nd Edition by Lahiru, 3rd Edition by Eranda & Dilini 09/10

Pyelonephritis Bacteriuria Antibiotic No UTI


Refer to paediatric Specialist negative treatment
Treat with oral antibiotics for 7-10 days. Eg: cephalosporin, should be
co amoxyclav started if
If oral can’t be given; IV cefotaxime or ceftrioxone for 2- clinically
4days followed by oral for total 10d. UTI

Cystitis
2) How do you differentiate pyelonephritis from cystitis?
Oral antibiotics for 3 dayseg: trimethoprim, nitofurantoiin, Pyelonephritis
cephalosporin, amoxicillin Bateriuria + fever >=38C or
Bacteriuria + fever < 38C + loin pain/ tendeness
CRP alone can’t differentiae pyelonephritis from
If parental treatment is required and IV treatment is not possible cystitis
consider intramuscular. Cystitis
If a child receiving prophylaxis develops UTI again; Rx with bacteriuria + but no systemic signs/symptoms
different antibiotic ( not increasing dose of same drug).
No antibiotics for asymptomatic bacteriuria
3) How do you prevent a recurrence?
Address dysfunctional elimination syndromes and constipa-
Target Questions tion
Encourage to drink an adequate amount.
1) How do you interpret UFR & culture results?
Should have ready access to clean toilets when required and
should not be expected to delay voiding.
Microscopy Pyuria Pyuria
results positive negative
4) What’re the recommendations for antibiotic prophylaxis?
Bacteriuria UTI + UTI+ Not routinely recommended following 1st UTI
positive May consider in recurrent UTI

Page | 91
FMAS, RUSL
UNDER THE GUIDANCE OF DR. Anurudhdha Padeniya
Done by Hasara 07/08 2nd Edition by Lahiru, 3rd Edition by Eranda & Dilini 09/10

No prophylaxis for asymptomatic bacteriuria

5) What’s “atypical UTI” & “recurrent UTI”?

Atypical UTI includes:


o seriously ill
o poor urine flow Recommended imaging schedule:
o abdominal or bladder mass Test Responds Atypical Recurrent
o raised creatinine well to UTI UTI
o septicemia treatment
o failure to respond to treatment with suitable antibiotics within within
48 hours 48 hours
o infection with non-E. coli organisms. Ultrasound No-for any Yes Yes
during age - for any - for only <6/12
Recurrent UTI: the acute age
two or more episodes of UTI with acute pyelonephritis/up- infection
per urinary tract infection, or Ultrasound Yes- for only No Yes
one episode of UTI with acute pyelonephritis/upper urinary within <6/12 - for any -for only >6/12
tract infection plus one or 6 weeks age
More episode of UTI with cystitis/lower urinary tract DMSA 4–6 No- for any Yes Yes
infection, or months age -for only - for any age
three or more episodes of UTI with cystitis/lower urinary tract following the <3 yrs
infection acute
infection
MCUG No- for any Yes Yes
age - for only - for only <6/12
<6/12

Page | 92
FMAS, RUSL
UNDER THE GUIDANCE OF DR. Anurudhdha Padeniya
Done by Hasara 07/08 2nd Edition by Lahiru, 3rd Edition by Eranda & Dilini 09/10

Indications & uses dysplastic kidneys,


scarring
Imaging Indication uses d. Other structural
USS recommended for anomalies: VUR,
all children duplex system,
following first ureterocele
attack of UTI
neonates, in
complicated MCUG a. Suspected bladder Used to diagnosed
UTI and persistently outlet obstruction posterior urethral
distended bladder or b. Dilated ureter / valvesand VUR
ballotable kidneys. dilated ureter and
dilated pelvicalyceal
DSMA Under 1 year of Look for renal system in
age scarrring the USS under 5
a. All children with years of age
febrile UTI c. Abnormal DMSA
scan:
Under 5 years of Significant
age scarring under 5
a. highly suggestive years of age
of acute Dysplastic
pyelonephritis kidneys
other than fever d. First attack of
b. Second attack of febrile UTI under 1
febrile UTI before 5 year of age and
years of age DMSA scan is
c. Abnormal USS: not available
hypoplastic /

Page | 93
FMAS, RUSL
UNDER THE GUIDANCE OF DR. Anurudhdha Padeniya
Done by Hasara 07/08 2nd Edition by Lahiru, 3rd Edition by Eranda & Dilini 09/10

e. Second attack of Importance of continuation of prophylaxis


febrile UTI under 5 Increase fluid intake to have good UOP
years of age Avoid predisposing factors
f. Clinical picture is Double micturition in pts with VUR
highly suggestive of General Measures
acute pyelonephritis - Good hydration
of - Avoiding constipation
children below 5 - Improving the hygiene of the child
years and DMSA - Wiping of the perineum from front to back
scan is not available. - Changing the child’s nappies frequently
DTPA a. Suspected pelvi- Used to diagnosed
ureteric or vesico- PUJ and VUJ 8) What is the recommended investigation routine to investi-
ureteric junction obstruction gating for Recurrent UTI
obstruction Can also give idea
about different renal
funtion UTI

USS

6) When to discharge the child? Abnormal Normal


When child is no longer having fever & urinary
symptoms, IV antibiotics are over, child is well looking & repeat Less than 1 yr – DMSA after 6 months
urine culture shows no growth Relevant
Investigations 1 to 5 years – DMSA if recurrent UTI
7) What advices would you give parents if child is getting re- or if 1st episode of pyelonephritis
current UTI?
Stress on importance of early recognition & prompt treatment of 9) How do you follow up?
further attacks No need of follow up for children not undergoing imaging
Page | 94
FMAS, RUSL
UNDER THE GUIDANCE OF DR. Anurudhdha Padeniya
Done by Hasara 07/08 2nd Edition by Lahiru, 3rd Edition by Eranda & Dilini 09/10

Asymptomatic bacteriuria is not an indication for follow-up. a) Those children with normal USS following an afebrile or a
When results are normal; no need of follow up simple febrile UTI will be followed up without prophylaxis
Recurrent UTI/ abnormal imaging results – assess by a paedi- or further investigations. (Refer figure 1)
atritian b) Those with structural abnormalities or recurrent UTI need
Children with renal parenchymal dx- assess height, wt, BP & prophylaxis till 5 years or longer.
routine testing for Proteinuria

9) Prophylaxis for UTI Antibiotic prophylaxis are as following


Drugs are given as a single dose in the night
Antibiotic prophylaxis is indicated for all children below Antibiotic Use
5 years following the first attack of UTI until an USS of Cephalexin Recommended in first 1 -3 mont5hs
the kidneys isavailable Cotrimoxazole Avoid in infants < 1 month of age
Nitrofurantoin Avoid in less than 3 months
First attack <1 year Nalidixic aci8d Avoid in less than 6 months
a. USS is normal in infants with afebrile UTI it is recommended
to stop the prophylaxis and to follow up without further investi-
gations.

b. In infants with febrile UTI, it is continued till recommended im-


aging studies are available or until their first birthday 10) What’re the methods of urine collection?

CCMS (Clean catch mid-stream sample)


SPA (Suprapubic aspiration)
c. Those with structural abnormalities or recurrent UTI need Catheter sample
prophylaxis till 5 years or longer.

11) How to advice parents on collecting CCMS urine sample?


First attack between 1-5 years Wash hands and genitalia with water - No antiseptics

Page | 95
FMAS, RUSL
UNDER THE GUIDANCE OF DR. Anurudhdha Padeniya
Done by Hasara 07/08 2nd Edition by Lahiru, 3rd Edition by Eranda & Dilini 09/10

(Retract the prepuce of the older boys)


Do not wash the urine culture bottle and do not leave
the lid opened for a long time
Send first few ml of urine out and collect a mid
stream specimen directly into the sterile culture bottle
without contamination
Close the cap and hand over immediately

12) How to transport a urine sample?

Send immediately to lab, if can’t send within 2h; refrigerate @ 4C


or in boric acid maximum for 24h

Page | 96
FMAS, RUSL
UNDER THE GUIDANCE OF DR. Anurudhdha Padeniya
Done by Harsha (07/08) 2nd Edition by Lahiru 08/09 3rd Edition by Aroosiya & Madushani 09/10

50% vesico ureteric reflux &distal ureter obstruction


POSTERIOR URETHRAL VALVE /
Mild hydroneprosis to sever renal dysplasia
OBSTUCTIVE UROPATHY 30% end stage renal disease or chronic renal insufficiency
Clinical presentation

Prenatal Maternal USS – B/L hydronephrosis, distended Investigation


bladder, if severe hydroneprosis
UFR
Neonate Palpable distended bladder, weak urinary stream
Urine culture & ABST
Infant FTT, uremia, sepsis
FBC
Mild cases Difficult in achieving diurnal urinary
in later incontinence. Recurrent UTI USS
life DMSA } to detect renal scaring

BU / S. Creatinine
It’s the most common cause of sever obstructive uropathy in
children. It is a V shaped mucosal fold in prostatic urethra.
Diagnosis
Due to obstruction prostatic urethra dilate
bladder muscle undergo bladder chronically VCUG (MCUG) or by perinatal USS
hypertrophy distended
Shape of cashew fruit

Has to be done after 6 wks except BOO(immediate)


97 | P a g e
FMAS, RUSL
UNDER THE GUIDANCE OF DR. Anurudhdha Padeniya
Done by Harsha (07/08) 2nd Edition by Lahiru 08/09 3rd Edition by Aroosiya & Madushani 09/10

Pre procedure Clinics follow up

Resent U. culture (5D before) must be negative With urine culture report

Prophylaxis AB should be converted to therapeutic dose 3D Assess height, weight and blood pressure
prior
Serum electrolytes
Exclude phimosis and labial adhesions
Paediatric Nephrology referral
Allergic Hx-important(IV hydrocotizone???)
Children with polyuria can dehydrate quickly- assess hydra-
tion

Septic &Uraemic infant – life saving measures Some can have renal tubular acidosis; they require oral bicar-
bonate.
Promote correction of electroletes imbalance, control of in-
fection by appropriate antibiotic,

Sometime drainage by precutaneouseneprostomy, and hae- What is DTPA? It is a radio isotope scan. Use Tc99
modialysis
Assess vascularity of the kidneyand functioning of the kidney.
After stabling --- cystourethroscopic ablation or vesicostomy

Prophylaxis antibiotics use for prevent infection before and


after surgery and it reducesthe amount of remissions. After
surgery alsohydroneprosis can preservefor many years so
prophylactic are essential.

Those children evaluate annually with USS

98 | P a g e
FMAS, RUSL
UNDER THE GUIDANCE OF DR. Anurudhdha Padeniya
Done by Rukman07/08, Muthumalika 08/09, Arun Selvendran 09/10

Yellowish discoloration of eyes, itching of skin, pale stools, dark


Nephrotic syndrome urine (LIVER?)

Presenting complaint- Arthralgia, arthritis, oral ulcers, alopecia, photo sensitive or any
other skin rash.
Edema- initially periobital and ankle.
Frontal headache, visual disturbance (HT)
Late ascites and pleural effusions

protein urea-initially frothy urine. After diagnosis by


heat precipitation or dipstick. What did up to today: GP treatment

History of presenting complain- Hospital admissions

Generalized body swelling? Duration Investigations

Onset: how noticed 1st Complications Of the disease-

Sites of edema: eye, abdomen, scrotum, ankle • Flank pain with gross haematurea-renal vein throm-
bosis(RVT)
Progression: site to site
• Calf pain+/- difficulty in breathing-DVT and PE
Diurnal variation (HF or RF)
• Collapse and syncope- hypovolemia
Preceding or presenting fever (ig a nephropathy, HUS, HEP B, SLE,)
• Fever with abdominal pain- subacute bacterial peritoni-
Urine: oliguria, hematuria, frothy urine, dysuria, frequency
tis(SBP)
Vomiting, diarrhoea (HSP)
• SOB, orthopnoea-PE
orthopnoea, PND, malaise, lethargy (HF)
• Headache, visual disturbances, seizures-HT, stroke

99 | P a g e
FMAS, RUSL
UNDER THE GUIDANCE OF DR. Anurudhdha Padeniya
Done by Rukman07/08, Muthumalika 08/09, Arun Selvendran 09/10

• severe vomiting-intestinal oedema Polyurea/polydypsia/FBS blood transfusion

gastric irritation due to oedema Malaria chronic leg ulcers

• abdominal pain-hypovolemia Birth Hx: neonatal jaundice and ……………..others

SBP Immunization: live vaccines vaccines, pnemococcal, meningococcal

Cellulitis F/H : 3 gen. pedigree Renal disease, female hematurea,


consanguinity of marriage
RVT
SOCIAL: Education. Knowledge regarding Dx
Complications of long term prednisolone use.
Family support/, income , Hospital stay, clinic follow up, Dx
Recurrent infections.
how affect to child and family distance to hospital/, vehicle in
Short stature. home

Cataract. Daily activities/, school absent / school performances

Impaired glucose tolerance. Crowded places,

EXAMINATION
Cushinoid features.
Anthropometry with centiles weight, height, body surface area,
PMHx:
BMI
Known pt of nephritic Sx: diagnosed at (age)
General: appearance
Clinic follows up drugs knowledge on urine test relapses
Fever, icterus, pallor cataract rash, alopecia, joint swelling edema
Episodic hematurea and loin pain oral ulcers leg ulcers

100 | P a g e
FMAS, RUSL
UNDER THE GUIDANCE OF DR. Anurudhdha Padeniya
Done by Rukman07/08, Muthumalika 08/09, Arun Selvendran 09/10

CVS: pulse (low?) BP, HF (CHD) ESR, ASOT , C3 , C4, ANA Ab (if macroscopic or persistent
microscopic hematuria present after excluding UTI)
RES: pleural effusion, RTI, B/L fine crepts in HF
Chest X-ray (if suggestive HF)
ABDOMEN: ab wall edema, swelling umbilicus CLD features
Renal biopsy (indications in nephritic)
Peritonitis, tenderness
MANAGEMENT
Bowel sounds
Supportive
INVESTIGATION
Daily renal chart (urine albumin, IP/OP, abdominal girth and BP tds)
Initial
Strict bed rest not indicated,
Early morning urine ward test
Diet- Adequate calorie & protein, sodium restriction until gross
UFR – Albumin, red cells & casts, pus cells, org., fat casts
edema settled,
Urine culture + ABST ( if pus cells positive)
Early morning urine ward test
FBC – Hb (increased with haemoconcentration, decreased in
Scrotal edema elevate with a pillow
malnutrition)
DRUGS
RFT- SE , BU , S.Cr,
1st episode oral prednisolone 60/m2/day (maximum 60mg per day)
LFT- S. Protein with fractions, (S. ALBUMIN), PT/INR for 4weeks
24 hour urine/ urine creatinine: protein ratio) Then 40mg / m2 /day (maximum 40mg per day) EOD for 4 week
Further Ix and tapering of the dose over 2-5 months

O.penicilin 250 micrograms/day for 10 days


S. CHOLESTEROL (lipid profile)- Increased but transient

101 | P a g e
FMAS, RUSL
UNDER THE GUIDANCE OF DR. Anurudhdha Padeniya
Done by Rukman07/08, Muthumalika 08/09, Arun Selvendran 09/10

Diuretics- caution, do not use without correcting hypovolemia 4. Why infections common in nephrotic

QUESTIONS Ig lost with protein

1 management in a relapse Steroids reduce immunity

60mg/m2/day(maximum 60mg per day) until pr– in urine nil for Edema fluid good culture medium to infections
continuous 3 days ,,,, then 40mg/m2/day(maximum 40mg per day)
Low blood perfusion to spleen due to hypovolemia
FOR 4 weeks in EOD.
5. Complications of nephrotic
2. Frequent relapse management?????
Peritonitis
Add levimasole monthly or cyclophosphomide or cyclosphorine
(please look I0 THESE DRUGS AND SIDE EFFECTS OF THEM) DVT (AT 3, protein S and C passed with urine. Clotting factors are
large molecules retained and hypercoagulability resulted)
3. Advises to mother on discharge (please read)
Hypovolemic and pre renal failure/shock
Nephrotic syndrome?
Relapses
Compliance of drugs
CKD risk 5% even in MCD (this not tell in nelson)
Urine early morning test
Hyper cholesterolaemia
Salt restriction while edema
6. features suggesting steroid sensitivity
3+++ for 3 days/severe edema/fever admit to hospital
Age 1-10y
Nephritic card to show when take Rx from a GP and to show while
vaccination No hematuria. NL BP, NL complements, N LCreatinine
School avoidance in 60mg/m2/day period

102 | P a g e
FMAS, RUSL
UNDER THE GUIDANCE OF DR. Anurudhdha Padeniya
Done by Rukman07/08, Muthumalika 08/09, Arun Selvendran 09/10

7. Types of nephrotic syndrome (S.Cr, FBC, Bleeding time, Clotting profile, Renal USS-
perform a day prior to the Sx &mark the entry point.advice
1ry glomerular disease 2 nd glomerular mother to do not wash off)
disease
Cross match blood
MCD SLE
keep fasting for 6 hours
FSGS AMYLOIDOSIS
10. How to manage the post Operation period?
MEMBRANOUS NEPROPATHY DRUGS
(penicillamine, gold) Monitor vital parameters,UOP
MPGN DM Collect all urine samples
INFECTIONS (hep
Complete bed rest until haematuria settles
B….)

8. Recommendations for renal biopsy.

1. Onset < 6 months of age ADVICE GIVEN TO THE MOTHER


2. Initial macroscopic hematuria (without infection)
3. Persistent microscopic haematuria with hypertension
4. Renal failure not attributable to hypovolaemia I advised to the mother of baby who is having
5. Persistently low plasma C3, C4 levels nephritic syndrome. First I explained the mother
6. Steroid resistance about the disease condition. I told that this is the
benign one in most of the cases. Relapses occur in
9. How to prepare a patient for a renal biopsy?
1/3 of affected people very little proportion
Discuss with the team and arrange a date develops renal failure. Otherwise there is no very
bad issue so I asked her to not to about disease.
Initial workup of the patient by following investigations

103 | P a g e
FMAS, RUSL
UNDER THE GUIDANCE OF DR. Anurudhdha Padeniya
Done by Rukman07/08, Muthumalika 08/09, Arun Selvendran 09/10

• First I advised her how to do the urine I emphasized the mother following aspects of
protein test. management.

• Should do early this in early morning Urine examination for protein at home. Examination
should be done every morning during a relapse,
• 2/3 of test tube filled with urine, hold it
during intercurrent infection or if the child has even
slanted & heat the upper 1/3 of urine level until the
mild, periorbital oedema
boiling point.
Urine is examined twice/thrice a week during the
• When there is turbidity add 2-3 drops of
remission
vinegar & observe whether turbidity disappears.
The dipstick test is carried out by dipping the marked
• If it persists assess the turbidity by using
end of the strip in urine in urine for 3 seconds &
printed paper behind the test tube.
comparing the colour change at the code given in
• Nil-no turbidity the park.

• Trace –slight turbidity with no reading I asked the mother to maintain a diary showing
difficulty. protein area, medications received & intercurrent
infections
• + turbidity & difficult to read
I asked her to ensure normal activity & school
• ++ cannot read but can notice the black attedence.it is important that the child participates
in all activities & sports
• +++ cannot notice black
I explained about diet including normal home diet-
• ++++ cannot notice black & precipitation
reduced fat containing diet-carbohydrates are best
present
given in complex forms. a modest reduction in salt is
advised in the presence of oedema. Snacks

104 | P a g e
FMAS, RUSL
UNDER THE GUIDANCE OF DR. Anurudhdha Padeniya
Done by Rukman07/08, Muthumalika 08/09, Arun Selvendran 09/10

containing high salt are best avoided during this


period. Fruits & fruit juices can be given without
restrictions. No need to restrict the proteins.

I asked to make sure that all vaccines including in EPI


program me should be offered to this baby while
receiving drug. But should be aware that some
vaccines are not given while on treatment with
steroids.

If child is having contact history of measles come to


the hospital immediately.

I explained to the mother about side effect of drugs


such as weight gain, increased appetite, diabetes
mellitus, hypertension, recurrent infection,
abdominal irritation therefore need frequent meal.
But at the same time, highlighted the mothers’
importance of having continuous medication
continuously until doctor stops.

At the end of discussion I told that if not responding


to prednisolone, other drugs will be started.

105 | P a g e
FMAS, RUSL
UNDER THE GUIDANCE OF DR. Anurudhdha Padeniya
Done by Rohan 07/08, 2nd Edition by Sanjeeva 08/09, 3rd Edition by Arunprasath 09/10

Hypercalciuria
Nephritic syndrome Glomerular causes.
How do they present: AGN (usually with proteinuria)
Hematuria Chronic Glomerular nephritis (usually with proteinuria)

Edema characteristically around the eyes IgA nephropathy


Familial nephritis – Alport syndrome.
Oliguria
The causes for acute nephritis in childhood
HTN Hypertensive encephalopathy{Headache, altered con-
scious level, seizures, coma} 1. Post infectious (including streptococcus)

DD’s 2. Vasculitis (HSP, rarely SLE, PAN, Wegener granulomatosis,


microscopic polyarteritis)
Haematuria
3. IgA Nephropathy & mesangiocapillary glomerulonephritis
Non glomerular causes.
4. Goodpasture syndrome – very rare.
Infection(bacteria,viral,TB,Schistosomiasis)
Trauma to genitalia, urinary tract or kidneys
Oedema
Stones
Nephrotic syndrome
Tumours
Nephritic syndrome
Sickle cell disease
HF
Bleeding disorders
CLD
Renal vein thrombosis
ARF / CRF

Page | 106
FMAS, RUSL
UNDER THE GUIDANCE OF DR. Anurudhdha Padeniya
Done by Rohan 07/08, 2nd Edition by Sanjeeva 08/09, 3rd Edition by Arunprasath 09/10

Allergic conditions Color of urine: Coca cola color- true hematu-


ria originating from kidney, Pink/red from
bladder, urethra with or without clots.
History:
Duration: IgA nephropathy episodic short
Introduction→ period ass. With URTI and resolving after the
infection, PSGN up to 1 year
o Age
Recurrent or not
AGN -5-15 yrs
Painless/Painful: AGN Painless
Nephrotic- 3-5 yrs
Throughout(KIDNEY), Mid or end
o Sex: more common in boys stream(LUT)

Hx/P/C→ To exclude DDs…

o Proceeding history: Fever with sore throat/skin infec- UTI - frequency, dysuria, loin pain,
tions / itching (2ry infected scabies) suprapubic pain, strain during micturi-
tion, past UTI, Anomalies
o If above present what has mother done? Took Rx?
Drugs and Ix done? (BP, UFR,FBC) Calculi - Past Hx colicky pain(loin to
groin)
o Analyze haematuria
PAN – petechial rash
True haematuria or not?

Page | 107
FMAS, RUSL
UNDER THE GUIDANCE OF DR. Anurudhdha Padeniya
Done by Rohan 07/08, 2nd Edition by Sanjeeva 08/09, 3rd Edition by Arunprasath 09/10

HSP - abdominal colicks, joint pain Site


and swelling, Purpuric rash in but-
tocks and shin, constipation, malena, Progression: Site to site and diurnal variation
hematemesis
Associated fever
SLE - mild on and off fever, weight
DDs
loss, rashes, hair loss, oral ulcers, ar-
thralgia Frothy urine Nephrotic Syndrome
Bleeding disorders - easy bruising, Dyspnoea, orthopnoea, PND LVF ,
gum bleeding, malena Peripheral oedema, ascites RHF
HUS - acute GE, URTI, blue skin Recent allergy
patches, DPT vaccine
Oliguria/anuria, snake bite, poison
G6PD deficiency-recent ingestion of ARF
foods like Kuppamenia, drugs like an-
timalarials, antibiotics, and anti-in- CLD features
flammatory drugs.

Hx of beetroot consuming, anti TB


drugs o What has happened up to now

Hx of trauma GP/Pediatrician opinion

o Analyze oedema Ix done and results

Duration BP/ Urinary Protein- if checked

Page | 108
FMAS, RUSL
UNDER THE GUIDANCE OF DR. Anurudhdha Padeniya
Done by Rohan 07/08, 2nd Edition by Sanjeeva 08/09, 3rd Edition by Arunprasath 09/10

o Complication assessment o Congenital abnormalities of the urinary tract.

ARF: UOP, Uraemic symptoms-itching, hic- P/S/Hx


cups, dry skin, anorexia, nausea, vomiting,
and muscle cramps. o Related to UTI

HTN/ Hypertensive encephalopathy: Blurred Drug Hx


vision, frontal headache, drowsiness, new on-
o Especially for JIA (Gold, MTX, Penicillamine)
set seizures
Birth Hx
HF
Developmental Hx
Ass. Nephrotic syndrome
Growth Hx
P/M/Hx→
Imm Hx→
o Similar illnesses
o DPT associated with HUS
o Recurrent tonsillitis
Diet Hx→
o Recurrent infections IDcy
o Salt intake associate with HTN
o Previous bleeding manifestations
o Food which cause red color urine
o Prophylactic antibiotic Rx
Family Hx
o Polycystic kidney disease.
o Renal transplantation
o Previous instrumentations.

Page | 109
FMAS, RUSL
UNDER THE GUIDANCE OF DR. Anurudhdha Padeniya
Done by Rohan 07/08, 2nd Edition by Sanjeeva 08/09, 3rd Edition by Arunprasath 09/10

o SLE o Wt/ Ht/ BMI (have to plot in a centile chart and com-
ment)
o HTN
o Well/ill
o Polycystic kidney disease.
o Dyspnoeic
o Haematuria
o Fever
o Bleeding Disorders
o Pale
Social Hx→
o Icterus Hemolysis, CLD
o Economic state
o Fundal examination - papilledema
o Hygiene
o Cervical lymphadenopathy
o Education level of parents
o Edema(around eyes, abdomen, sacrum)
o Nearest hospital
o Hydration
o Home conditions, overcrowding.
o Skin rashes, scabies infection, hair loss (SLE,HSP)

o Joint swelling

o Throat (enlarged tonsils) IgA nephropathy, PSGN


Examination:
CVS→
General→
o Pulse

Page | 110
FMAS, RUSL
UNDER THE GUIDANCE OF DR. Anurudhdha Padeniya
Done by Rohan 07/08, 2nd Edition by Sanjeeva 08/09, 3rd Edition by Arunprasath 09/10

o BP o Renal masses.(poly cystic kidney disease, Willm’s


tumors)
o JVP?

o Features of HF: tachycardia, gallop rhythm, tachyp-


nea, displaced Apex, tender Hepatomegaly
CNS→
RS→
o Features of encephalopathy: papilledema, power, re-
o RR
flexes, tone, GCS in older children.
o LRTI features

o Bibasal creps – HF

o Pleural effusion - chest expansion,stoy dull to per-


Investigations:
cuss, air entry, absent breath sound (consider the
age of the child to do the relevant examination) UFR→

o Pus cells UTI do U. culture and ABST

Abdomen→ o RBC: Casts and Dysmorphic RBC seen in AGN,


also seen in urinary calculi.
o Tenderness
o Albuminuria
o Hepatosplenomegaly
FBC: Anaemia
o Ballatable kidneys (poly cystic kidney disease)
Renal functions: S.cr, BU, SE
o Free fluid
Special Ix depending on the circumstance
Page | 111
FMAS, RUSL
UNDER THE GUIDANCE OF DR. Anurudhdha Padeniya
Done by Rohan 07/08, 2nd Edition by Sanjeeva 08/09, 3rd Edition by Arunprasath 09/10

o Sorethroat+ : ASOT, swabs Avoid high K+ diet (king coconut water, fruit juice)

o Skin sepsis+ : swabs Prophylactic antibiotics

o Complement levels: o O.Penicillin 250mg/bd 10 days or Benzathine penicil-


lin after sensitivity test
C3 ↓ PSGN
SRNifedipine 20mg bd
C3 C4↓ SLE
Captopril: start with low dose(6.25mg)
o CRP/ESR : ↑in SLE, infection, Inflammation
Frusemide: if fluid overload/pulmonary oedema/HF
o If persistent haematuria: ANA, USS abdomen, Renal
Biopsy

Management: Target Questions

Daily weight chart 26) Difference between Nephrotic syndrome and nephritic syn-
drome?
Abdominal girth
Nephrotic Nephritic
BP tds Very severe oedema Mild oedema
No macroscopic haematuria Microscopic and
IP/OP chart
macroscopic
Urine protein ward test haematuria
pre school age Commonly 5 – 15 yrs
Monitor BP, PR, RR Spo2 frequently Change in capillary Inflammation of
permeability Pr- leakage glomerulus due to
Fluid restriction: 2/3rd of maintenance from glomeruli ↓oncotic deposition of Ag-Ab
pressure in vascular complexes and C3
Salt restriction compartment fluid leak : Haematuria

Page | 112
FMAS, RUSL
UNDER THE GUIDANCE OF DR. Anurudhdha Padeniya
Done by Rohan 07/08, 2nd Edition by Sanjeeva 08/09, 3rd Edition by Arunprasath 09/10

oedema Hypovolaemia ↑ : GFR 29) What are the vasculitic causes of AGN?
risk of thrombosis ↓ hypervoluaemia
BP ↑ 1) HSP

2) PAN
27) Discharge criteria
3) Wegener’s granulomatosis
1) When BP, UOP normal
4) Microscopic polyarteritis
2) Oedema settled
30) What is the familial cause of AGN

1) AlportSx: x- linked reccesive


28) Discharge plan
Prognosis to ESRF
1) Maintain UOP and oedema diary
Cause nerve deafness
2) Measure BP every other day
Mother may have hx of haematuria.
3) R/V after 1 week
31) What are the main complications of Nephritic Syndrome?
4) Trace UFR
Hypertensive encephalopathy
5) After 1 week if BP normal no further BP measurement
required Acute Renal Failure

6) R/V @ 3, 6, 9 month with UFR Cardiac Failure

32) How are you manage Acute Hypertension ?

Page | 113
FMAS, RUSL
UNDER THE GUIDANCE OF DR. Anurudhdha Padeniya
Done by Rohan 07/08, 2nd Edition by Sanjeeva 08/09, 3rd Edition by Arunprasath 09/10

Diagnose Hypertension
Drop the blood pressure
Assess the severity and manage accordingly slowly as..

1/3over 6hrs

Next 1/3 over 24 hrs

Final 1/3 over 48 hrs


Type Definition Management

Hypertensive Elevation Oral antihypertensives


33) How to manage Acute Renal Failure?
emergency of blood
pressure (oral Nifedipine )
Fluid and electrolyte balance
without
symptoms
IV Fluids -(UOP of previous day +insensible
or
evidences loss )
of target
organ Mx of Hyperkalaemia
damage
Salbutamol nebulization
Hypertensive Elevation Mx at ICU
encephalopathy of blood IV calcium gluconate for stabilization
pressure Assess ABC of the myocardium
with target
organ IV antihypertensives Insulin-dextrose infusion
damage
(IV
Potassium binding resins
hydralazine,nitropresside)

Page | 114
FMAS, RUSL
UNDER THE GUIDANCE OF DR. Anurudhdha Padeniya
Done by Rohan 07/08, 2nd Edition by Sanjeeva 08/09, 3rd Edition by Arunprasath 09/10

Consider renal replacement therapy if re-


quired

34) What are the causes of glomerulonephritis which have re-


duced complement level?

Primary renal disease

Post streptococcal GN {C3 reduced, C4 normal}

Mesangiocapillary GN {C3 & C4 reduced}

Systemic disease

SLE { C3 & C4 reduced}

Page | 115
FMAS, RUSL
UNDER THE GUIDANCE OF DR. Anurudhdha Padeniya
Done By Wathsala Nirmani 07/08,2nd By Lochana ,3rd By Narmada 09/10

Meningitis
o If >2months
Introduction :
o Fever
Age
o Irritability
Duration is important as it may insidious onset or acute onset.
o Lethargy
History:
o ALC
P/C→ o Anorexia
o If <2months-
o Nausea
o fever o Vomiting
o hypothermia
o Photophobia
o irritability o Fits
o drowsiness
Differential diagnosis
o lethargy Brain abscess
Subdural/epidural abscess
o poor Feeding
Brain tumors
o vomiting Central nervous system (CNS) leukaemia
Lead encephalopathy
o fits CNS tuberculosis

116 | P a g e
FMAS, RUSL
UNDER THE GUIDANCE OF DR. Anurudhdha Padeniya
Done By Wathsala Nirmani 07/08,2nd By Lochana ,3rd By Narmada 09/10

Hypersensitivity to drugs (eg, trimethoprim-sulfa- History of intracerebral abscess (re-


methoxazole, intravenous immune globulin, and an- cent or in the past)
tithymocyte globulin)
Disorders associated with vasculitis (eg, Kawasaki Brain tumors
disease and collagen vascular disease) LOA,LOW
Central nervous system (CNS) leukemia
Hx/P/C→ Focal neurological signs
Lead encephalopathy
Symptom analysis Exposure to the lead
CNS tuberculosis
o fever-onset, associated rashes, chills and rigors, response to PCM, Contact history of tb, night sweating,
evening pyrexia
in between fever activeness of baby, Hypersensitivity to drugs (eg, trimethoprim-
sulfamethoxazole, intravenous immune glob-
o What mother did up to now?
ulin, and antithymocyte globulin)
o Any antibiotic treatment taken or not? Disorders associated with vasculitis (eg, Ka-
wasaki disease and collagen vascular disease)
Excluding the differential diagnosis
Discovering the etiology
Brain abscess
Mental status changes o Depending on age ask possible other symptoms included in P/C.
Focal neurologic deficits
o Look for infective foci-ear discharge, sore throat, cough & cold,
Subdural/epidural abscess diarrhea & abdominal pain, crying during Micturition , frequency ,
Headache - Initially focal and later dysuria , skin sepsis ,bone pain
generalized
Blurred vision (amblyopia) o Preceding history of URTI
Speech difficulty (dysphasia)
o Any recent head trauma, contact history of meningitis

117 | P a g e
FMAS, RUSL
UNDER THE GUIDANCE OF DR. Anurudhdha Padeniya
Done By Wathsala Nirmani 07/08,2nd By Lochana ,3rd By Narmada 09/10

Discovering the complications o prosthetic valve

o fits,

o focal neurological signs Birth Hx→


o hearing loss o Term or not?

o Birth weight,

P/M/Hx→ o antenatal maternal pyrexia


o Meningitis o congenital infection
o recurrent chest infections o Smelly vaginal discharge
o congenital heart disease o meconium aspiration
o septicemia o condition at birth
o SABE o PBU admissions

P/S/H→ Imm Hx→


o any neuro sx o BCG vaccination and scar.
o meningomyelocoele

o VP shunt

118 | P a g e
FMAS, RUSL
UNDER THE GUIDANCE OF DR. Anurudhdha Padeniya
Done By Wathsala Nirmani 07/08,2nd By Lochana ,3rd By Narmada 09/10

Diet Hx→ o alert

Growth & Dev→ o drowsy

o irritability

o Fontanels- size, bulging or not? , suture separation


Social Hx→
o Anthropometry including OFC

o Pallor
o schooling environment
o Icterus
o overcrowding environment
o sunset eyes
o siblings condition?
o cyanosis
o contact history of meningitis?
o hydration

o stigmata of infective endocarditis


Examination:
o BCG scar
General→
o Rash (MP, pethichae, purpura)
o ill looking
o spina bifida
o febrile or not
o Meningomyelocoele
o active
o ear and throat examination

119 | P a g e
FMAS, RUSL
UNDER THE GUIDANCE OF DR. Anurudhdha Padeniya
Done By Wathsala Nirmani 07/08,2nd By Lochana ,3rd By Narmada 09/10

o bone tenderness o scars of spleenectomy , palpable liver ,spleenomegaly

CNS→ Management:
o complete examination to look for focal neurological Initial management,
signs and signs of increased ICP or herniation
A – open airway and O2 supplementation
o Neck stiffness, kernig’s sign
B- respiratory support with bag and mask
o brudzinki sign intubation
o papilloedema

C – if in shock rapid infusion of normal saline 20ml/kg with iv access

CVS→

o CRFT, PR, BP, murmur Do not restrict fluids unless there is evidence of:

raised intracranial pressure, or


RS→
increased antidiuretic hormone secretion[2]
o signs of lung abscess ,pneumonia

Abd→

120 | P a g e
FMAS, RUSL
UNDER THE GUIDANCE OF DR. Anurudhdha Padeniya
Done By Wathsala Nirmani 07/08,2nd By Lochana ,3rd By Narmada 09/10

Intravenous fluid resuscitation in meningococcal septicaemia ventilation

In children and young people with suspected or confirmed meningococcal Bacterial meningitis and meningococcal septicaemia
septicaemia:
D - if ALC present with AF bulging and hypertention with bradycardia mannitol
if there are signs of shock give an immediate fluid bolus of 20toml/kg
be given
sodium chloride
E – look for meningococcal rash and control temperature
0.9% over 5–10 minutes. Give the fluid intravenously or via an
intraosseous route and F – fit should manage accordingly(phenobarbitone,phenytoin)

reassess the child or young person immediately afterwards G – blood glucose,BU , SE if any abnormality correct them initially

I – investigations – FBC(low platlets with DIC) , CRP , blood culture


if the signs of shock persist, immediately give a second bolus of 20 ml/kg
of LFT & RFT (deteriorate with sepsis)
intravenous or intraosseous sodium chloride 0.9% or human albumin Gold standard for diagnosis is isolation of causative organism from
4.5% solution CSF
over 5–10 minutes Lumber puncture – only do after stabilizing the patient
if the signs of shock still persist after the first 40 ml/kg: Send CSF for full report , glucose , gram stain , culture
immediately give a third bolus of 20 ml/kg of intravenous or intraosseous CT brain – only if indicated
sodium chloride 0.9% or human albumin 4.5% solution over 5–10
minutes

call for anaesthetic assistance for urgent tracheal intubation and


mechanical

121 | P a g e
FMAS, RUSL
UNDER THE GUIDANCE OF DR. Anurudhdha Padeniya
Done By Wathsala Nirmani 07/08,2nd By Lochana ,3rd By Narmada 09/10

patient centered care

good communication

Empirical antibiotic treatment

Age Drugs
Neonates Ampicillin Cefotaxime
(50mg/kg 4 hrly)(50 mg/kg 6 hrly)
1 – 2 months Benzyl penicillinCefotaxime or
(60 mg/kg 4 hrly)ceftrioxone Corticosteroid therapy
2 months – 5 yrs Cefotaxime or Ceftrioxone
Dexamethasone 0.15 mg/kg iv dose 6 hourly for 4 days.
> 5 yrs Cefotaxime or Ceftrioxone
(100mg/kg daily) Indications :- >3 months
Meningomyelocoele, vancomycin Cefotaxime or
vp shunts post ceftazidime Not pre treated with parenteral antibiotics
neurosurgery
frankly purulent CSF

CSF white blood cell count greater than 1000/microlitre

raised CSF white blood cell count with protein concentration

greater than 1 g/litre

bacteria on Gram stain

122 | P a g e
FMAS, RUSL
UNDER THE GUIDANCE OF DR. Anurudhdha Padeniya
Done By Wathsala Nirmani 07/08,2nd By Lochana ,3rd By Narmada 09/10

Follow up

Chemoprophylaxis:- o Children and young people should be reviewed by a


paediatrician with the results of their
Hib -for all household contact irrespective of age- rifampicin daily x 4days
o hearing test 4–6 weeks after discharge from hospital
If patient is <2yr start full course of hib vaccination
to discuss morbidities associated with
<5yr contacts who are unvaccinated should immunized.
o their condition and offered referral to the appropriate
services. The following morbidities

Meningococcal -all household and daycare contact irrespective of age – o should be specifically considered:
rifampicin 12 hrly x 2 days
o hearing loss (with the child or young person having
Patient with meningococcal or Hib should give rifampicin prior to undergone an urgent assessment
discharge from hospital to eliminate organism
o for cochlear implants as soon as they are fit)

o orthopaedic complications (damage to bones and


joints)

o skin complications (including scarring from necrosis)


Isol Isolation
o psychosocial problems
(Meningococcal , Hib)
o neurological and developmental problems

o renal failure.

123 | P a g e
FMAS, RUSL
UNDER THE GUIDANCE OF DR. Anurudhdha Padeniya
Done By Wathsala Nirmani 07/08,2nd By Lochana ,3rd By Narmada 09/10

o Bacterial meningitis and meningococcal sep 6. When in a shock

Target Questions: 7. Superficial local infection

1. Risk factors for development of meningitis? 8. Coagulation disease

Primary infection – OM , URTI , pyoderma 3).Complications of meningitis?

Hoto
Head trauma Haematogenous – pneumonia , infective endocarditis , osteomyelitis
, empyma
differentiate traumatic tap from positive findings?
Immune deficiency , spleenectomy
WBC into CSF per L =WBC peripheral x RBC csf x 106 /L
Congenital anomalies such as spina bifida meningomyelocele & any neuro
RBC peripheral
surgical procedures
If this value is less than actual WCC it is positive
In a new born- prematurity , LBW , PROM , maternal sepsis , congenital
infection 1000 x 106/L RBC in CSF------------------- increase protein level by
0.015 g/l
2).What are the indications to delay lumber puncture?

1. Signs of increased ICP


Csf gram stain is single test for rapidly diagnose bacterial meningitis and
2. Recent or prolong fit
initiating appropriate therapy.
3. Focal or tonic seizure
With antibiotics this may be negative ,in such situations can be detected by other
4. GCS < 13 indices

5. Strong evidence of meningococcal infection

124 | P a g e
FMAS, RUSL
UNDER THE GUIDANCE OF DR. Anurudhdha Padeniya
Done By Wathsala Nirmani 07/08,2nd By Lochana ,3rd By Narmada 09/10

Shock Mental retardation


Seizure Hydrocephalus
Cerebral abscess Ventriculitis
Subdural empymal DIC
effusion
Cerebral herniation Arachnoditis
Due to increased ICP
Focal neurological signsWater house-
friedrichson syndrome
Auditory impairment Myocarditis

125 | P a g e
FMAS, RUSL
UNDER THE GUIDANCE OF DR. Anurudhdha Padeniya
Done by amila priyadarshani 07/08,asanka 08/09, 3rd by Anusha 09/10

History:

ENCEPHALITIS P/C→

Presentations Acute fever with seizure o Fever

Fever +seizure + altered consciousness o Seizure

Acute fever with nonspecific symptoms o Altered level of consciousness

(Malaise, anorexia, vomiting, abdominal pain, myalgia, o Fever , headache, clouding of consciousness together
headache) with seizures and focal neurology in some cases
Specific viral syndrome include(pharyngitis, rash, diar- H/P/C→
rhea, cough, or other respiratory symptoms)
o Fever : duration, onset, character, responds to antipy-
retics, associated factors-malaise, anorexia, vomiting,
o DD’s: headache

Encephalitis o Seizure: type, duration,

Epilepsy in status epilepticus o Alteration of sensation, drowsiness, headache ma-


laise,
Cerebral malaria
Cognitive and behavioral changes
Acute disseminated encephalomyelitis (ADEM)
Alterations in the sensorium, drowsiness,

126 | P a g e
FMAS, RUSL
UNDER THE GUIDANCE OF DR. Anurudhdha Padeniya
Done by amila priyadarshani 07/08,asanka 08/09, 3rd by Anusha 09/10

Confusion and restlessness which progress rapidly could be seen o Recent vaccinations-measles, pertussis
Maculopapular rash and severe complications such as fulminant
Coma, transverse myelitis, anterior horn cell disease, peripheral Diet Hx→
Neuropathy

Cerebral malaria: characteristic fever pattern, in an


endemic area Social Hx→

Examination:

ADEM: 2-3 weeks following a viral respiratory tract General→


infection, visual loss may occur
Ill looking / aggressive / drowsy
Vaccination within preceding 2 weeks or and
infection (measles, mumps. rubella chicken fox) Febrile

(Should exclude other causes for acute fever)

P/M/H CVS→

Mumps, measles, epilepsy o pulse rate, blood pressure

Birth Hx→ RS→

ImmHx→ o Respiratory rate

o Any sign of infection


o JE given or not

127 | P a g e
FMAS, RUSL
UNDER THE GUIDANCE OF DR. Anurudhdha Padeniya
Done by amila priyadarshani 07/08,asanka 08/09, 3rd by Anusha 09/10

CNS→ UFR

o GCS Urine culture

o Neck stiffness, kerning sign Specific investigations

o Focal neurological deficit Lumbar puncture & CSF –after exclude contraindications. Spe-
cially increased ICP
o Increased reflexes
Lymphocytic pleocytosis, initially can be neutrophil
Investigations: predominate
Basic investigations Elevated protein
FBC → leukocytosis common in ADEM and relative lympho- Normal
cytosis in viral hepatitis
The CSF in viral encephalitis typically shows a
Serology- Ab to arbovirusus Lymphocyticpleocytosis, elevated protein content and
normal or mildly depressed glucose content.
Chest X ray Approximately 5%-10% of patients with biopsy-proven HSV
encephalitis have normal cell counts on the initial CSF examination
Serum Electrolytes

B. urea EEG-should obtain in all cases if possible

S .creatinine Most of them have abnormal EEG

Random blood sugar CT brain-take 3-4 days to appear changes

Blood culture MRI brain-imagine of choice

128 | P a g e
FMAS, RUSL
UNDER THE GUIDANCE OF DR. Anurudhdha Padeniya
Done by amila priyadarshani 07/08,asanka 08/09, 3rd by Anusha 09/10

Monitoring –

GCS

Management: Seizure type

Encephalitis is a pediatric emergency Clinical signs of increased ICP

While in the ward/ICU, monitoring of suspected encephalitis BP, O2 saturation 1hry


patient should include
Glasgow Coma scale score .(GCS) UOP 4hrly (0.5 ml/Kg/h)
Seizure type and frequency.
Clinical signs of intracranial hypertension. Calculate serum osmolality every 24h
Blood pressure (continuous/hourly):
Urine output every four hours: maintained (0.5ml/kg/hr.) Control of increased ICP
Calculated serum osmolality: every 24 hours (preferably
Positioning the head-keep in midline tilt up at 150-300
every 12 hours)
Temperature control-give antipyretics
Continuous oxygen saturation monitoring,
. Sedation-pain & arousal increase cerebral blood flow, so in-
Central venous pressure(CVP) for severe coma
creased ICP.can give diazepam 0.1-0.3mg/Kg dose
Ensure air way, breathing, circulation
Seizure control-phenytoin 15-20mg/Kg loading dose in
Treatment of shock, correction of hypoglycemia if present 1mg/Kg/min followed by 5-8mg/Kg/day

After initial resuscitation management include general and Hyperventilation-bag & mask ventilation
specific measures

129 | P a g e
FMAS, RUSL
UNDER THE GUIDANCE OF DR. Anurudhdha Padeniya
Done by amila priyadarshani 07/08,asanka 08/09, 3rd by Anusha 09/10

Pco2 25mgHg for 10-30s reduce ICP, gradually withdraw after 36) What is ADME
30-60min
1) Demyelination of white matter following an infection most
Mannitol 2.5-5ml/Kg of 20%mannitol over 30min.can repeat probably due to neuroimmunological response
every 4-6h 2.5ml/Kg/30min
37) What are the causes for encephalitis
Frusemide 1mg/Kg/12hrly
o Majority viruses -HSV, JE, Enterovirus, CMV, EBV
Thiopentone if all failed-5mg/Kg over 30-60min followed by
1mg/Kg/h infusion o Bacterial-TB

Fluid therapy-N/2 saline 2/3 or ¾ maintenance For infection o Malaria

Until bacterial cause excluded IV 3rd generation cephalo- o 25-50% no specific pathogen isolated
sporin
o Arthropod-borne viruses (especially West Nile virus, St.
Ex: cefotaxime + penicillin Louis, California, Lacrosse, and equine encephalitis viruses)

Antiviral therapy o Epstein-Barr virus


o Adenovirus
IV acyclovir for 14days o Human immunodeficiency virus
o Borreliaburgdorferi (Lyme disease)
Target Questions o Bartonellahenselae (cat-scratch disease)
Mycoplasma pneumoniae
35) What is encephalitis Rickettsia rickettsii
1) Encephalitis is acute inflammation of brain tissue.it almost Adapted from Willoughby RE
always accompanied by inflammation of adjacent meninges
38) How can prevent

130 | P a g e
FMAS, RUSL
UNDER THE GUIDANCE OF DR. Anurudhdha Padeniya
Done by amila priyadarshani 07/08,asanka 08/09, 3rd by Anusha 09/10

Vaccination- MMR, JE o By Arbovirus

Vector control o Commonest endemic encephalitis

39) What is the prognosis of encephalitis o Primarily in children, rare in young adult

o Depend on cause, severity, age o Vector is Culextritaneorhyncus

o Most completely recover o Amplifier host-pigs, birds

o Sever parenchymal involvement permanent neurological o Permanent neurological sequence 50%


damage or death
What are the complications of encephalitis?

Paresis
Herpes simplex encephalitis Spasticity,
Cognitive impairment
o Most common non epidemic encephalitis Weakness,
Ataxia and recurrent seizures
o An acute necrotizing infection HSV encephalitis – mortality rate over 70% and survivors
usually have severe neurological impairment
o Generally involve frontal & temporal cortex & limbic
system Counselling ….
o After neonatal period almost always by HSV1 Counsel the parents regarding the disease and the child’s cur-
rent condition

Acknowledge the parents about the possible complications


Japanese encephalitis and how it affects to child’s future life
131 | P a g e
FMAS, RUSL
UNDER THE GUIDANCE OF DR. Anurudhdha Padeniya
Done by amila priyadarshani 07/08,asanka 08/09, 3rd by Anusha 09/10

(If the child is infected with HSV encephalitis there may be


severe neurological impairment)

Emphasize the parents about the importance of drug compli-


ance

Acknowledge the parents about the MMR and JE vaccines and


vector control regarding the prevention of the disease

132 | P a g e
FMAS, RUSL
UNDER THE GUIDANCE OF DR. Anurudhdha Padeniya
Chamidri(07/08),Nayanajith (08/09),Ganga,Indunil (09/10)

Differential diagnosis:
Cerebral Palsy
Degenerative diseases:- FTT, poor sucking, weak cry, re-
PRESENTATIONS peated vomiting ,frequent epileptic seizures, developmental
regression , speech problems
Can have various presentations
Spinal cord tumors:- bladder problems / bowel problems
Developmental delay ,foot deformities, lethargy, poor feeding, irritable stridor
Abnormal posture Congenital myopathy:- family history of myopaty ,history
of reduce fetal movements, limb, face weakness, hypotonic.
No head control
Inherited metabolic disorders
Hand preference before age of 1 year
Metabolic neuropathy
Walking delay
History
Gait abnormalities
HPC ;- should include
Abnormal crawling
Possible etiology
Can be associated with
Complications and other associated disorders
Feeding difficulties
How was diagnosis made and things happen up to now
Irritability
How ADL affected and what has done upto now
Baby not responding to stimuli
Etiology
Visual loss/ deafness

Page | 133
FMAS, RUSL
UNDER THE GUIDANCE OF DR. Anurudhdha Padeniya
Chamidri(07/08),Nayanajith (08/09),Ganga,Indunil (09/10)

Start with birth history Epilepsy

Planned/unplanned/expected pregnancy

Preconception care (folic acid, rubella) infertility treatment ADL

Fever in first trimester, exposure with cats, history of her-


pes,VDRL,
Feeding-----sucking,bottle,spoon,food consistency,
UTI any antibiotic history duration to finish a meal

DM/PIH/APH/Recurrent MC/antiphospholipid sx,aspirin Toileting

USS—multiple pregnancy,IUGR,cerebal malformations Personal grooming

Perinatal;- PROM, foul smelling discharge of amniotic fluid Mobility

Maternal sepsis Communication

Labour;- term or pretem delivery, birth weight, prolonged or


obstructed labor, mode of delivery
Treatment done up to now
Post natal:-hx of meconium aspiration,hypoglyceamia,cried PAST M HX:-
Handle and sucked well,PBU admissions,neonatal sep- Any significant thing than above
sis,meningitis,neonatal convulsios
BIRTH HX:-
Developmental deformities;-age of mile stone achievement
As above
Learning difficulties
Page | 134
FMAS, RUSL
UNDER THE GUIDANCE OF DR. Anurudhdha Padeniya
Chamidri(07/08),Nayanajith (08/09),Ganga,Indunil (09/10)

IMM HX :- Child’s relationship with othes

Age appropriate any complication Housing condition—stairs,toilet,accidental prone areas

DIET HX:- Support from social services

Beast feeding how many times Nearest hospital and transport facilities

Bottle feeding, complementary feeding Accessto rehabilitation center

Way of using feeding equipments If child is schooling---transport facility,is there any special
unit to this child,school performance,attitude of teachers and
classmates
FAMILY HX:-
Physiotherapy given or not,is it given at home
3 generation pedigree neurological disorders,myopathies,me-
tabolism dx
EXAMINATION
Consanguinity, age of parents
General----
Inherited clotting disorders
\posture and abnormal movements(dystonia/tremor/athetoid
SOCIAL HX:-
movements)
How the condition has affected the family
Hygiene(look for nappies and clothes)
Father, mother education level/economy/knowledge on dx
Availability of walking aids/NG tubes/catheters
Sibiling trivial
Growth(measure height,weight,OFC)

Page | 135
FMAS, RUSL
UNDER THE GUIDANCE OF DR. Anurudhdha Padeniya
Chamidri(07/08),Nayanajith (08/09),Ganga,Indunil (09/10)

Dysmorphic features and manifestations of congenital infec- May have features of PDA (rubella syndrome)
tions
RS---
Eyes for squints,nystagmas,cataract
Signs of aspiration (pneumonia)
Sppech (no speech,slurred speech) dental hygiene
ABD---
Midline defects
Palpable bladder/fecal masses
bed sores
Feeding tubes
consioussness and intelligence
Gastrostomy/jeujostomy
DIB/wheezing
CNS---
Mouth drooling of saliva,grinding teeth
Gait—walking on tip toes,circumduction walk
Suspend the child by axilla-scissoring posture,lower extremi-
Short steps or rotated hip
ties
Broad based gait with arm spread

Muscle wasting/contractures/deformities

Tone—inceased (clasp knife rigidity)

CVS---- Power—weakness of the affected mucles

Murmur Reflexes –ankle clonus positive,deep tendon reflexes may be


exaggerated o brisk

Page | 136
FMAS, RUSL
UNDER THE GUIDANCE OF DR. Anurudhdha Padeniya
Chamidri(07/08),Nayanajith (08/09),Ganga,Indunil (09/10)

Babinski sign positive

Sensory

Cerebeller signs---intense tremor,past pointing MANAGEMENT

Cranial nerves To reduce the impact on the body

Check hearing and vision To improve quality of life

Rehabilitation is archived via multidisciplinary


approach
DEVELOPMENT ASSESMENT

1. Feeding---

-swallowing assessed by speech therapist


INVESTIGATIONS
-difficulty in swallowing –change the
consistency
Clinical diagnosis -introduce modified utensils
Depend on situation -NG tubes---
gastrectomy/gastrojejunostomy
MRI scan
-if growth failure or obese ----dietician
Test of hearing and visual function
advice

Page | 137
FMAS, RUSL
UNDER THE GUIDANCE OF DR. Anurudhdha Padeniya
Chamidri(07/08),Nayanajith (08/09),Ganga,Indunil (09/10)

-more pone to vit D and calcium -arrange motorized wheelchairs,special feeding


deficiency devices and customized seating arrangements with
head bands
2. Spasticity

-spastic diplegia
DRUG RX
Physiotherapy initially
-BDZ---sedetion is the side effect
Assistance of adaptive equipment;
walkers,poles,extending frames -beclofen---lower the seizure threshold

Hip disclocation; surgical,psoas In severe spasticity beclofen intra thecally via implanted pump
transfer and release
For localized spasticity -----botulinum toxin

For ataxia and movement disorders


If severe :rhizotomy(division of spinal nerve roots)
Atane ---dystonia movements
Spastic hemiplegia

-if tight heel cod---tenitomy of Achilles tendon


Resepine/terabenzine----atetosis and chorea
-constrained induces movements;non affected side
movements 3. Mobilization

Are restricted with casts while impaired side is


improved with exercises -if any special chairs are available at home assess,their suitability
Quadriplegia -povide walking aids with support of social services.

Page | 138
FMAS, RUSL
UNDER THE GUIDANCE OF DR. Anurudhdha Padeniya
Chamidri(07/08),Nayanajith (08/09),Ganga,Indunil (09/10)

(7). Behavioral Problems, ADHD


Help of Psychiatrist, Psychologist, Cognitive & behavioral
Therapist.
(8). Learning disabilities.
Arrange special schools
help of social services
(9). House modifications
Occupational Therapist & Social Services.
(10). Provide written information to parents.
(4). Bowel habits
(11) Prenatal treatment of magnesium (lower prevalence)
If not corrected constipation can lead to urinary tract obstruc-
tion & VUR
Target Questions
So Change the diet Fiber intake
H2o intake (1).What are the types of cerebral palsy?
(5). Micturition Spastic cp (70%)-
If recurrent retention clean intermittent catheterizing
Ataxic hypotonic cp (10%)
(6). Communication Skills
Dyskineticcp(10%0
Use of Bliss Symbols.
Talking typewriters Mixed pattern (10%)
Electronic Speech generating devices.

Page | 139
FMAS, RUSL
UNDER THE GUIDANCE OF DR. Anurudhdha Padeniya
Chamidri(07/08),Nayanajith (08/09),Ganga,Indunil (09/10)

All 4 limbs are affected

Arms>legs

Presentation - trunk in extensor posture,poor head


control,low central tone,seizures,microcepaly,severe
(2).what are the features of each types? intellectual impairment
Spastic cp Diplagia
damage to UMN pathway All 4 limb
Limb tone increase Legs>arms
Brisk deep tendon reflex Presentation - abnormal walking
Extensor plantar response
Ataxic hypotonic cp
3 types early trunk and limb hypotonia
Hemiplegic Poor balance,delay motor development,incordinate-
Unilateral involvement arm and leg movements,intentiontremor,ataxic gait

Arms>legs,face spared

Presentation - fisting of effected Dyskinetic cerebral palsy


hand,flexedarm,pronatedforearm,toe-heel gait in dyskinesia
effected side
Involuntary movements-chorea and dystonia and athetosis
Quadriplegia

Page | 140
FMAS, RUSL
UNDER THE GUIDANCE OF DR. Anurudhdha Padeniya
Chamidri(07/08),Nayanajith (08/09),Ganga,Indunil (09/10)

(3).what are the aetiology for cp?

intrauterine exposure to maternal infection (chorioamni- Level 2 Has the ability to walk indoors &outdoors&
onitis, inflammation of placental membrane, umbilical cord climb Stairs With a railing
inflammation, maternal sepsis) Has difficulty with uneven surface. Inclines or In crowds.
genetic factors Minimal ability to run & jump

prematurity (intra cerebral hemorrhage, periventricular leu-


comalasia) Level 3 Walks with assistive mobility devices in-
doors &outdoor: On level Surface.
multiple pregnancy
May be able to Climb stairs using a railing.
birth asphyxia
May propel a manual Wheelchair.
kernicterus

metabolic diseases Level 4 Walking ability Severely limited even 𝑐̅ as-


sistive devices. Uses wheelchairs most of two time. May
mitochondrial diseases
Participate in standing transfer.
4).According to Gross Motor Function Classification System to
Which level does your child belong ?
Level 5 Physical impairments restricting voluntary
Level 1 Can walk indoors & outdoors & climb
Control of Movements. Failed head & neck Control is im-
Stairs Without using hands for support.
paired in all areas of motor Function.
Can perform running & jumping but with decreased Speed,
Cannot sit or Stand independently , even 𝑐̅ adaptive
balance & Co – Ordination
equipment.

Page | 141
FMAS, RUSL
UNDER THE GUIDANCE OF DR. Anurudhdha Padeniya
Chamidri(07/08),Nayanajith (08/09),Ganga,Indunil (09/10)

Cannot independently Walk. If unresolving go for sx Options.


Eg : Tendon lengthening sx ( cleidiotomy )
5).What is a contracture.
Loss of mobility of a joint from congenital or acquired
causes & are caused by periarticular soft tissue Fibrosis or
involvement of muscles crossing the joint.

In L /L :Tendoachillis
Hamstring
Iliopsoas
Hip adductors.
In U/L : Bicep &Brachioradialis
Pronator
Long flexors of wrist
Type History
How do you manage Child with Contractures.
Spastic diplegia (35%) Legs > Arms Evident at the t
- Introduce exercises & R/V. If not improving can practice
crawling ( dragging the legs)
temporary measures such as. Difficulty in applying a diape
Stretch & Splints Unable to sit
Drugs Seizures – Minimal
Botulinum toxin if localized.

Page | 142
FMAS, RUSL
UNDER THE GUIDANCE OF DR. Anurudhdha Padeniya
Chamidri(07/08),Nayanajith (08/09),Ganga,Indunil (09/10)

Ataxic hypotonic Babinsk1+ Early trunk & limb hypotonic


balance

Spastic Hemiplegic (25%) Spontaneous Mov. Of affected M tone Equinovarus deformity of Thrombophilic do Infection
Hand Preference foot tip toe / circumductn gait Genetic
Arms > legs U/L flexed position Ankle clones+
Delayed walking Gait abnormalities Babinsk1+
Arms > legs U/L flexed position Ankle
1/3rd seizure Hx
Delayed walking Gait Babinsk1+
Learning difficulties
abnormalities 1/3rd seizure

Spastic Quadriplegia High association & MR & epilepsy Hx4 limbs.


Tone in all Ischemia
Arms > legs Learning difficulties
Brisk reflexes Infection
Flexion contractures
Spastic Quadriplegia High association & MR & Endocrine
Tone in all 4 limbs.
Speech / Visual
epilepsy Metabolic
Brisk reflexes
disturbances
Arms > legs Geneticcontractures
Flexion
Dyskinetic FreauentInvol Movements Chorea Dystonia Thetosis Speech / Visual
Asphyxia Kernicterus
Hypotonia disturbances
Poor head control
With marked head lag

Page | 143
FMAS, RUSL
UNDER THE GUIDANCE OF DR. Anurudhdha Padeniya
Chamidri(07/08),Nayanajith (08/09),Ganga,Indunil (09/10)

Dyskinetic FreauentInvol Movements Chorea Dystonia Thetosis Asphyxia Kernicterus


Hypotonia
Poor head control
With marked head lag
Ataxic hypotonic Early trunk & limb Incardination Intent’ tremor Ataxic gait
hypotonic Poor balance

Page | 144
FMAS, RUSL
Under The Guidance Of Dr. Anuruddha Padeniya
Done by Sampath and Nayanajith 07/08,Senthuran 08/09, Sandaruwan
and Nipuni 09/10
……………………………………………………………………………………………………………
………………………………………………

Epilepsy Idiopathic(70-80%)
Epilepsy Secondary

1) Cerebral disgenesis/ mal formation


2) Cerebral vascular occlusion
Seizure 3) Cerebral damage
a. Congenital infection
b. HIE
c. InterventricularHaemorrage
d. Ischemia
4) Cerebral tumour
5) Neurodegenerative disorder
6) Neurocutaneous syndrome

Non Epileptic
1. Febrile seizure
2. Metabolic
a. Hypoglycemia
b. Hypocalcemia/ low Mg
c. Low Na/ High Na
3. Head trauma
4. Meningitis/encephalitis
5. Poison/toxin

Paroxymal disorder

(funny turns)
Breath holding attacks

Reflex anoxic seizures

Syncope

Migrane

Beningn paroxysmal vertigo

Others

Cardiac arrhythmia

TICS

Page | 222
FMAS, RUSL
Under The Guidance Of Dr. Anuruddha Padeniya
Done by Sampath and Nayanajith 07/08,Senthuran 08/09, Sandaruwan
and Nipuni 09/10
……………………………………………………………………………………………………………
………………………………………………

Epilepsy seizure types

Generalized Focal

Absence seizures Parital


Myoclonic seizures Frontal
Tonic seizures Temporal
Tonic clonic seizures occipital
Atonic seizures

ILAE[2010] – classification [5s] fulfill the 5s during history taking and examination.

1. Seizure phenomena/seminology ( what mothers said)


Prodromal, aura, ictal, postictal

(Prodromal- not immediately befor the sizure,1-2 dys prior

Aura- immediate phenomenon just before the sizure)

2. Seizure type- focal,general


3. Seizure syndrome-age,sex,associated with development
4. Seizure aetiology-idopathic,secondary
5. Associated with complication( language deficiency syndrome,autism,ADHD)

Page | 223
FMAS, RUSL
UNDER THE GUIDANCE OF DR. Anurudhdha Padeniya
Done by Sampath and Nayanajith 07/08,Senthuran 08/09, Sandaruwan and Nipuni 09/10

History Fever

Malaise

Mild headache
Introduction -name, age, sex ,from
LOA
Mother-education and occupation

Relevant or not.
2.Aura-common with focal seizures depend on temporal
lobe involvement
P/C- abnormal body movement duration -Feeling smell
( 1st episode, diagnose patient with epilepsy) -Distortion of sound and shape(hallucination)
LOC -Chest discomfort

[Seizure type]............................................

3.Ictal phase
H/P/C- Describe what happens [What the child was doing prior to -consciousness
incidence]
-duration

-frequency

-trigering factors.
[Seizure semiology-what mother said]

1.Prodromal period- non specific symptoms


Page | 224
FMAS, RUSL
UNDER THE GUIDANCE OF DR. Anurudhdha Padeniya
Done by Sampath and Nayanajith 07/08,Senthuran 08/09, Sandaruwan and Nipuni 09/10

a.Focal Tonic- generalised increase tone

1.Simple- temporal-auditory or sensory smell test Tonic and clonic - increase tone, cyanosis, eye rolled
up, LOC, deep sleep
Frontal-motor phenomenon
Incontinence, tongue bite
Parietal-contralateral altered sensation
Atonic-transient loss of tone,fall on the floor or drop
Occipital-positive or negative phenomenon head

2.Complex-automatism
4.Post ictal
Altered behavior Child what did-recover fully
Lipsmaking Drowsy ,sleep ,LOC-duration,paralysis
Chewing At what time happen-how long?, after that what does the mother
Swallowing did?

Excessive salivation
Is this first episode or not

3.2ndary generalized Already diagnosed- is it similar to previous one

b.Generalized Drug compliance, missed drug

Absence-transient LOC+some flickering eyelids Sleep deprivation

Myoclonic-repetitive jerky movement Watching TV


Page | 225
FMAS, RUSL
UNDER THE GUIDANCE OF DR. Anurudhdha Padeniya
Done by Sampath and Nayanajith 07/08,Senthuran 08/09, Sandaruwan and Nipuni 09/10

How long- how many episodes.

Last episode what happen. 1st episode wt happen.

Exclude DD
1. Syncope- long standing was on hot environment. Complication

2. Migraine- aura headache tightness, GIT disturbance. 1. Latter onset of epilepsy

3.Recurrent vertigo- standing position, unsteadiness, history of 2. Language delay syndrome


falling. 3. Altered behavior
4.Cardiac cause-palpitation 4. HIE
5.Head trauma-

6.SOL-early morning vomiting, nausea

7.Metabolic-dehydration- diarrhea vomiting PMHx - DM, migraine , malaria ,recent mumps ,measles,
Last meal- glucose chickenpox

8. meningitis. fever, altered behavior. If already diagnosed patient

Describe- 1 stattack -duration, what happen.


Aetiology
Idiopathic(70-80%) EEG done-result MRI/ct done or not
Secondary
Page | 226
FMAS, RUSL Cerebral disgonosis/ mal formation

Cerebral vascular occlusion


UNDER THE GUIDANCE OF DR. Anurudhdha Padeniya
Done by Sampath and Nayanajith 07/08,Senthuran 08/09, Sandaruwan and Nipuni 09/10

Other Ix& results.

Similar other episodes +/- ferquency Developmental Hx - - State whether is it appropriate or


not
Clinic follow up
immunisation - JE given /not
On what drugs
Pertusis (not give when there is progressive neurological disease)
Were drugs changed

Compliance Allergy Hx -
SE of drugs –weight gain ,alopecia Drug Hx- What are the drugs child is taking, when changed last.

FHx - Hx of epilepsy in 3 generation Pedigree


PSH- head surgery Consanguinious Other neurological disorders

Birth Hx -

Antenatal period - Infection , APH, PDH, GDM, trauma Social Hx


intra partum - Prolonged/ Precipitated labour, Instrumental Parent education. economically stable /Not ,House condition,
delivery knowledge on dx , prevention, drug compliance, what to do when
Post partum - Prolonged jaundice a seizure occur, teachers know about the dx ,school performance of
child , social interaction ,Hazardous at home
Neonatal seizures meningitis ,sepsis
Near road
Birth wt, term /not, mode of delivery.

Page | 227
FMAS, RUSL
UNDER THE GUIDANCE OF DR. Anurudhdha Padeniya
Done by Sampath and Nayanajith 07/08,Senthuran 08/09, Sandaruwan and Nipuni 09/10

Uncovered well Neurocutaneousxd-

Swimming, cycling , climbing trees. Tuberous sclerosis

Depigmentations (‘ash leaf ‘ shape patches )

Crisis plan- Close hospital, Emergency transport Shagreen patches


,Impact on family other siblings ,Parent’s fear
,knowledge on prognosis Child’s idea about the dx Adenoma sebaceum

Subungualfibromata

Examination

OFC- microcephaly , Hydrocephaly Neurofibromatosis

≥6 Café –au – lait spots


Wt,Ht
> 5 mm – before puberty
Febrile +/-
> 15 mm – after puberty
Pallor ,Cyanosis
>1 Nearofibroma
Sings of head trauma

E.N.T infection Axiliary freckle

Lymphadenopathy Sturge Weber dx – Port wine stain.

Tense fontanellae

Fundi (increased ICP) CVS – PR-arrhythmia, murmurs.

Page | 228
FMAS, RUSL
UNDER THE GUIDANCE OF DR. Anurudhdha Padeniya
Done by Sampath and Nayanajith 07/08,Senthuran 08/09, Sandaruwan and Nipuni 09/10

RS -any abnormality, cyanosis. • Altered responsiveness or impairment of consciousness /


awareness.

3• What is a convulsion?
Abdomen- Hepatosplenomgaly
Predominantly, an uncontrollable & involuntary
Polycystic kidney- tuberous sclerosis. contraction/relaxation or spasm of a group or groups of muscles.

4• What is epilepsy?
CNS- Full neurological examination. The term epilepsy is generally used when a person has a tendency to
general - posture, defomities, squint, wasting, fundi, have unprovoked, repeated seizures (minimum of two).misdiagnoses
are common.
craneal nerve examination. upper/lower limb examination see any
focal neurological findings.
5.how do you Mx this chid?
Possible questions
(a) INVESTIGATIONS
1• What is a seizure?
• Electroencephalography (EEG) EEG is performed to support the
A seizure is the manifestation of an abnormal, paroxysmal discharge diagnosis of epilepsy.
of a group of cortical neurons. This discharge may produce
o EEG is usually done after the second epileptic seizure but in
subjective symptoms or objective signs.
certain circumstances, as evaluated by a specialist, may be
2• What are the key features of a seizure? considered after first epileptic seizure.

• Paroxysmal nature of the event. o EEG should be done when there is uncertainty about the diagnosis
of the conditions mentioned earlier in the differential diagnosis.
• Associated abnormal movements / subtle phenomena.

Page | 229
FMAS, RUSL
UNDER THE GUIDANCE OF DR. Anurudhdha Padeniya
Done by Sampath and Nayanajith 07/08,Senthuran 08/09, Sandaruwan and Nipuni 09/10

o EEG helps to determine the seizure type such as generalized Indications for MRI
seizures, focal seizures, myoclonic seizures etc.
a) If the patient is < 2 years.
o EEG helps to determine epileptic syndromes such as West
syndrome, benign Rolandic epilepsy,absence seizures etc. b) If there is evidence of focal onset on history, examination and
EEG (provided it is not benign focal epilepsy).
o In children presenting with the first unprovoked seizure, the
epileptiformactivity on the EEG may help to assess the risk of c) In suspected neurocutaneous syndromes.
seizure recurrence. d) If seizures persist despite treatment with first line drugs.
o Repeat EEG may be helpful if the diagnosis is not certain. This e) In neurodevelopment regression.
should be a sleep or a sleep deprived EEG (In children a sleep EEG
is best achieved through sleep deprivation or use of chloral or (CT scan is used to identify underlying gross pathology if MRI is
melatonin) not available

o Video EEG may be used in patients with diagnostic difficulties. ( or if anaesthetic necessary for MRI.)
eg; pseudo seizures, night terrors)
• Other Investigation (when applicable)-
0. Epileptic encephalopathy:
When frequent disabling seizures often accompanied by 1. ECG and Holter monitoring in suspected cardiac syncope /
severe epileptiform EEG abnormalities result in neurological Long QT sndrome
and cognitive impairment - west syndrome ,lennox-gaxtaut syndrome.
2. Blood sugar, electrolytes, calcium and phosphorus to deter-
• Neuro Imaging mine an underlying cause for the epilepsy.

Neuro Imaging should not be routinely requested when a diagnosis 3. Metabolic screen
of epilepsy has been made.
• Neuropsychological Assessment
MRI should be the investigation of choice in epilepsy.
is necessary if,

Page | 230
FMAS, RUSL
UNDER THE GUIDANCE OF DR. Anurudhdha Padeniya
Done by Sampath and Nayanajith 07/08,Senthuran 08/09, Sandaruwan and Nipuni 09/10

a) Child with epilepsy has educational difficulties. • Children with febrile seizures, even if recurrent should not be
treated with long term AED.
b) MRI has identified abnormalities in cognitively important brain
regions. Choice of first AED depends on

c) Child is found to have memory loss or cognitive deficit or • The seizure type/ syndrome
decline.
• The potential adverse effects

• Co- morbidity
(b) PHARMACOLOGICAL TREATMENT OF EPILEPSY
• The availability and cost
Anti-epileptic drugs (AED) should only be started once the
diagnosis of epilepsy is confirmed.

Initiation of AED Principles of AED Therapy

• In children, should be by a Paediatrician/ Paediatric Neurologist. • Should use monotherapywherever possible

• Is generally not recommended after a first unprovoked tonic- • Unsuccessful initial therapy, try monotherapy with another drug
clonicseizure. • Ifmonotherapyin the maximum dose has failed, a second drug
• May be considered after a first unprovoked seizure if ; should be started. The second drug could be alternative first line.

o the individual has a neurological deficit • If the second drug reduced the seizure frequency, taper off the first
and continue monotherapy with the second.
o a further seizure is unacceptable to the family
• If there is no improvement within a month, taper off either the first
o brain imaging (where indicated) shows a structural abnormality or the second, depending on their relative efficacy.

Page | 231
FMAS, RUSL
UNDER THE GUIDANCE OF DR. Anurudhdha Padeniya
Done by Sampath and Nayanajith 07/08,Senthuran 08/09, Sandaruwan and Nipuni 09/10

• If both drugs do not work, another second line drug may have to be Newer AED for uncontrolled epilepsy
introduced as monotherapy.
1.Vigabatrin
• If the response is poor consider blood levels if facilities are
available. 2.Topiramate available in Srilanka

3.Lamotrigine
Consider add on or combination therapy only when
monotherapy has failed. Prior to initiation of combination 4.Gabapentine
therapy consider the following.
1.Levetiracetam
• Is the diagnosis correct?
2.Oxcarbazepine
• Adherence to treatment
3.Tiagabinecurrently not available
• The appropriateness of the AED for the seizure type.
4.Sulthiam
• The quality of the drug
5.Stiripentol
Long term AED therapy

• Should be planned by a specialist.


Withdrawal of AED therapy should
• Involves adjustment of drug dosage according to the weight.
• be individualized.
• Should include discussion with the individual regarding possible
side effects, rationale of treatment and what should be done if a dose • be under the guidance of the specialist.
is missed or during illness.
• be considered in those who have been seizure free for at least two
• Should involve a simplified medication regimen with clear verbal years.
and written instructions..

Page | 232
FMAS, RUSL
UNDER THE GUIDANCE OF DR. Anurudhdha Padeniya
Done by Sampath and Nayanajith 07/08,Senthuran 08/09, Sandaruwan and Nipuni 09/10

• be done slowly ( at least over 2 to 3 months) by reducing 10=25 % Childhood & Na Topiramate, Cmz,
in each 1-2 week juvenile absence valproate[both], lamotrigine phenytoin,
ethosuximide[both vigabartin
• take longer (up to 6 months or longer) when withdrawing chilhood]
benzodiazepines and barbiturates. Focal epilepsy Cmz, Na Clobazam,
valproate, topiramate,
• be abandoned if seizure recurs. lamotrigine,
phenytoin
• not involve routine EEG prior to withdrawl of treatment. Myoclonic Na valproate lamotrigine Cmz,
phenytoin,
vigarbartin
Infantile spasm ACTH, Na Cmz
Prednisolone, valproate,
Vigabatrin clonazepam,
topiramate,
Benign rolandic Cmz, Na Topiramate, Cmz
epilepsy[BCECTS] valproate, lamotrigine
Lennox- gastautxn Na valproate, ACTH Cmz
clobazam Topiramate,
lamotrigine
Drug Side-effects
Drug options by type of epilepsy or epilepsy syndrome.

Valproate- Weight gain, hair loss,Rare idiosyncratic liver failure


Epilepsy 1st option 2nd option Drug best
syndrome avoid Carbamazepine/oxcarbazepine-Rash, neutropenia, hyponatraemia,
Generalize T-C Na valproate, cmz Topiramate, vigabartin ataxia Liver enzyme induction, can interfere with other medication
lamotrigine

Page | 233
FMAS, RUSL
UNDER THE GUIDANCE OF DR. Anurudhdha Padeniya
Done by Sampath and Nayanajith 07/08,Senthuran 08/09, Sandaruwan and Nipuni 09/10

Vigabatrin- Restriction of visual fields, which has limited its use This above definition includes convulsive as well as non-convulsive
Sedation seizure disorders,

Lamotrigine- Rash For practical purposes, the approach to the child who presents with a
tonicclonicconvulsion lasting more than 5 minutes should be the
Ethosuximide Nausea and vomiting same as the child who is in “established” status2. 5% of children
Topiramate- Drowsiness, withdrawal and weight loss with febrile seizures and 1-5% of epilepsy patients develop status
epilepticus. Overall mortality is 10-15%.
Gabapentin- Insomnia
Lab Studies
Levetiracetam Sedation – rare
Laboratory studies should be done according to the likely diagnosis
Benzodiazepines (clobazam, clonazepam,diazepam, nitrazepam based
)-Sedation, tolerance to effect, increased secretions
on age, history and clinical signs.
All the above may cause drowsiness and occasional skin rashes.
• Blood glucose using immediate bedside testing (e.g. Dextrostix)

• Serum electrolyte, Calcium and magnesium

• Liver function tests, blood urea/creatinine


MANAGEMENT OF AN ACUTE SEIZURE & STATUS
EPILEPTICUS • Arterial blood gas

Definition • Toxicology screen (always keep some blood for future tests)

Status epilepticus (SE) is defined as clinical seizure activity lasting • Anticonvulsant levels (if indicated by history of ingestion or
more than 30 minutes, constituting a neurological emergency. existent therapy and if available)
Seizure activity may be either continuous or intermittent without the • FBC and septic work-up
patient recovering consciousness in between.

Page | 234
FMAS, RUSL
UNDER THE GUIDANCE OF DR. Anurudhdha Padeniya
Done by Sampath and Nayanajith 07/08,Senthuran 08/09, Sandaruwan and Nipuni 09/10

• Blood film for malarial parasites 2wk-10mg 8hly

Imaging Studies: Not essential unless clinical evidence supports a 3wk-10mg 12hly
CNS lesion.
4wk-10mg mane
• Stabilize all children before CT scanning or other imaging studies
are performed. Obtain imaging studies based on likely aetiologies. Then plan to repeat EEG,f spasm continue to be present despite of
steroid Rx (10mg 6hly) can increase dose upto 15mg 6hly for
• Cervical spine x-rays, if potential trauma mximum2wk,if still present called as Steroid resistant infantile
spasm
• A head CT scan is the best diagnostic imaging study, particularly if
the following are suspected:

Haemorrhage,Midlineshift ,Mass lesion Councelling & advices

• MRI is not a diagnostic tool, unless it is immediately available and Introduction


the child’s cardio-respiratory status is stable.
The term epilepsy is generally used when a person has a tendency to have
Electroencephalogram unprovoked, repeated seizures (minimum of two).

A seizure is the manifestation of an abnormal, paroxysmal discharge of a


Lumbar punctureis indicated only under following circumstances
group of cortical neurons. This discharge may produce subjective
(once the patient is stable and there is no evidence of raised ICP): symptoms or objective signs.

• For prolonged status epilepticus of unknown etiology. key features of a seizure

Paroxysmal nature of the event.


• For suspected cases of meningitis or encephalitis.
Associated abnormal movements / subtle phenomena.
Management of infantile spasm-
Altered responsiveness or impairment of consciousness /
1wk-10mg 6hly
awareness.

Page | 235
FMAS, RUSL
UNDER THE GUIDANCE OF DR. Anurudhdha Padeniya
Done by Sampath and Nayanajith 07/08,Senthuran 08/09, Sandaruwan and Nipuni 09/10

Usually self limiting, serious injury is rare.

If age appropriate advice both child and parent about If they are in dangerous place (read, by a fire, near sharp objects,
at the top of stairs, by edge of water) patient should be moved to a
The aim of TX & the need for its continuity even if seizures are safe place.
controlled, should be fully explained. Stress the necessity of regu-
lar medication & compliance-not curative but rather suppress sei- Avoidance of precipitants-sleep deprivation, flickering of lights
zures. Dose adjustments & changes may require. from TV, computer games (if only photosensitivity is there) infec-
tions.
Advice care givers about risk of abrupt withdrawal of antiepileptic
drugs. Severe seizures usually follow abrupt AED withdrawal. Don’t restrain the patient’s seizure.

Discuss the AED side effects.specially sedative effects of some Turn patient to left lateral position - To prevent tongue blocking
AED & some possible interactions. airway

Child’s guardian, teacher & child himself (if age appropriate) must No attempt should be made to open patient mouth or force any-
understand type of seizures & type of medication & their side ef- thing between teeth.
fects.
Wait until stop seizure no medical attention is needed for most sei-
Advice to tell & give a card to show when taking medications zures.
from GP or any other place (to prevent drug interactions).
However come to hospital if,
Seizure frequency & medication intake must be documented in an Seizures last > 5 min.
accurate record or calendar (helpful for both child & physician).
If followed immediately by another seizure.
The risk of your child to develop epilepsy in the later life is very
rare. If this is the patients 1st seizure

Treat your child as a normal child. The patient is injured.

How to cope & seizure

Page | 236
FMAS, RUSL
UNDER THE GUIDANCE OF DR. Anurudhdha Padeniya
Done by Sampath and Nayanajith 07/08,Senthuran 08/09, Sandaruwan and Nipuni 09/10

Activities & environment Need frequent clinic visits in titration & adjustment phase of
AED.some can tapered off drugs if they are seizure free for 1-2
Restrictions should be according to patient’s seizure frequency yrs.it will be decided by physician.
Cautions
Asses;
Climbing trees/ladders
Child general school performance
Bathing /swimming in drivers
Side effects of AED
Open fires
Excess nystagmus, tremor, ataxia.
Riding bicycles
Baseline & follow up blood testing eg: liver enzyme levels.
Protect open spaces in upstairs
Growth & development monitoring-wt, ht, OFC, milestone.
Cover unprotected wells, garbage pits
The risk of your child to develop epilepsy in the later life is very
School activities: most children at epilepsy can they rare.
should attend normal schools. Their teachers should be Treat your child as a normal child
correctly informed about epilepsy & encouraged to have
open minded &optimistic attitude toward condition.
School teacher & class mates should be educated about
what to do when seizure develops.

Sports: restricts boxing & sports may resulting repeated


head truma, it will cause further neurological damage. No
adventures sports. If child has a good control of seizure,
only minimal restriction needs to be placed over child’s
activities.

Advises regarding follow up

Page | 237
FMAS, RUSL
UNDER THE GUIDANCE OF DR. Anurudhdha Padeniya
Done by Sampath and Nayanajith 07/08,Senthuran 08/09, Sandaruwan and Nipuni 09/10

Page | 238
FMAS, RUSL
UNDER THE GUIDANCE OF DR. Anurudhdha Padeniya
Done By Susita Kalum Liyanage 07/08, 2nd Edition By Nuwan 08/09, 3nd Edition By Lakshitha Bandara
09/10

Febrile Convulsion
Etiology for fever
HISTORY Ear infection – earache, itching, discharge
Introduction RTI – cough, cold, sore throat wheezing
Age (FC occur in 6m to 5y) UTI – dysuria, frequency
Sex (slight male predominance) GIT – loos stools, exposure to unhygienic food
From where and how many km away from hospi- Skin infection
tal(THA) Muddy water contact, river bath (Leptospirosis)
History taken from whom, Recent travel history (Dengue), Contact history
Educate up to which level/grade for viral fever
And was she with the child at the time where Any other infective foci
convulsion occur Seizure
History much reliable or not (Ask to eye witness)
P/C Preictal period
Fever x duration Behavior
Convulsion x duration Aura +/-; epigastric pain, chest pain , fear , cry (to
Hx of P/C exclude epilepsy)
Fever Mood - upsetting, pain, anger, frustrating (to ex-
Onset, grade ( low 99 – 100, high > 103 ) clude the epileptiforms)
Documented or not, pattern, Associated Ictal period
with chills and rigors or not Onset (At what time, what has been doing at the
Child well or not at the time between fe- time)
ver spikes Character ( focal/ generalized, tonic clonic or not)
What is doing mother for fever at home ( Duration (more than 15 min or less )
antipyretics with doss and frequency and Posture (flexion, extension)
responded to PCM or not, wet )
Page | 239
FMAS, RUSL
UNDER THE GUIDANCE OF DR. Anurudhdha Padeniya
Done By Susita Kalum Liyanage 07/08, 2nd Edition By Nuwan 08/09, 3nd Edition By Lakshitha Bandara
09/10

Signs of true seizer (urinary and/or faecal inconti-


nence, eye rolling, tongue bite, clenching teeth,
EPILEPTIC Causes
frothy in mouth)
Consciousness, verbalization Idiopathic (70–80%) – known but presumed genetic (FHx)
Cyanosis/pale
Symptoms of automatism (chewing, swallowing, Secondary to
• Cerebral dysgenesis/malformation
smacking, in partial epilepsy)
• Cerebral vascular occlusion
Postictal period • Cerebral damage, e.g. congenital infection
Drowsy/ sleepy/ headache/ with duration ,
hypoxic-ischaemic encephalopathy
Hemi paresis ,
intraventricular haemorrhage/ischaemia
Baby having aspiration feature ( cough, dysphonic)
Cerebral tumour
What did mother do?
Neurodegenerative disorders
After fit- medical interventions (GP, hospital admis-
sion, treatment, prognosis Neurocutaneous syndromes

Exclude DD for fever fit

NON EPILEPTIC Causes


Poisons/toxins

CNS inflammation Metabolic


Other benign condition
Meningitis/encephalitis Hypoglycaemia Reflex anoxic Seizures
Hypocalcaemia Breath-holding Attacks
Cerebral abscess hypomagnesaemia Syncope
Hypo/hypernatraemia
SOL
Cerebral malaria Page | 240
FMAS, RUSL
Head trauma
UNDER THE GUIDANCE OF DR. Anurudhdha Padeniya
Done By Susita Kalum Liyanage 07/08, 2nd Edition By Nuwan 08/09, 3nd Edition By Lakshitha Bandara
09/10

Exclude DD for fever fit


Past medical Hx
CNS inflammation recurrent attacks of seizer with fever
Meningitis- ill, vomiting, headache, photophobia, recurrent infection
neck and back pain.
Viral – mumps, measles, chicken pox-
Encephalitis- bleeding , drowsy , irritability ,De-
crease level of conscious level+ meningioma’s signs Bacterial - Hx of sepsis

Cerebral abscess – recurrent head trauma otitis media No past history of epilepsy, meningitis,
, sinusitis encephalitis, DM, electrolytes imbalance,

SOL- unilateral weakness, frequent falling ,difficulty Past surgical Hx


in walking Meningoceole, meningomyoloceolrepair, neuro-
surgeries,
head trauma
spleanectomy, prosthetic valves (more vulnerable
to infection)
Metabolic Drug Hx
hypoglycemia- fasting, diagnosed DM Long term use of antibiotics, anticonvulsant Hx
hypernatremia- vomiting , diarrhea
Allergy Hx
Hypocalcaemia – numbness and tingling sensation in Allergy for food, drugs, plaster
perioral area or Birth Hx
in fingers and toes and muscle cramps
Antenatal- planned pregnancy, maternal infec-
Toxins tion.
exposure to jewelries , batteries, chemicals

Page | 241
FMAS, RUSL
UNDER THE GUIDANCE OF DR. Anurudhdha Padeniya
Done By Susita Kalum Liyanage 07/08, 2nd Edition By Nuwan 08/09, 3nd Edition By Lakshitha Bandara
09/10

Intranatal -NVD/LSCS with reason, instrumental Any development delay.


delivery, at preterm or pre term?
Family Hx
LBW, birth trauma, insult , asphyxia, resuscita- Marriage of parent consanguineous/ not
tion
FHx of neurodegenerative dx (Parkinson’s, Alz-
Post natal- jaundice, sepsis, meningitis, encepha- heimer’s, Huntington’s) , Neurocutaneous dx
litis, neonatal hypoglycemia, fits congenital heart (neurofibromatosis, tuberous sclerosis), FC, epi-
disease. lepsy

Diet Hx
Social Hx
Immunization Hx Education – school performance, Attendance, so-
MMR (slight risk of getting Fibril Convulsion cial bag ground
within 1-2 weeks because
Economy status,
Both fever and rash tend to occur 7 to 10 days
after administration Knowledge of illness, drugs, prognosis, preven-
tion, how to act on emergency.
And may last up to 1 -2 days
Parental anxiety
DPT-Hib(slight risk getting Fibril Convulsion
within 1-2days) Hazards in house., environment- unprotected well
, wewa, tank
Risk decrease with no of vaccinated got
Nearest hospital and transport
JE within last year ( is a risk factor)
Need 12 month of convulsion free period to give
the vaccine
Developmental Hx

Page | 242
FMAS, RUSL
UNDER THE GUIDANCE OF DR. Anurudhdha Padeniya
Done By Susita Kalum Liyanage 07/08, 2nd Edition By Nuwan 08/09, 3nd Edition By Lakshitha Bandara
09/10

EXAMINATION Joint tenderness

Aim- find out the cause of fever, conform the febrile con- Back- spine , scar
vulsion by excluding DD for febrile fit

General
Anthropometry- wt/ht,lenth/ofc with centile Meningeal signs- kerning’s, brudzinki’s, neck
stiffness
Appearance – ill/well/drowsy, consciousness , ir-
ritability, posture, cannula Lymphadenopathy

Head- signs of trauma

Febrile or not Odema

Pale/icteric CVS
Pulse, BP
ENT, oral hygiene
Murmurs
Cyanosis
RS
Hydration Signs of RTI

Skin lesion –infective foci , Neurocutaneous man- ABD


ifestation Hepatomegaly (metabolic disorder)

BCG scar Ballatable kidneys ,renal angle tenderness

Hand- clubbing Genitalia

Peripheral stigmata of SABE, CLD CNS

Page | 243
FMAS, RUSL
UNDER THE GUIDANCE OF DR. Anurudhdha Padeniya
Done By Susita Kalum Liyanage 07/08, 2nd Edition By Nuwan 08/09, 3nd Edition By Lakshitha Bandara
09/10

Full CNS examination and confirm it’s normal. LP- not for all but for,

1. Pt with sign and symptoms of CNS inflamma-


tion

- History of irritability,
DD’s
- Decrease feeding,
Febrilconvaltion
- Lethargy, drowsy, systematically
epilepsy ill.
Non epilecticattak - Clinical signs of meningitis, en-
Epileptiform attac cephalitis

INVESTIGATION 2. Prolong post ictal altered consciousness or


neurological defect.
Etiology of fever- FBC (WBC/DC, Hb, Plt) ESR,
CRP 3. Probably child is <18 month or almost cer-
tainly <12 month,
SE- in febrile convulsion can be abnormal (Hypo/
hypernatremia)
At the occurrence of first fit.
in a case of electrolyte
imbalance ( dehydration, vomiting) 4. After complex convulsion

RBS- should exclude hypoglycemia, 5. After pretreatment with antibiotics.

if prolong seizure/ prolong post-Ictal EEG to


impairment of consciousness/prolonged fasting CT/MRI

Page | 244
FMAS, RUSL
UNDER THE GUIDANCE OF DR. Anurudhdha Padeniya
Done By Susita Kalum Liyanage 07/08, 2nd Edition By Nuwan 08/09, 3nd Edition By Lakshitha Bandara
09/10

2). Seizure – acute Mx

Management 3). Advise parent and prevent febrile fit.

1). Fever Counseling of parents

Adequate fluid intake.

Physical method to reduce body temperature Introduction


–fanning, light clothing It is a condition which is provoked by increased body
temperature,
Antipyretics Commonly occur in children between 3 months to 5
years.
o Parasitamol- max 4 doses in 24 Your child had developed once, so she has a chance to
hours develop again & again

- <3/12- 60mg If the temperature is not coming down or if you have any
other concerns consult a doctor.
- 3/12-1yr- 60-120 mg
When there is fever (>38.5c) child will be hot so should take
- 1-5yr- 120-250mg
their temperature regularly.
o Ibuprofen (3-4 times per day)
If child has a persistent high Temperature please brings the
- 1-2yr – 50mg child to hospital immediately.

- 3-5yr – 100mg Give extra fluids (eg: more breast feed, water) and
Give some easily digestible watery diet & boiled wa-
(Suspecting dengue ter
avoid NSAIDs)
Dress your child lightly. keep the child under the fan.

Page | 245
FMAS, RUSL
UNDER THE GUIDANCE OF DR. Anurudhdha Padeniya
Done By Susita Kalum Liyanage 07/08, 2nd Edition By Nuwan 08/09, 3nd Edition By Lakshitha Bandara
09/10

Do not bathe in cool water and Do not give tapid After every convulsion episode you have to consult
sponging by cloth soaked in cold water the doctor.(Even convulsion occur less than 1
minutes)
(This can cause shivering, increase the body’s
core Temperature further) Vaccination of child

Give paracetamol according to the weight of your JE Need 12 month of convulsion free period to
child 15mg/kg-6 hourly. give the vaccine
. Ask mother to inform health care worker who
is going to vaccinate the child

Despite all these measures if your child develop convulsion.


MMR (slight risk of getting Fibril Convulsion
Don’t run here & there taking the child. within 1-2 weeks because
Keep the child in a safe place & put in a safe place &
put in left lateral position. Both fever and rash tend to occur 7 to 10 days
after administration
Remove any harmful objects surrounding the child.
And may last up to 1 -2 days
Don’t keep anything inside the child’s mouth.
DPT-Hip(slight risk getting Fibril Convulsion
Observe the pattern of convulsion, because you have within 1-2days)
to describe this to doctor.
Risk decrease with no of vaccinated got
If the convulsion continues immediately bring the
child to the hospital. Ask mother to alert and observe the child, if
getting fever consult your doctor

Page | 246
FMAS, RUSL
UNDER THE GUIDANCE OF DR. Anurudhdha Padeniya
Done By Susita Kalum Liyanage 07/08, 2nd Edition By Nuwan 08/09, 3nd Edition By Lakshitha Bandara
09/10

Target question Provoked fits; by somatic disorders originating outside the


brain. Eg- high fever , infection, hypoxia, head trauma,
1. What is the seizure? toxins..eg
Manifestation of an abnormal, paroxysmal discharge of a 1. Febrile seizure
group of cortical neurons causing proximal rhythmic,
involuntary abnormal movements (stiff/relax)sensation or 2. Metabolic
behavior with altered or impaired consciousness.
- Hypoglycemia
2. Classification and different between them
- Hypocalcaemia
Epileptic
- Hypomagnesaemia
Unprovoked fit; seizures recurrently triggers within the brain
eg: - Hypo/hypernatremia

1. Idiopathic (70-80%) 3. Head trauma.

2. Secondary- cerebral digenesis/malformation, cerebral 4. Meningitis/ encephalitis


vascular occlusion cerebral damage( congenital infection, 5. Poison/toxin
hypoxic ischemic encephalopathy, IVH)
Epileptiforms/mimickers of epilepsy
3. Cerebral tumor.
Not due to abnormal electrical impulses of the brain. Eg;
4. Neurodegenerative disorder.
1. Syncope
5. Neurocutaneous disorders.
2. Breath holding attacks.
Non- epileptic
3. Pseudo seizures(NEAD)

Page | 247
FMAS, RUSL
UNDER THE GUIDANCE OF DR. Anurudhdha Padeniya
Done By Susita Kalum Liyanage 07/08, 2nd Edition By Nuwan 08/09, 3nd Edition By Lakshitha Bandara
09/10

4. Cardiac origin- HOCM, stoke Adams ,SSS, prolong QT - No brain damage, intellectual performance simi-
Xn lar to normal.

5. Sleep disorders - Eye rolled up

6. Restless leg Xn Complex fc

3. What is the febrile convulsion - Focal/

A non-epileptic seizure occurring in child hood associated - Tonic clonic


with febrile illness, not caused by an infection of the CNS,
metabolic imbalance, without previous neonatal seizure or - Lasts >15 minutes
unprovoked seizure and not meeting the criteria for other - Recurs within 24 hours / 2 episode in same illness
acute symptomatic seizures.
- Chance of developing epilepsy 4-12%
4. Classify fc and how are going to differentiate them?
- Usually eyes rolled up.
Simple FC
Febrile status epilepticus
- Primary generalized
- If>30 mins may a seizure or series of seizure oc-
- Usually tonic clonic curred.
- Last<15 minutes - Commonly; may occur signs of true seizure (uri-
- No recurrence within 24 hours/ 1 episodes in nary and fecal incontinence, eye rolling up,
same illness. clenching teeth , frothy in mouth)

- Chance of developing epilepsy 2-5% 5. Briefly describe pathophysiology

Page | 248
FMAS, RUSL
UNDER THE GUIDANCE OF DR. Anurudhdha Padeniya
Done By Susita Kalum Liyanage 07/08, 2nd Edition By Nuwan 08/09, 3nd Edition By Lakshitha Bandara
09/10

When high grade fever occur following child hood - Fast hx of epilepsy
infections, URTI, viral fever, MO , AGE reduces seizure
threshold and occur - Complex fc

6. Who are more prone to get recurrence? - Abnormal neurological finding

Major criteria - 8. Indications for admission to hospital after fc

- Age<1 yr - First febrile fit.

- Fever >24 hrs - Age <18 months

- Fever 38-39 c - Incomplete recovery after one hr

Minor - Any kind of CNS infection


- Family Hx of fc in first degree relatives but not - A complex fc
epilepsy
- Fever has lasted> 48 hr before onset of fever
- Complex fc
- Home in adequacy/ excessive parental anxiety
- Male gender( M:F =2:1 ) and inability to cope
- Low serum Na

- Maternal alcoholic &smoking during pregnancy 9. Is fc harmful


- Residual weakness after fit. - Vast majority harmless
7. What are the risk factors for developing epilepsy? - During fit small chance of injured by falling,
- Age <9/12 chocking

Page | 249
FMAS, RUSL
UNDER THE GUIDANCE OF DR. Anurudhdha Padeniya
Done By Susita Kalum Liyanage 07/08, 2nd Edition By Nuwan 08/09, 3nd Edition By Lakshitha Bandara
09/10

- Risk epilepsy

No evidence of brain damage & mortality in simple


fibril convulsion

Page | 250
FMAS, RUSL
UNDER THE GUIDANCE OF DR. Anurudhdha Padeniya
Done by S.Liyanarachchi &C.Lakmini 07/08, 2nd edition by Buddhika & Natha 08/09 3rd edition by Thejani
Bandara 09/10

Acute Flaccid Paralysis


Presentation:

Weakness of the lower limbs


History:
DD’s:
P/C→
Spinal cord- spinal cord injury
o Acute lower limb weakness
- Acute stage of Transverse Myelitis
o duration
Anterior horn cell disease-poliomyelitis
Hx/P/C→
-spinal muscular atropy type1
o Onset of weakness-sudden/gradual
Peripheral nerve-Guillain Barre syndrome

Neuromuscular junction o Distribution of it

Botulinum toxin o Progression over time

Myasthenia gravis o Neurological impairment at each stage in the chrono-


logical order-based on activities of the child(running
Snake venom ,playing, walking, sitting and etc)

Muscles- Ducchene muscular dystrophy o Associated neurological symptoms(this can be asked


by thinking of possible DD based on the site of the
- Myopathy
lesion)
- Dystropia myotonica

Page | 251
FMAS, RUSL
UNDER THE GUIDANCE OF DR. Anurudhdha Padeniya
Done by S.Liyanarachchi &C.Lakmini 07/08, 2nd edition by Buddhika & Natha 08/09 3rd edition by Thejani
Bandara 09/10
o DD exclusion GBS- preceding history of diarrhea or respiratory
tract illness, weakness usually begins in lower
Spinal cord injury-history of trauma to the spine limbs trunk upper limbs bulbar mus-
,associated backache cles, irritability, dysphagia and facial weakness,
aspiration symptoms, urinary incontinence and
retention ,respiratory muscle weakness as compli-
Acute stage of transversemyelitis-discom- cations
fort/overt pain in the neck/back depend on the
level of lesion, numbness, anaesthesia, sensory
impairment, flaccidity followed by spasticity, uri- Botulinum-possible ingestion of contaminated
nary retention early then incontinence later canned food, initial cranial nerve symptoms and
descending paralysis

Poliomyelitis-initial history of fever, URTI and


GI symptoms followed by a symptom free period MG-preceding history of drooping of the eyelids
of a few days, then recurrence of fever and myel- or weakness which is prominent towards after-
gia, neck stiffness and paralysis(asymmetrical) noon.
within 24-48 hrs.

Snake venom-recent history of snake bite


SMA type1-lack of antegravidity of hip flexion

Page | 252
FMAS, RUSL
UNDER THE GUIDANCE OF DR. Anurudhdha Padeniya
Done by S.Liyanarachchi &C.Lakmini 07/08, 2nd edition by Buddhika & Natha 08/09 3rd edition by Thejani
Bandara 09/10
Myositis-muscle pain, skin rashes o Polio vaccination

o Antirabis, influenza, conjugated meningo-


coccal vaccine
o What was done to the child up to hospital admis-
sion o Diet Hx→

o Complications of relevant disease

o Specific management done o Social Hx→

o Present status of child o General introduction to family

o Impact of the disease on child, parents

o P/M/Hx→ o Environment around home, accessibility


to the house, indoor environment(any ob-
o P/S/Hx→tonsillectomy, adenoidectomy, tooth stacles, hazards ), toilet type in home
extraction(predisposing factors for polio)
o How child travels to school, location of
classroom and accessibility, type of toilets
available at school
o Birth Hx→
o Support available

o Education of parents regarding disease


o Developmental hx→describe other milestones
o Psychological status of child and parents

Examination:
o Imm Hx→

Page | 253
FMAS, RUSL
UNDER THE GUIDANCE OF DR. Anurudhdha Padeniya
Done by S.Liyanarachchi &C.Lakmini 07/08, 2nd edition by Buddhika & Natha 08/09 3rd edition by Thejani
Bandara 09/10
General→
Sensory Sensory no Sometimes
o Anthropometry in centiles level sensory
involvement
o Ptosis
Other Bladder Bulbar Can have
heliotope rash & periorbital edema in important bowel weakness associated
dermatomyositis features incontinenc may be lower cranial
e nerve
Absent or involvement
o Drooling of saliva, facial asymmetry
reduced
reflexes Absent
o Neurocutaneous manifestations
reflexes
o Skin rash RS→
CNS→ o Respiratory rate
o Full CNS examination including cranial nerves o Features of respiratory distress

o Do single breath count test

Abd→
Spinal cord Polio GBS
lesion o Palpable bladder
Weakness B/L Asymmetri Symmetrical o scoliosis
weakness c ascending
paralysis CVS→ autonomic instability in GBS
Hypotonia
o PR-profound bradycardia

o BP-postural hypotension
Page | 254
FMAS, RUSL
UNDER THE GUIDANCE OF DR. Anurudhdha Padeniya
Done by S.Liyanarachchi &C.Lakmini 07/08, 2nd edition by Buddhika & Natha 08/09 3rd edition by Thejani
Bandara 09/10
o cardiomyopathy

Management: 1) Bed rest

Investigations: 2) Monitoring

GBS--Diagnosis is entirely clinical. Investigations done only


if any doubt of clinical diagnosis.(for confirmation)
PR, BP (autonomic instability)
o LP done after 10th day of appearance of symptoms.
RR - respirometer ,single breath count,cough effort
Cytoprotein dissociation.(CSF protein increases more
than twice of upper limit)glucose normal, cells<10 Neurological parameters -rate of progression of paralysis
WBC/mm.bacterial cultures negative. with grading of muscle power Features of bulbar
palsy
o NCS—slowing of conduction.
ICU if

rapid progressive paralysis


To exclude polio --
Bulbar palsy
o Stool samples-sample should be collect within 2 wks
of onset of the paralysis into provided special con- Respiratory involvement
tainer (within ice packs, cold container) send to MRI Autonomic cardiovascular involvement
within 72 hrs of collection.
Respiratory paralysis-consider ETT/tracheotomy/ventilation

If incontinence -catheter
To exclude transverse myelitis--MRI
Skin care, mouth care, eye care as appropriate
1.How do you manage a pt presented with acute flaccid
paralysis,(mainly GUILLAN BARE SYNDROME) Nutrition –NG or oral

Page | 255
FMAS, RUSL
UNDER THE GUIDANCE OF DR. Anurudhdha Padeniya
Done by S.Liyanarachchi &C.Lakmini 07/08, 2nd edition by Buddhika & Natha 08/09 3rd edition by Thejani
Bandara 09/10
Specific therapy-IV IG therapy 0.4 g/kg for 5 days, 6) Initiate eradication program –administer extra dose of OPV
start infusion slow initially to the children of same age and below living around 2KM ra-
dius of index case.
Plasmaparesis/immunosuppressive drugs as alternative.
7) If polio --isolation of patient, stool disinfection and careful
Physiotherapy -chest physiotherapy ,limb physiother-
disposal is important.
apy(passive, active)

Educate patient –spontaneous recovery 2-3 wks after in in-


verse direction to the paralysis.

Relapse, contine physiotherapy


3) method of stool collection—
Can transfer to a nearest hospital if respiration, feeding is
alright. Rehabilitation-occupational therapy Samples should be corrected within 2 weeks of the onset of
paralysis into provided containers.
Start schooling when child is able to walk. Do not wait until
100% normal Two samples should be collected 24-48 hours apart. Weight 8-
10g (size of 2 thumbnails)
2.Notification and other important investigations regarding
acute flaccid paralysis(AFP) Label correctly with date of onset of paralysis, date of collection
of stool. Last date of polio vaccination)
3) A child under 15 yrs of age should notify immediately to
MOH office, RE. The container packed in ice - sample should be at >8oC at the
time of arrival to MRI
4) MOH personally investigate the case of AFB .
Samples should be sent to MRI within 72hr of collection.
5) Collect stool samples from 3-5 contacts and sent to MRI
with in 72 hrs of collection. 4) What is Miller fisher syndrome?

Page | 256
FMAS, RUSL
UNDER THE GUIDANCE OF DR. Anurudhdha Padeniya
Done by S.Liyanarachchi &C.Lakmini 07/08, 2nd edition by Buddhika & Natha 08/09 3rd edition by Thejani
Bandara 09/10
A rare proximal variant of GBS that initially affects the ocular With which vaccine is paralytic poliomyelitis associated?
muscles and in which ataxia is prominent.
OPV, particularly in immunodeficient individuals.
Pathology
Management-
Demyelinating neuropathy – immune response Isolation with proper excretion of excreta
Notification
The immune response depends on certain bacterial factors – Stool samples-( Viral excretion in stools maximum during
lipooligosaccharides and host factors. first 2 weeks after initial symptoms)
Passive physiotherapy – start early
Antibodies to lipooligosaccharides can cross-react with specific
nerve gangliosides and can activate compliments.

Poliomyelitis
Active physiotherapy once fever is settled
(last reported case in SL-1993) Support paralytic limbs and splints
Nutrition
Enterovirus (picornavirus): Viral subtypes 1(Brunhilde), Monitor for evidence of respiratory failure/BP, pulse/Air
2(Lansing) and 3(Leon) way management
Transmission -Person to person via faeco-oral route
Infection starts in the GI tract →invades the nervous Follow-up at 60, 90 and 180 days (stool sampling)
system→ damages the anterior horn cell

Notification-
What are the types of vaccines? On clinical diagnosis every case of AFP notified to the Epi-
demiologist immediately by telephone, telegram, fax or
Vaccination Trivalent OPV (Sabin) email
IPV- killed polio virus (Salk) to RE and MOH
Live attenuated virus (enhanced potency vaccine)
OPV - high intestinal immunity,IPV - less intestinal Notification done by MO in attendance (HO, MO or Special-
immunity each vaccine containing all 3 strains of the virus. ist) – using special form (Form No.1 Form E13.1/ 95)

Page | 257
FMAS, RUSL
UNDER THE GUIDANCE OF DR. Anurudhdha Padeniya
Done by S.Liyanarachchi &C.Lakmini 07/08, 2nd edition by Buddhika & Natha 08/09 3rd edition by Thejani
Bandara 09/10
Notification form filled and sent via fax/ post to Epidemio-
logical Unit Transverse myelitis
•Transverse myelitis is a neurological condition caused by
Stool sampling- inflammation of the spinal cord resulting in axonal
Stool sampling2 samples 24-48 hours apart, within 2 weeks demyelination . It is often develops after viral infection or
of onset of paralysis occur during autoimmune disease
6-8 g (quantity of 2 thumbnails/ 2 tamarind seeds) •Varicella zoster , Herpes simples directly invade the spinal cord
Clean screw capped bottle; lid tightly closed to prevent leak- produce symptoms of transverse myelitis.
age and drying(a special container) •Transverse myelitis is sometimes associated with other
Clear correct labeling –introduction as in any sample,date of diseases, like systemic lupus erythematosis and sarcoidosis
onset of paralysis, date of collection of sample,date of dis- •Usually B/L involvement
patch of stools,last date of polio vaccination
Packed in a container with ice ,transport to MRI with in 72hr Symptoms and signs-
of collection develop rapidly over a period of hours.

The container packed in ice - sample should be at >8oC at the Symptoms include weakness and numbness of the limbs as well as
time of arrival to MRI motor, sensory, and sphincter deficits
Abnormal sensations: Patients report sensations of tingling,
numbness , coldness or burning below the affected area below the
MOH-Stool samples also from 3-5 immediate contacts of a spinal cord.
case Pain: Pain is sharp, shooting sensations begins suddenly in neck or
One sample each from contacts back and radiate to legs, arm or abdomen depending on the part of
spinal cord that is affected.
Out break response immunization Weakness of arms or legs: Weakness to severe paralysis of arms and
Limited outbreak response legs depending on the part of spinal cord that is affected.
Day after the Ix May be total paralysis and sensory loss below the level of the
House-to-house immunization of all children under age of lesion.
the AFP case within 2 km radius of his/ her residence (250-
300 people) upper cervical cord is involved- all four limbs may be
Only one dose of OPV involved and there is risk of respiratory paralysis
Contacts immunized after stool sampling (segments C3,4,5 to diaphragm).
Page | 258
FMAS, RUSL
UNDER THE GUIDANCE OF DR. Anurudhdha Padeniya
Done by S.Liyanarachchi &C.Lakmini 07/08, 2nd edition by Buddhika & Natha 08/09 3rd edition by Thejani
Bandara 09/10
Lesions of the lower cervical (C5–T1) region will MRI shows brain and spinal cord may show inflammation
cause a combination of upper and lower motor neu-
ron signs in the upper limbs, and exclusively upper Management-
motor neuron signs in the lower limbs. hospital admission
lesion of the thoracic spinal cord (T1–12) will pro- Intravenous steroid-IV dexamethasone
duce upper motor neuron signs in the lower limbs, Plasma exchange therapy
presenting as a spastic diplegia. Pain management : NSAID- aspirin, ibuprofen.
lesion of the lower part of the spinal cord (L1–S5) of- Physical therapy: To increased the muscle power .
ten produces a combination of upper and lower motor Occupational therapy
neuron signs in the lower limbs. Other: some patients well responds to intravenous cyclo-
Bowel and bladder dysfunction phosphamide
Muscle spasma
Headache If Transverse myelitis is treated early prognosis is excellent
Fever

complications of Transverse myelitis Botulism


Partial or total paralysis Botulism is an acute neurologic disorder that causes potentially life-
Spasticity of muscle threatening neuroparalysis due to a neurotoxin produced by
Osteoporosis Clostridium botulinum. The 3 main clinical presentations of
Depression. botulism are as follows:
Infant botulism
Diagnosis-
history ,physical examination Foodborne botulism
lab tests Wound botulism
radiological imaging . The disease does not usually affect consciousness or cause a fever.
Lumber Puncture: Shows high numbers of white blood cells or Pathophysiology
protein in the fluid , suggesting an infection or an inflammation. The mechanism of action involves toxin-mediated blockade of
neuromuscular transmission in cholinergic nerve fibers. This is
Blood test: Shows positive antibodies for neuromyelitisoptica.???? accomplished by either inhibiting acetylcholine release at the

Page | 259
FMAS, RUSL
UNDER THE GUIDANCE OF DR. Anurudhdha Padeniya
Done by S.Liyanarachchi &C.Lakmini 07/08, 2nd edition by Buddhika & Natha 08/09 3rd edition by Thejani
Bandara 09/10
presynaptic clefts of the myoneural junctions or by binding Additional neurologic manifestations include symmetrical
acetylcholine itself. descending paralysis or weakness of motor and autonomic
Signs and symptoms nerves
The muscle weakness of botulism characteristically starts in the Respiratory muscle weakness may be subtle or progressive,
muscles supplied by the cranial nerves. advancing rapidly to respiratory failure
The weakness then spreads to the arms (starting in the shoulders and
proceeding to the forearms) and legs (again from the thighs down to The autonomic nervous system is also involved in botulism, with
the feet) manifestations that include the following:
More than 90% of patients with botulism have 3-5 of the following
signs or symptoms:
Paralytic ileus advancing to severe constipation
Nausea
Gastric dilatation
Vomiting
Bladder distention advancing to urinary retention
Dysphagia
Orthostatic hypotension
Diplopia
Reduced salivation
Dilated/fixed pupils
Reduced lacrimation
Extremely dry mouth unrelieved by drinking fluids
Other neurologic findings include the following:
Generally, botulism progresses as follows:
Changes in deep tendon reflexes, which may be either intact
Preceding or following the onset of paralysis are nonspecific or diminished
findings such as nausea, vomiting, abdominal pain, malaise,
Incoordination due to muscle weakness
dizziness, dry mouth, dry throat, and, occasionally, sore
throat Absence of pathologic reflexes and normal findings on sen-
sory and gait examinations
Cranial nerve paralysis manifests as blurred vision, diplopia,
ptosis, extraocular muscle weakness or paresis, fixed/dilated Normal results on mental status examination
pupils, dysarthria, dysphagia, and/or suppressed gag reflex Diagnosis
. Toxin may be identified in the following:
Serum
Page | 260
FMAS, RUSL
UNDER THE GUIDANCE OF DR. Anurudhdha Padeniya
Done by S.Liyanarachchi &C.Lakmini 07/08, 2nd edition by Buddhika & Natha 08/09 3rd edition by Thejani
Bandara 09/10
Stool Clindamycin
Vomitus
Gastric aspirate
Suspected foods Antitoxins
Electromyography These agents are essential in the treatment of foodborne botulism
and wound botulism. Heptavalent antitoxin
Characteristic electromyographic findings in patients with botulism
include the following: Prevention
avoid giving honey to infants less than 12 months of age
Brief, low-voltage compound motor-units
proper food preparation. The toxin is destroyed by heating to
Small M-wave amplitudes
more than 85 °C (185 °F) for greater than 5 minutes.
Overly abundant action potentials
Treatment
Medical
Management ofrespiratory failure is the most important
Surgical Care
Wound botulism requires incision and thorough debridement of the
infected wound
Activity
Bedrest is initially required.
Increase activity as tolerated
When botulism develops following a wound infection, anti-
biotic therapy and meticulous debridement of the wound are
essential.
Antibiotics
Penicillin G
Chloramphenicol
Page | 261
FMAS, RUSL
UNDER THE GUIDANCE OF DR. Anurudhdha Padeniya
Done by S.Liyanarachchi &C.Lakmini 07/08, 2nd edition by Buddhika & Natha 08/09 3rd edition by Thejani
Bandara 09/10
Conditi Pressure
Leukocyt ProteinGlucose
(mg/dL)(mg/dL)Comments extended
on es (/μL) period
Normal 50- <4; 60- 20- >50 or 75% Organism may be
180 70% 45 blood glucose seen on Gram
mm lymphocy stain and Tuberculous
Usua 10-500; 100- <50 usual; Budding yeast
H2O tes, 30- recovered by meningitislly PMNs 500; decreases with may be seen;
40% culture eleva early but may time if organism may be
monocyte ted; lymphocy be treatment not recovered in
s, 1-3% may tes and high provided culture; India ink
neutrophi be monocyte er in preparation or
ls low s prese antigen may be
Acute bacterial
Usua 100- 100- Depressed Organisms may beca predomin nce positive in
meningitislly 60,000+; 500 compared with be seen; use ate later of cryptococcal
eleva usually a blood glucose; pretreatment may of CSF disease
ted few usually <40 render CSF sterile CSF bloc
thousand; in pneumococcal bloc k
PMNs and k in
predomin meningococcal adva
ate disease, but nced
antigen may be stage
detected s
Partially treated
Nor 1-10,000; >100 Depressed or Acid-fast
bacterial mal PMNs normal organisms may be Fungal Usua 25-500; 20- <50; decreases Enteroviruses may be recovered
meningitis or usual but seen on smear; lly PMNs 500 with time if from CSF by appropriate viral
eleva mononucl organism can be eleva early; treatment not cultures or PCR; HSV by PCR
ted ear cells recovered in ted mononucl provided
may culture or by ear cells
predomin PCR; PPD, chest predomin
ate if x-ray positive ate later
pretreated
for

Page | 262
FMAS, RUSL
UNDER THE GUIDANCE OF DR. Anurudhdha Padeniya
Done by S.Liyanarachchi &C.Lakmini 07/08, 2nd edition by Buddhika & Natha 08/09 3rd edition by Thejani
Bandara 09/10
Viral meningitis
Nor PMNs 20- Generally Profile may be
Or mal early; 100 normal; may completely
meningoencephalitis
or mononucl be depressed normal
sligh ear cells to 40 in some
tly predomin viral diseases
eleva ate later; (15-20% of
ted rarely mumps)
more than
1000 cells
except in
eastern
equine
Abscess Normal 0-100 20- Normal
(parameningeal or PMNs 200
infection) elevated unless
rupture
into
CSF

Page | 263
FMAS, RUSL
UNDER THE GUIDANCE OF DR. Anurudhdha Padeniya
Done By Chalani Rajasekara & Chathurika Nawarathne 07/08, 2nd Edition By Sanka 08/09, 3rd By Prasad &
Anusitha 09/10
o Hematological malignancies - Leukemia - low-grade fever,
oral ulcers, bleeding gums, bruises, recurrent chest Infec-
tions, bone pain
Thalassaemia o Chronic malaria- residence In endemic area, fever
o Blood losses
History
Hx/P/C-
Presentations: Mention what happened in chronological order as a story
Diagnostic
investigating for anemia features of progressive chronic Diagnosis
hemolytic anemia (pallor ,fatigue ^lethargy ,poor sucking) what was the Initial presentation
including mild jaundice anaemic features, FTT, recurrent Infection
FTT with recurrent Infection how was the diagnosis made
2-6 months of life At which age
Treatments
Management When was the blood transfusion started
For routine blood transfusion treatment intervals
Due to complication - disease itself, iron overload (HF fea- changing of frequency
tures- (Orthopnoea,PND, fatigue), chronic liver dx..., how many up to now
any reactions to blood transfusions / allergy
DD's: Iron chelating
Anemia when
o Haemolytlcanaemia - Hx of jaundice, dark urine, family hx methods & drugs used
of blood dieases if drugs changed reasons
o Exclude other causes for hemolytic anemia
Haemogloblnopathles Other medications- FA ,vit c
enzyme defects
membrane defects Complications of the dx
o Nutritional anemia-diet Severe anaemia +/- heart failure
Bone pain and fractures

Page | 264
FMAS, RUSL
UNDER THE GUIDANCE OF DR. Anurudhdha Padeniya
Done By Chalani Rajasekara & Chathurika Nawarathne 07/08, 2nd Edition By Sanka 08/09, 3rd By Prasad &
Anusitha 09/10
Complication of the recurrent blood transfusion and iron
overload
Infections ( recurrent) (hepatitis) P/M/Hx
Features of iron overload
Feeding difficulties - SOB, LOA Hx of liver dx, malaria, recurrent infections, if
Liver haemosiderosis - RHC mass& pain, hepatic en- complications having like DM, detailed Rx plan
cephalopathy, malaena, haematemesis Birth Hx
Endocrine - DM (poly urea, polydypsia, nocturea). Hy-
pothyroidism (constipation, lethargy, facial puffiness), Neonatal jaundice
Growth failure (delayed puberty)
Skin pigmentation Developmental and growth failure
Heart, failure features-cardiomyopathy,PND„ orthop-
noea, palpitation, syncopal attacks Immi Hx

Hib, pneumococcal, Meningococal, Hepatits B


SE of iron chealatlng drugs

Desferrioxamine – ototoxicity, retinal changes, bone dyspla-


sia, trunkal shortening, injection site problems ( abscess, Dite Hx
Yersinia infection, hypersensitivity) Tea following meals
Inquire on advices in mentioned in discussion (regarding of
Deferasirox ( Asundra) – cannot be given kids < 2 yrs, iron rich foods)
liver/renal toxicity, cytopenia, gastritis, blood stained diar-
rhea, nausea. Family Hx
Consanguinity- mention the degree(autosomal recessive),
Hx of repeated blood transfusions
Deferiprone – arthropathy, agranulocytosis, red urine, GI
Other siblings affected
disturbabces. Other Hemolytic anemia among 3 generation pedigree
Familial Dx of vulnerable organs in thal
Area of birth
If P/C is different – exclude D/D accordingly

Page | 265
FMAS, RUSL
UNDER THE GUIDANCE OF DR. Anurudhdha Padeniya
Done By Chalani Rajasekara & Chathurika Nawarathne 07/08, 2nd Edition By Sanka 08/09, 3rd By Prasad &
Anusitha 09/10
Social Hx Skeletal deformities- Dystal ulnar dysplastlc features,
Level of education of the parents Bony tenderness & fractures, osteoporosis, Deformities
Knowledge of the dx & it's inheritance of legs similar to rickets.
Economical states o Pallor, jaundice(haemolysis&ireffective erythropoiesis)
Family support o Features of hypothyroidism( due to Hypopituitarism, Thy-
Monthly cost for the treatment – travelling, insulin pump, sy- roid gland itself is not affected by iron overload)
ringes….. o Oedema ( Heart failure, liver dx, hypothyroidism, nutri-
Is there any social group or NGO helping them tional)
Impact on family members, &other siblings, family life o Peripheral stigmata of chronic liver dx-spider naevi, palmer
How the child cope up the condition, schooling ( attendance, erythaemae .t.c.
performances, relationships) o Skin pigmentation, gray- nail beds, elbows, knees.
Nearest hospital with transfusion facilities/tike treatment o Broad & thick hands (Thickend metacarpal & phalanges)
from o Secondary sexual characteristics present or not
Family planning
Social withdrawal

Examination
Abdomen
General
o Anthropometry -Height, Weight plot in centile charts (pitui- Inspection
tary failure), BMI distension, scars around umbilicus & injection site reactions,
o Tanners pubertal assessment (pituitary failure) - boy>14, scars of splenectomy /liver biopsy
girls>13. Palpation
Head to toe thalassemic features(extramedullaryhaenopoie- hepatomegaly due to iron overload & heart failure ( com-
sis) ment on upper & lower border margins, consistency, surface)
Splenomegaly
❖ Typical thalassaemlc facies - Scapocephaly, Prominent Percussion
maxilla, prominent parietal eminence, Frontal/skull free fluids
bossing, Flat nasal bridge, Malocclusion of teeth, (no
paranasal sinus except ethmoid) CVS-
Basically look for the heart failure features

Page | 266
FMAS, RUSL
UNDER THE GUIDANCE OF DR. Anurudhdha Padeniya
Done By Chalani Rajasekara & Chathurika Nawarathne 07/08, 2nd Edition By Sanka 08/09, 3rd By Prasad &
Anusitha 09/10
Pulse- regularity (arrhythmias) MCV
BP,JVP(elevated in RHF) MCHC
Apex beat-site, character(HF & cardiomyopathy) MCHC-N1
Para sternal heaves in RV hypertrophy RDW-NL
3rd heart sound-HF TIBC-NL
S.ferritne-NL
RS
Bilateral fine crepitatlons(HF) Blood picture
• hypochromic microcytic RBC, poikilocytosis, anisocytosis,
CNS normoblasts (Immature RBC), reticulocytes, target cells, RBC
Vision & Hearing (Desferrloxamine/Desferal SEs) fragment., Heinz bodies, tear drops cells
According to the presentation; If hepatic I encephalopathy -
consciousness... Hb- Electroporosis
Slow relaxing ankle reflexes which associated with hypothy- confirmatory test – HbF , HbA2 slightly , HbA1 absent( ln
roidism. major)
HPLC (High Performance Liquid Chromatography) - thalas-
Musculoskeletal semia major or trait
Joint, bone pain
Serum iron studies- serum ferritine, TIBC

INVESTIGATIONS Other supportive lx • Radtograph-


skull; hair on end appearance, widened dploid spaces, thickened
Diagnosis of thalassemia fonntal bone
Full blood count Hand-Widened, lattice pattern fracture
Exclude pancytopenia, reticulocytosis (Usually increased early &
then drops) Increased unconjugated serum bilirubin
Retie count-<8%
Hb Genetic/Chromosomal study - Definitive diagnosis
■ <10g/dl in thal.intermedia
■ <5g/dl in thal.major
Increased osmotic fragility; due to increased target celts
Red cell indices
Page | 267
FMAS, RUSL
UNDER THE GUIDANCE OF DR. Anurudhdha Padeniya
Done By Chalani Rajasekara & Chathurika Nawarathne 07/08, 2nd Edition By Sanka 08/09, 3rd By Prasad &
Anusitha 09/10
Management
Alkaline denaturation test- HbF is resistance to denaturation by
strong alkali Aims treatment are

Bone marrow biopsy Supress ineffective erythropoiesis

Ix to asses iron overload Prevent bone deformities

Allow normal growth and development


cardiac-CXR/ECG/ 2DEch
Mx outline

Llver-LFT/blopsy 1. Regular blood transfution

2. Iron chelating therapy


Endocrine-Blood sugar, GTT, TFT, Hormone level
3. Splenectomy

4. General mx
o once a month – Hb
o once In 03 months- FBC, liver enzymes, Albumlne, RFT, 5. Mx of complications
SGPT, SGOT
6. Genetic counselling
o once in 06 months - Ferrltine, FBS, TFT(T3 & T4)
o annually - Eyes, ENT, Echo 7. Prevention Education of pt & par-
ents

o Routine blood transfusions

To correct anaemia, supress extra medullary erythropoiesis

Page | 268
FMAS, RUSL
UNDER THE GUIDANCE OF DR. Anurudhdha Padeniya
Done By Chalani Rajasekara & Chathurika Nawarathne 07/08, 2nd Edition By Sanka 08/09, 3rd By Prasad &
Anusitha 09/10
To improve physical and mental wellbeing by maintaining Example-
Hb to sustain growth & development /3-4 Wkly
Needed blood volume to be transfused =1000ml
Indications Body weight =25kg
So total amount per day= 20*25=500ml
I. Severity of anemia (<6.5g/dl on 2 occations2 week apart) 1000/500=2days
II. Symptomatic
IV. splenectomised pts Hb drop by
III. FTT
1g/dl/day-splenectomised pts
IV. Borne deformities
1.5g/dl/wk-in non splenectomised pts
V. extra medullary erythropoiesis
Calculate annual blood requiment every 6 months
VI. progressively enlarging liler
Total blood volume given past 12 months
Facts about transfusions Average weight
If >200ml/yr hypersplenism or allo/auto
I. keep pre transfusion Hb level 9-10.5g/dl

II. target post transfusion Hb level 14-15g/dl ( should not ex- Assess spleen size each month & record in cm occasions
ceed 15g/dl) better to do direct coombs test 24-72hrs after transfusion

III. thalassaemia major pt Refer to consultant haematologist


Needed volume of blood = (14.5-Hb%) *4*BW(kg)
general mx
15-20ml/kg over 2-3 hrs per day
advices
(if HF 5ml/kg for 3-4 hrs)
must tell the importance of parents active participation
for the mx of child

Page | 269
FMAS, RUSL
UNDER THE GUIDANCE OF DR. Anurudhdha Padeniya
Done By Chalani Rajasekara & Chathurika Nawarathne 07/08, 2nd Edition By Sanka 08/09, 3rd By Prasad &
Anusitha 09/10
must describe the chronic nature of the dx,importance It is a genetic disorder
of regular blood Txn & chelating therapy,taking sup- Inherited in autosomal dominant manner & result in haemo-
plement like FA,vit C lytic anaemia according to the severity (quantitative loss of
educate regarding complications associated with Mx & Hb chains)
the measures should take to overcome from them Know about Hb synthesis ,minor verities of the thlassaemia
2.post transfusion reactions?
Dietary advice Febrile non haemolytic reaction(due to donor lymphocytes
take a balanced diet and decrease consumption of dark cytokines)
green vegetables, egg york, red meat,iron containing Allergic reactions ( due to plasma proteins)
foods Acute intra vascular hemolysis( blood group incompatibility)
drink tea after meals to reduce iron absorption Delayed intra vascular hemolysis(Ag –Ab reaction)
do not give row foods infec- Acute extra vascular hemolysis(IgG attached to RBC)
tion rate is high due to low im- Auto immune haemolytic anaemia
munity Transfusion related acute lung injury
avoid crowded places Transfusion associated circulatory failure
high calcium and folic acid supplements 3.preferred blood product?
normal schooling and exercise Group & Rh specipic leukocyte deplete / poor
psychological and social support New blood
inform school teachers regarding the condition 4.place for splenectomy?
Delayed till 6 year
Indications-
curative mx-Bone marrow transplantation and gene therapy I Hypersplenism(WBC<4000/Plt < 10000)
II Blood reqierment >250-275 ml /kg/year
Target Questions? III Possibility of rupture(large spleen)
1.what is thalassaemia? IV Massive splenomegaly with symptoms Eg- satiety
A spectrum of disease characterized by reduced or absent Risk of splenectomy : thrombocytosis, arterial ve-
production of more globin chains . nous thrombosis, infections(capsulated organism)

Page | 270
FMAS, RUSL
UNDER THE GUIDANCE OF DR. Anurudhdha Padeniya
Done By Chalani Rajasekara & Chathurika Nawarathne 07/08, 2nd Edition By Sanka 08/09, 3rd By Prasad &
Anusitha 09/10
Vaccination prior to splenectomy : 2 weeks before Not indicated in children < 2 years or with altered re-
surgery, vaccinate against capsulated organisms- nal functions
pneumococcal, meningococcal,Hib S,cr, liver enzymes , blood count checked
After splenectomy; Monthly full blood count
Need lifelong penicillin prophylaxis( if allergy- o Deferiprone
erythromycine), monitor PLT count Oral drug, if serum ferritin > 2500 give with desfer-
4.how do you assess iron overlord? roxamine
After 10-12 transfusions assess serum ferritin level, Omit the drug if pt complain arthropathy ( permanent
Start iron chelating if s.ferritin > 1000ng /ml or or temporary)
1000microgm/L ,
Or 10th transfusions onward 6.initial mx of a newly diagnosed infant
5.iron chelating agents? Confirm the thalassemia ruling out other conditions
o Desferrioxamine of anaemia
slow subcutaneous infution over night (8-12) 5-6 day Send blood for red cell phenotyping before starting
per week(EOD) transfusions
20-40mg/kg/day <12 yrs 50mg/kg/day-adults Pre transfusion base line Ix- s. ferritin ,liver enzymes
Can give iv with blood G6PD screening , USS abdomen to determine the
15-20 vials monthly , 1 vial= 500mg size of the liver & spleen
Excessive doses ; cateract, retinal damage, nerve Look for any indication for the blood transfusion
deafness Prior to blood transfusion Hep B vaccine should be
Yersiniya enterocolities given
o Deferasirox
1st line drug , oral monotherpy 8.complications of thalassemia?
Dose- 20-40mg/kg/day Transfusional reactions
1 tab- dissolve in 100-200mlof water ,take with Recurrent infections
empty stomach ,30minutes before meal Splenectomy
Can not give with desferroxamine
Page | 271
FMAS, RUSL
UNDER THE GUIDANCE OF DR. Anurudhdha Padeniya
Done By Chalani Rajasekara & Chathurika Nawarathne 07/08, 2nd Edition By Sanka 08/09, 3rd By Prasad &
Anusitha 09/10
Hypersplenism S .calcium & S .phosphate; for all patient after
Immunosuppression due to chronic dx ,due to drugs 12yrs 6 monthly
Due to blood transfusions-Hep-C,HIV,CMV,Ma- Fasting blood sugar ;annually for every patient af-
laria,Pavo B 19 ter 12 yrs of age ,if high endocrinologist referral
Iron overlord- hypopitiuarism, hypothyroidism,hypo- Audiometry & ophthalmic assessment; yearly or
parathyrodism,DM when clinically detected
Hypogonadotrophichypogonadism( testicular or Regular follow up at clinic-monitor growth ,look for
ovarian failure) complications

9.how do you investigate for complications? 9.Causes of death?


Cardiac-2D echocardiogram perform yearly start- Around 20 yrs specially due to HF
ing from 10 years ( <12yrs-yearly / > 12yrs- every Cardiac failure and anaemia
year) Infections
If heart failure-need more frequent transfusions Hepatoma
maintain pretransfusion Hb level 11-12g/dl Complications of bone marrow transplantation
Liver- liver enzymes do every 3 monthly, hepatitis
screening 10.Prevention
Renal- S-Cr every 3 monthly Genetic counselling , Discourage consanguine mar-
Bone- age >10 years DEXA scan perform yearly, riage
if diagnosed osteoporosis , treat with Bisphospho- Antenatal diagnosis by chorionic villi biopsy
nate Screening –cascade ( family screening/ siblings/ pa-
Age > 12years start Ca supplements & vitamin D rental)
Carrier detection ( MCH< 21,HPLC,MCV<70 – blood
Endocrine; growth assessment by annually – tanner picture)
staging for girls > 13yrs and boys > 14yrs Discourage marriage between carriers
If no signs of puberty; refer to endocrinologist
TSH level; annually after 12 yrs

Page | 272
FMAS, RUSL
UNDER THE GUIDANCE OF DR. Anurudhdha Padeniya
Done By Chalani Rajasekara & Chathurika Nawarathne 07/08, 2nd Edition By Sanka 08/09, 3rd By Prasad &
Anusitha 09/10

What are the questions that parent ask…..


1. How this disease spread…
This is autosomal recessive disease. An affected individual is
homozygous for the abnormal gene, having inherited an abnormal
allele from each parent, both of whom are unaffected heterozygous
carriers. For two carrier parents, the risk of each child, male or female,
being affected is 1 in 4 (25%). All offspring of affected individuals
will be carriers.

2. Is this can completely cure..?


Yes.

3. How is that….?
Bone marrow transplantation is the cure. It should be done
before more transfusion occur. Usually within 1-2 years. (
before extra medullary haemopoiesis occur ) this has higher (
≈ 99 %) success rate.

4. Is it available in Sri Lanka


Yes. Colombo national hospital.

Page | 273
FMAS, RUSL
UNDER THE GUIDANCE OF DR. Anurudhdha Padeniya
Done by R.Priyadharshana, M.Rathnayaka, S. Premathilaka 07/08,2nd edition by Shashika 08/09
3rd edition by Gishan Budhdhika 09/10

Haemolytic anaemia
Sex→G6PD deficiency is common among males
Presentations:

Symptoms of anemia (Pallor, Fatigue, SOB)


P/C→
jaundice
o Symptoms of anemia (Pallor, Fatigue, SOB)
DD’s:
o Jaundice
Haemolytic anaemia
From intrauterine life – Hemolytic disease of
Anaemia due to ineffective erythropoiesis Eg:-Fe/Folic defi- the newborn (Rh incompatibility, ABO in-
ciency anaemia, Chronic inflammation(JIA),CRF compatibility, other minor blood group in-
compatibilities)
Red cell aplasia Eg:-Diamond-Blackfan anaemia, Parvovirus
B19 infection At birth or First few days of life – Hereditary
Anaemia due to blood loss Eg:-Meckel diverticulum,VWD spherocytosis , G6PD

Liver failure After 3 to 6 months – Thalassemia

Pancytopenia Middle ages – Acquired warm autoimmune


hemolytic anemia
History:
Aplastic crisis following infection – HS
Age →Congenital HA – common in neonates and pediatric
age group Drugs,chemicals,foods or infections –G6PD,
Acquired & drug induced HA - in any age group (more in PK deficiency, Drug induced HA,autoim-
mune/ non autoimmune HA
children)

Page | 274
FMAS, RUSL
UNDER THE GUIDANCE OF DR. Anurudhdha Padeniya
Done by R.Priyadharshana, M.Rathnayaka, S. Premathilaka 07/08,2nd edition by Shashika 08/09
3rd edition by Gishan Budhdhika 09/10
o Reactions following blood transfusion – due to in-
compatibility, symptoms are fever, chills, low BP,
o Dark urine –In most HA, due to increased bilirubin shock
&urobilinogen production, towards night and morn-
ing, increase with Fe Rx &Sx Hx/P/C→

o Failure to thrive – Mainly Thalassemia, others can Analyze the onset duration & associated com-
also contribute plications relevant to the presenting complaint

o Recurrent infections – Thalassemia, SCD Exclude other DDs

o Abdominal Pain (Upper abdomen)

Gallstones – High levels of bilirubin P/M/Hx→


and cholesterol from the breakdown of
RBC form gallstones (Painful) Hospital admissions

Enlarged spleen – due to hemolysis , Blood transfusions


can be painful
Past Hx of complications –Aplastic crisis in
HS, Vasooclusive crisis in SCD

o Vasoocclusive crisis – SCD P/S/Hx→

o Leg ulcers & pain – SCD ,sickle shaped cells obstruct o Colecystectomy due to symptomatic gall stones
the blood flow which causes leg sores & pain
o Splenectomy as a curative surgery
o Arrhythmia & Cardiomegaly – Due iron overload
o Also import in PNH
during Rx, Complication of HA
Drug Hx→
Page | 275
FMAS, RUSL
UNDER THE GUIDANCE OF DR. Anurudhdha Padeniya
Done by R.Priyadharshana, M.Rathnayaka, S. Premathilaka 07/08,2nd edition by Shashika 08/09
3rd edition by Gishan Budhdhika 09/10
o Antimalarials,Antibiotics(Sulphanamides,Quin- Parent’s knowledge on the Dx,Complica-
olones,Nitrofurantoin),Aspirin in G6PD tions,Tx and how to act in an emergency

o Oral folic acid-due to increased requirement Monthly income

o Daily oral Penicillin prophylaxis if splenectomy done Closest hospital and transport facilities

Birth Hx→ If there are recurrent hospital admissions &


etc. how that affect the family members
Usual birth Hx, Any developmental delay due
to chronic hemolysis

ImmHx→ Examination:

Usual immune Hx and if splenectomy done General→


,about Meningococcal,Pneumococcal and HiB
vaccinations o Anthropometry

Diet Hx→ o Pallor

Usual diet Hx o Icterus

Family Hx→ o Features of nutritional deficiency – koilanoichia,


glossitis, angular stomatitis
o SCD, Thal, G6PD, PK
o Peripheral Stigmata of liver failure
Social Hx→
o Leg ulcers
Family background and educational level of
the parents o Purpura, petechiae, - Pancytopenia

Page | 276
FMAS, RUSL
UNDER THE GUIDANCE OF DR. Anurudhdha Padeniya
Done by R.Priyadharshana, M.Rathnayaka, S. Premathilaka 07/08,2nd edition by Shashika 08/09
3rd edition by Gishan Budhdhika 09/10
o Bone deformity – Thalassemia o Hb↓(due to red cell destruction)

o Ankle oedema o Evidence of infection

CVS→ o Pancytopenia

o Irregular heartbeat, Retic count( due to compensatory increase of erythropoie-


sis)
o tachycardia
o To confirm increased RBC production
o Arrhythmias
o ↓ in Fe deficiency
o Cardiomegaly
o Increased in HA
o Systolic flow murmur
o Very low in Red cell aplasia
RS→
Serum bilirubin- (unconjugated fraction increased)
o Dyspnea, bronchial breathing, crackles (due to LRTI)
Serum haptoglobin↓(Hb binding & cleared more rapidly as
Abd→
conjugates)
o Tenderness Blood picture
o Hepatosplenomegaly o Spherocytosis

o Eliptocytosis

Investigations: o Sickle cells

FBC with red cell indices→ o Hypochromic microcytic picture- Thal,Fedef.

Page | 277
FMAS, RUSL
UNDER THE GUIDANCE OF DR. Anurudhdha Padeniya
Done by R.Priyadharshana, M.Rathnayaka, S. Premathilaka 07/08,2nd edition by Shashika 08/09
3rd edition by Gishan Budhdhika 09/10
Liver Function Tests – to assess the Liver function and in-
volvement

Management:
Special Tests
Membrane defects
Coombs – For Autoimmune hemolyticanemia

Osmotic Fragility test – Hereditary Spherocytosis Hereditary Spherocytosis


Diagnosed with spherocytes in blood film, nega-
Sickle Solubility test – Sickle Cell anemia tivecoombs test, increased osmotic fragility and abnor-
mal cytoskeletal protein analysis.
Hb Electrophoresis – Sickle Cell anaemia
Mild cases are treated with folic acid supplements to
USS abdomen – Hepatomegaly, Splenomegaly cope with increased RBC production(Hb>10g/dl)
Severe and poor growth with age<2rs blood transfu-
Skull X-ray – Bone marrow expansion on skull bones (hair sion
on end-due to bone marrow expansion) If increased serum bilirubin and Gallstones occur, Sple-
nectomy is considered with Vaccination for Pneumo-
G6PD Assay – G6PD Deficiency (reduced ability of RBC to
cocci, Meningococci and HaemophilusInfluenza bFol-
reduce the dye) lowed up by lifelong penicillin prophylaxis
Direct antibody test – Positive with IgG and Complements Hereditaryelliptocytosis
for Acquired warm autoimmune hemolyticanemia. Positive Diagnosed with ellipoctes in blood film,RBC mildly
with only complements for Acquired cold autoimmune hae- heat sensitive,abnormal cytoskeletal protein anaylsis.in
molyticanaemias mild cases no treatment required and chronic cases
transfusion as HS
Antenatal Screening – HDN Paroxysmal nocturnal hemoglobinuria
Flow cytometer assays, Bone Marrow Biopsy – PNHbU Lab findings are low WBCs hams test,bonemar-
rowaspiration and biopsy.

Page | 278
FMAS, RUSL
UNDER THE GUIDANCE OF DR. Anurudhdha Padeniya
Done by R.Priyadharshana, M.Rathnayaka, S. Premathilaka 07/08,2nd edition by Shashika 08/09
3rd edition by Gishan Budhdhika 09/10
Management in mild cases no treatment required and Spherocytes in blood picture
some cases folic acid 1mg
Positive direct coombs test
Chronic hemolysis and cytopathies-prednisolone 60mg
initially and then taper and maintain 15-40mg
Warm antibody type

Enzyme deficiencies Coombs test thermal amplitude 35-40 ‘C


G6PD Deficiency Treatment
Stop the causative drugs or avoid oxidant stress to
RBCs Prednisolone 2mg/Kg/24hr
Treat underlying infections with appropriate antibiotics IV immunoglobulin
Blood transfusion is needed if anemic.
Danazol
Splenectomy is not proven to be beneficial
Pyruvate kinase deficiency Splenectomy,immune suppressive

Pyruvate kinase assay Folic acid 1mg/24 if chronic

Severe anemia with poor growth and age below 2 years Cold antibody type
–blood transfusion
Agglutination or rouleux on blood picture,positive di-
Splenectomy>6yrs earlier if indicated rect coombs test. Test for underline Dx.serology for
IMN. serology for mycoplasma pneumonia
Folic acid 1mg
Treatment

If Hb>10g/dl and retic count <10% no treatment re-


Extracellular defects
quired
Autoimmune hemolytic anemia
Severe anemia -transfusion

Page | 279
FMAS, RUSL
UNDER THE GUIDANCE OF DR. Anurudhdha Padeniya
Done by R.Priyadharshana, M.Rathnayaka, S. Premathilaka 07/08,2nd edition by Shashika 08/09
3rd edition by Gishan Budhdhika 09/10
If severe-immunosuppression and plasma paresis Membrane lipids: abetalipoproteinemia

Splenectomy not useful


Enzyme deficiencies
Folic acid 1mg/24hr if chronic Glycolytic enzymes: pyruvate kinase, hexokinase
Enzymes of hexose monophosphate shunt: glucose-6-phosphate
dehydrogenase, glutathione synthetase
Fragmentation hemolysis Disorders of hemoglobin synthesis
DIC, TTP, HUS Deficient globin synthesis: thalassemia syndromes
Structurally abnormal globin synthesis (hemoglobinopathies):
Fragments of blood picture sickle cell anemia, unstable hemoglobin
42) How do you investigate a child with hemolyticanemia?
Treatments
43) How to you classify hemolyticanemia?
Treat underling cause

Transfusion Intrinsic (intracorpuscular) abnormalities


Hereditary
Membrane abnormalities
Membrane skeleton proteins: spherocytosis, elliptocytosis
Target Questions Membrane lipids: abetalipoproteinemia

40) How do you investigate a child with hemolyticanemia?


Enzyme deficiencies
41) How to you classify hemolyticanemia? Glycolytic enzymes: pyruvate kinase, hexokinase
Enzymes of hexose monophosphate shunt: glucose-6-phosphate
Intrinsic (intracorpuscular) abnormalities dehydrogenase, glutathione synthetase
Hereditary
Disorders of hemoglobin synthesis
Membrane abnormalities
Deficient globin synthesis: thalassemia syndromes
Membrane skeleton proteins: spherocytosis, elliptocytosis

Page | 280
FMAS, RUSL
UNDER THE GUIDANCE OF DR. Anurudhdha Padeniya
Done by R.Priyadharshana, M.Rathnayaka, S. Premathilaka 07/08,2nd edition by Shashika 08/09
3rd edition by Gishan Budhdhika 09/10
Structurally abnormal globin synthesis (hemoglobinopathies): Others – Phenacetin, Vitamin K analogs, Methylene blue Probenecid,
sickle cell anemia, unstable hemoglobin Acetylsalicylic acid,Phenazopyridine

Acquired CHEMICALS– Phenylhydrazine, BenzeneNaphthalene


Membrane defect: paroxysmal nocturnal hemoglobinuria
ILLNESS- Diabetic acidosis, Hepatitis, Sepsis

Extrinsic (extracorpuscular) abnormalities 45) How do you recognize hereditary spherocytosis from blood picture?
Antibody mediated
Isohemagglutinins: transfusion reactions, erythroblastosisfetalis anisocytosis and several dark-appearing spherocytes with no central
(Rh disease of the newborn) pallor.
Howell-Jolly bodies (small dark nuclear remnants) present in the red
Autoantibodies: idiopathic (primary), drug-associated, systemic
cells.
lupus erythematosus
Marked reticulocytes
Mechanical trauma to red cells
Microangiopathic hemolytic anemias: thrombotic thrombocytope-
nic purpura, disseminated intravascular coagulation
Infections: malaria 46) What are the complications in Hereditary spherocytosis

Aplastic crisis (precipitated by parvo B19 infection)

Hemolytic crisis

44) What are the precipitating factors of G6PD?It can be precipitated by Cholycystisis
exposed to an environmental factor (most commonly infectious agents
or drugs) that results in increased oxidant stress. Cholilitheasis (pigmented)

Folatedeficiency (due to increase bone marrow requirement)

Antibacterial – Sulfonamides, Trimethoprim-sulfamethoxazole, Nalidixic


acid, Chloramphenicol, Nitrofurantoin
47) How you manage anaphylaxis shock.
Antimalarial- Primaquine, Pamaquine Chloroquine, Quinacrine

Page | 281
FMAS, RUSL
UNDER THE GUIDANCE OF DR. Anurudhdha Padeniya
Done by R.Priyadharshana, M.Rathnayaka, S. Premathilaka 07/08,2nd edition by Shashika 08/09
3rd edition by Gishan Budhdhika 09/10
Assess A, B, C. Intubate if necessary.

Give high flow oxygen

Give adrenalin 0.1ml/kg IM 1:10000 (10micg/kg). Repeat af-


ter 5min if no improvement.

If no response IV adrenalin 10micg/kg

Gain IV access

Give chlorpheneramin IV over 1-2 minutes. (in neonates not


use, 1mnt-1yr=0.25m g/kg, 1-5yrs=2.5-5mg, 6-12yrs=5-10mg,
>12yrs=10-20mg)

Hydrocortisone 4mg/kg IV

Start N/S or Hartman20ml/kg IV, colloid if in shock

Page | 282
FMAS, RUSL
UNDER THE GUIDANCE OF DR. Anurudhdha Padeniya
Done by sunethra kumari 07/08,2nd Edition by vidushanka 08/09,3rd Edition Manoja 09/10

Haemophilia Preceding trauma/ spontaneous


Associated discolouration of skin
Joint deformity, unable to move the joint
Common presentations: joint pain and swelling
Bleeding manifestations

DD – bleeding manifestations
1) vWD, Vit K def. , liver disease – coagulation disorders
2) HUS, DIC
3) BM failure
4) Thrombocytopenia – viral infections/Rx/ AI/ sequestration
5) Vascular – HSP, scurvy, Ehler-Danlos What was done now – at home hospital
DD – joint pain and swelling If haemarthrosis – more pain, more quickly
1) Rheumatic fever than previous – joint arthropathy dev.
2) JIA
3) Septic arthritis If 1st presentation exclude DD
4) HSP/SLE Condition Points in the Hx
Rheumatic fever Fever. fleeting and flitting large joint
involvement, subcutaneous nodules, carditis
(new onset murmur) , established valvular
disease, chorea
JIA PUO, large and small joint involvement, Rash
Hx – already diagnosed/ 1st presentation with the height of fever
Nearly always ♂ Painful red eye – chronic anterior uveitis
Septic arthritis Usually monoarthritis, child does not move
HPC – onset, duration and progression of symptoms the joint, high fever
HSP Preceding URTI, palpable red rash over
extensors and buttocks

Hx of bleeding manifestations in the past

Page | 283
FMAS, RUSL
UNDER THE GUIDANCE OF DR. Anurudhdha Padeniya
Done by sunethra kumari 07/08,2nd Edition by vidushanka 08/09,3rd Edition Manoja 09/10
Infants – bruises with knocks on the cot/ pick up, with crawling Immunization – Hx of haematoma and bleeding following IM
painful joint swelling vaccination
Bleeding – esp. ankle, circumcision Hep B vaccination given
Haematoma following vaccination Vaccines given in the SC form

Children – severe bleeding following tooth extraction FHx – consanguinity


Haemarthrosis – esp. knee, elbow deformities, disuse Maternal uncles affected, any other siblings affected
atrophy Mother – menorrhagia (females can have bleeding due
Early recognition – warm, tingling haemophilia – due to lyonization – random inactivation of the
sensation normal x chromosome, homozygous form – rare, die before birth,
Gum bleeding, epistaxis, frequent bruises, oozing from turners Xn, xy ♀)
scars
Abdominal pain SHx – income and education level of parents
Complications – recurrent bleeds into same joint – target Family support
joint chronic arthropathy Effect on child’s day-to-day activities and school
ICH – focal fits, headache performance
Haematuria, haematemesis,malaena Overprotective parents
Bleeding into iliopsoas – hypovolemia, Effect on other family members
vague groin pain, cannot extend hip Knowledge about the disease, what to do during a bleed
Nearest hospital with Tx facilities, transport available, time
sed pt. – past hospitalizations, F VIII/IX Tx, child’s blood group, taken
response to F VIII/IX – inhibitors Awareness of the school teacher
Complications of bld Tx – reactions, anaphylaxis, Blood borne
infec. – HBV, HCV, HIV, malaria Examination
Regular screening for infections General – is the child in pain
Birth Hx – mode of delivary Fever – temperature chart (septic arthritis)
Cephalhaematomas, ICH – seizures, umbilical bleeding Pallor
(Factor VIII does not cross placenta, however during Nose - epistaxis
birth process VIII - since APP Dental carice
Factor IX physiologically low at birth) LN - generalized
Skin – bruises
Child abuse – finger pulp contusions

Page | 284
FMAS, RUSL
UNDER THE GUIDANCE OF DR. Anurudhdha Padeniya
Done by sunethra kumari 07/08,2nd Edition by vidushanka 08/09,3rd Edition Manoja 09/10
MS – haemarthrosis – painful/ tense swelling ITP N/L N/L N/L
Limitation of movement Liver N/L N/L
Deformities – disuse atrophy disease
Septic arthritis – very warm, red and tender, no movement DIC
Other joints and muscles – esp. iliopsoas – hip kept flexed and APTT
internally rotated

CVS – PR, pulse volume, BP – haemodynamic stability


ABD - I – abdominal distension, bruises
P – abdominal tenderness
Hepatosplenomegaly – JIA, leukaemia, lymphoma

CNS – GCS
Pupils
Fundi – retinal haemorrhage
UL, LL – weakness

Ix
• FBC – plt. count, Hb and other cell lines
• Blood picture – platelet morphology, presence of abnormal
cells (leukemia, lymphoma)
• Clotting profile

Plt. BT Clottin PT/INR(1 APT


cou (7min) g time 2s) T
nt (30-
40s)
Other Ix
Haemophi N/L N/L N/L • Factor assay
lia Haemophilia A – factor VIII
vWF N/L (plt N/L Haemophilia B – factor IX
dysfun (christmas disease)
c) Severity – severe <1%
Page | 285
FMAS, RUSL
UNDER THE GUIDANCE OF DR. Anurudhdha Padeniya
Done by sunethra kumari 07/08,2nd Edition by vidushanka 08/09,3rd Edition Manoja 09/10
Moderate – 1-5% • Gentle compression crape bandage/elastic Stocking around J.
Mild >5% –
• Inhibitor screen*
Via factor correction 1:1 Limit bleeding and Support the joint

Corrected -no (-) not corrected


(-) present
• Other
• DNA studies
• Prenatal sis – not in SL
Chorionic villous sampling
(8-10wks)
* 1:1 mixing of pt.s plasma with normal plasma correction of APTT
.In 25-35% due to presence of factor specific Abs/

Cold compressions - leukocyte-endothelial interaction

• Rehabilitation after pain and swelling settles

inhibitors APTT is not corrected. They are directed against the Start physiotherapy – static exercises
active clotting sites
Use walking aids
Quantitative Bethseda assay – find the Ab titre
Daily exercises to improve muscle strength
Acute management
Supportive care & maintain joint motion
• Pain relief – avoid aspirin and NSAIDs
Specific management
• Intermittent cold compressions
• Factor replacement therapy
• Immobilize the limb in its functional Position – splint/sling –
UL
Page | 286
FMAS, RUSL
UNDER THE GUIDANCE OF DR. Anurudhdha Padeniya
Done by sunethra kumari 07/08,2nd Edition by vidushanka 08/09,3rd Edition Manoja 09/10
Based on the type of bleed and the haemostatic factor Site of Haemostatic F Comment
correction required level

GIT Initially 100% Endoscopy highly


30% until recommended to find lesion.
healing Antifibrinolytic therapy useful
Dose to be infused = wt. (kg) x factor increment needed (%) GUT Initially 100% Look for stones or UTI.
30% until Lesions usually not found.
2 healing occurs Prednisolone may be useful
CNS Initially 100% Anticonvulsants.
In haemophilia B divide by 1 – which is the percentage rise 50-100% for 10- LP will require factor
in plasma with one unit of F IX 14 days coverage.
Trauma Initially 100% Pre & post op plan essential.
Site of Haemostatic F Comment or surgery 50% until Evaluate for inhibitors prior to
bleed level wound healing surgery
Joint 30-50% Rest, immobilization, begins. Then
minimum rehabilitation.
Target joint will need several 30% until healed
doses
• Bleeds into joint, muscle and from mouth require 50% fac-
Muscle 40-50% Calf / forearm bleeds
limb threatening large blood tor correction
minimum
loss in femoral /
• Life threatening bleeds require 100% correction
retroperitoneal bleeds
Oral Initially 50% Antifibrinolytic therapy is 1IU = amount of factor in 1ml of normal plasma
mucosa antifibrinolytic critical
therapy Eg: haemarthrosis – 50% x wt = 25IU/kg

Epistaxis Initially 80- Local measures ; pressure, 2


100% 30% until packing/ cautery useful for
Haemostatic level for factor VIII – 30-40%
healing severe/ recurrent bleeds

Page | 287
FMAS, RUSL
UNDER THE GUIDANCE OF DR. Anurudhdha Padeniya
Done by sunethra kumari 07/08,2nd Edition by vidushanka 08/09,3rd Edition Manoja 09/10
Haemostatic level for factor IX - >25-30% DDAVP therapy

Types of factor concentrates

1) Recombinant FVIII/IX – expensive, not in gov. sector; Adv. –


not plasma derived – no risk of infec.
2) High purity FVIII/IX – plasma derived but virally inactivated via
detergents/heat etc., available in SL
3) Plasma derived FVIII/IX – available in gov sector risk of TYPE OF
HBV,HCV, HIV HEMORRHAGE HEMOPHILIA A HEMOPHILIA B
4) Cryoprecipitate – used in SL Hemarthrosis D1 -40 IU/kg factor VIII D1- 60–80 IU/kg
5) FFP D2-5 20 IU/kg until D2-4 40 IU/kg
F VIII 1 bottle – 250IU joint function is N/L/
F IX 1 bottle – 600IU baseline. same
Cryoprecipitate – F VIII 150IU/pack Consider additional Rx
Used for F VIII correction EOD for 7–10 days.
FFP – used for F IX correction Consider prophylaxis.
F IX – provides around 80IU/unit Muscle / 20 IU/kg factor VIII 40 IU/kg F IX
(Considering only 40% factor recovery with Tx) significant SC concentrate; may need concentrate[‡]
One pack – 250ml hematoma EOD treatment until May need treatment
resolved. every 2–3 days until
resolved.
.. Mouth, deciduous 20 IU/kg factor VIII 40 IU/kg factor IX
tooth, or tooth concentrate – one dose concentrate[‡];
extraction antifibrinolytic therapy; antifibrinolytic
remove loose deciduous therapy[§];
tooth remove loose
If transfusing… cont. 5d after deciduous tooth.
• FFP – should be ABO compatible. Rh not needed Epistaxis Apply pressure for 15– Same
Contains no WBC no risk of CMV Tx & GVHD 20 min; pack with
• Cryoprecipitate – multiple donor risk of infec. petrolatum gauze fails
Now single donor recommended – usually the father after

Page | 288
FMAS, RUSL
UNDER THE GUIDANCE OF DR. Anurudhdha Padeniya
Done by sunethra kumari 07/08,2nd Edition by vidushanka 08/09,3rd Edition Manoja 09/10
antifibrinolytic therapy 30 IU/kg factor IX give prednisone (unless prednisone (unless
fails concentrate[‡] HIV-infected). HIV-infected).
20 IU/kg factor VIII Prophylaxis 20–40 IU/kg F VIII 30–50 IU/kg factor IX
concentrate [*][*] concentrate EOD to concentrate[‡] every
Major surgery, 50–75 IU/kg factor VIII 120 IU/kg factor IX achieve a trough level of 2–3 days to achieve a
life-threatening concentrate initiate concentrate[‡], 50–60 ≥ 1%. trough level of ≥ 1%.
hemorrhage continuous infusion of IU/kg every 12–24 hr [‡] dose given for recombinant FIX. plasma derived FIX – 70% of above dose

2–4 IU/kg/hr to maintain to maintain factor IX Other therapy


factor VIII > 100 IU/dL at > 40 IU/dL for 5–7
for 24h give 2–3 days > 30 IU/dL for 1) anti- fibrinolytic Rx
IU/kg/hr continuously 7 days. - used in mucous membrane bleeds (which occur due to fibrinolysis
for 5–7 days to maintain preventing clot stabilization)
the level at > 50 IU/dL Eg : Trenexamic acid – best absorbed from the buccal mucosa
an additional 5–7 days at Dose – 20mg/kg – tablet dissolved in 10ml
a level of > 30 IU/dL. of water solution kept in mouth as
Iliopsoas 50 IU/kg factor VIII 120 IU/kg factor IX Long as possible swallowed
hemorrhage concentrate, then 25 concentrate[‡]; then
IU/kg every 12 hr until 50–60 IU/kg every 2) Prednisolone – in macroscopic upper tract haematuria
asymptomatic, then 20 12–24 hr to maintain Dose – 0.5mg/kg x 5d 0.25mg/kg x 5d
IU/kg every other day factor IX at > 40
for a total of 10–14 IU/dL until 3) Desmopressin (DDAVP)
days.[∥] asymptomatic, then I - mild – moderate haemophilia A, type I vWD
Confirmed – USS/CT 40–50 IU every other Action – causes the release of vWF from endogenous stores in
day for a total of 10– endothelial cells vWF and F VIII
14 days.[∥][¶] Dose – IV 0.2 -0.3g/kg in 50-100ml of N/S
Hematuria Bed rest; Bed rest; 1½ × Intra-nasal spray – 1puff - <50kg
hydrate well - 1½ × maintenance fluids; if 2 puffs >50kg
maintenance fluids; not controlled in 1–2 S/E – flushing, fluid retention, Na+ seizures (infants)
if not controlled in 1–2 days, 40 IU/kg factor Adv. – cheap, no risk of infection
days, 20 IU/kg factor IX concentrate[‡]; if
VIII concentrate; fails not controlled, give 4) other – Ca alginate (seaweed derivative) I - epistaxis

Page | 289
FMAS, RUSL
UNDER THE GUIDANCE OF DR. Anurudhdha Padeniya
Done by sunethra kumari 07/08,2nd Edition by vidushanka 08/09,3rd Edition Manoja 09/10
Danazole – short term, following CNS bleeds, target joint
haemarthrosis, vWD - ♀

Long term Mx • Avoid overprotection


• Parent education • Early recognition of bleeds. Esp, life threatening
• Nature of the disease – life long risk of bleeding, no bleeds
cure GIT – abdominal pain, melaena, haematemesis
• Genetic basis – x linked inheritance, maternal car- CNS – early morning headache, altered
riage, can be a spontaneous mutation consciousness
Risk to future pregnancies – 50% of
boys will be affected • Routine immunization
• Preventive measures Not via IM route
<3yrs 1) child should be supervised in bed/ allow to All given subcutaneously via 26G needle apply firm
sleep on a mattress on the floor pr. over injection site for 3-5min
2) Barricade any stairways HBV vaccination
3) Ensure toys have no sharp edges, • Maintain normal BMI
prevent access to sharp instruments
4) Protective gear – car seats, seat belt,
helmets, protective pads • Long-term physiotherapy – promote strong muscle , protect
> 3yrs 1) bicycle – stabilizers/ guard wheels to joints and improve fitness
prevent falls • Regular joint and muscle Ex
2) Avoid contact sports, encourage
swimming, cycling • Complication Mx
1) chronic haemophilic arthropathy
• Avoid dental caries, brush 2x/day
• Early presentation to hospital for acute bleeds chronic joint bleeds target joints chronic inflammation of
• First aid the synovium thickening
• Avoid aspirin and NSAIDs
• Initiation of replacement therapy at home – via indwelling Gross swelling of J.
venous access
Not in SL Mx – control the bleeding
• Inform Dr/ dentist/ peers and teachers – carry diagnostic card
Synovectomy -
Page | 290
FMAS, RUSL
UNDER THE GUIDANCE OF DR. Anurudhdha Padeniya
Done by sunethra kumari 07/08,2nd Edition by vidushanka 08/09,3rd Edition Manoja 09/10
2) Inhibitor development – 4) Septic arthritis

• Desensitization programmes – high dose factor con- Prophylactic factor correction


centrates to saturate Ab level and dev. Tolerance

• If still unresponsive – plasmapheresis

Recombinant F VIIa / activated prothrombin


complex (PTC) –

Bypass inhibitors by direct activation of prothrombin


(II) needed for clot formation

3)Blood borne infections

Page | 291
FMAS, RUSL
Under the guidance of Dr.Anuruddha Padeniya
Done by Nalaka Rathnayaka 07/08,2nd Edition by Lilani 08/09,3rd Edition by Chinthani 09/10
…………………………………………………………………………………………………………………………………………………………..
Involved sites
ITP
Associated symptoms
9
Thrombocytopenia is reduce platelet count less than 150 x10 /L
Fever , lymph node enlargement, etc
Is the commonest cause of thrombocytopenia in children

History;
Time duration Short period - Acute ITP, Dengue etc
Age- Common among (1-10)yrs, Peak age (1-4) of age
Prolong period- Chronic ITP, SLE, etc
Sex- no difference
Aggravating factors
P/C
Recent viral/bacterial infection
Superficial bleeding, Generalized patachae, purpura
Drugs- Aspirin ,NSAIDs, Valproate, High dose of penicillin
or blistering
Severity-important in management
Mucosal bleeding, Gum bleeding, Epistaxis,

Profuse bleeding ,menorrhagia, etc

Any other bleeding, manifestation, intra cranial


bleeding, melena ,etc

Duration -short duration I or 2 days

Hx/P/C

On set Acute on set- ITP

Insidious on set -Chronic ITP,Systemic


condition like SLE

Page | 292
FMAS, RUSL
Under the guidance of Dr.Anuruddha Padeniya
Done by Nalaka Rathnayaka 07/08,2nd Edition by Lilani 08/09,3rd Edition by Chinthani 09/10
…………………………………………………………………………………………………………………………………………………………..

Severity

MILD MODERATE SEVERE

Mild superficial Pronounced su- Severe superficial


bleeding perficial bleeding bleeding

(Bruising) More trouble- Severe epistaxis


some epistaxis menorrhagia
Petechiae
Menorrhagia Melena
Minor epistaxis
Moderate inter- Bleeding epi-
Very little interfer- ference with sodes require
ITPance with daily of
is a diagnosis liv- exclution daily living transfutions or
ing hospitalization
Exclude DD’s –for ITP
Interfere seri-
ously with daily
Page | 293
living
FMAS, RUSL
Under the guidance of Dr.Anuruddha Padeniya
Done by Nalaka Rathnayaka 07/08,2nd Edition by Lilani 08/09,3rd Edition by Chinthani 09/10
…………………………………………………………………………………………………………………………………………………………..

Destructive Combined PLT Impaired PLT sequestration Non thrombocytopenic


thrombocytopenia and fibrinogen production purpuras
consumption
syndromes
Acute and chronic DIC Hereditary disorders Hyperspenism HSP
ITP
Leukemia ( ALL) Acquired disorders- Hypothermia Sepsis-meningococcal
aplastic
anemia,neoplasia,ne
utritional
deficiencies
SLE Burns Trauma
Drug induced (
HIT)
Evans syndrome
HIV
Thrombocytopenia
of Infections –
dengue,bacteraemia
Thrombotic
microangiopathic
disorders-HUS,TTP
vWD

Page | 294
FMAS, RUSL
Under the guidance of Dr.Anuruddha Padeniya
Done by Nalaka Rathnayaka 07/08,2nd Edition by Lilani 08/09,3rd Edition by Chinthani 09/10
…………………………………………………………………………………………………………………………………………………………..
ITP- precede by Viral infection 1-4 weeks back, purpura and Hematoma formation
petechial Rash, bleeding from gums and nostrils laceration-
pro long bleeding

Leukemia-recurrent infections, bruises ,bleeding excessively P/M/Hx -pervious episodes


,anemic features (dyspnea, pallor, Fatigue)Feeling sick, fever P/S/Hx - splenectomy
with chills, night sweat
Drug hx -Aspirin, Valproate ,high dose of penicil-
LOW, LOA if involves CNS headache migraine, Seizure ask lin
about cytotoxic drugs RX, Radiation
Birth Hx -umbilical bleeding ,sustain ICH, any
Dengue-fever arthralgia, myalgia, retro-orbital pain ,dengue bleeding manifestation
endemic area, petechial rash
Immunization Hx-heamatoma formation following
HSP-purpuric rash typically distributed over buttocks, IM injections
thighs and legs, arthralgia,myaligia
Diet Hx -usual diet Hx
HUS- bloody diarrhea (after 10 dys of diarrhea) ,oliguria and
hematuria in ARF ,Purpura and petechial rash, anemia fea- Family Hx -Bleeding disorders ,connective tissue
tures, features to suggest HT disorders

SLE-malar rash ,photosensitive rash, alopecia oral ulcers Social Hx -family background, monthly income,
join pain anemia features nearest hospital, transport facilities ,how disease af-
fect child’s day to day life
Complication

Anemia due to severe bleeding


EXAMINATION
chronic ITP
General
ICH headache, confusion, LOC, vomiting ,seizure

Page | 295
FMAS, RUSL
Under the guidance of Dr.Anuruddha Padeniya
Done by Nalaka Rathnayaka 07/08,2nd Edition by Lilani 08/09,3rd Edition by Chinthani 09/10
…………………………………………………………………………………………………………………………………………………………..
Anthropometry Free fluids

Well/ill CNS

Pallor(anemia) Features of ICH

Purpura and petechial rash Focal signs

Malar rash, photosensitive rash

Alopecia,qral ulcers

Generalized lymph adenopathy

Joint swelling Investigations

CVS FBC with red cell indices

PR,PV Thrombocytopenia (other component NL

BP(HT in HUS) Hb (reduce in profuse bleeding)

CRFT Evidence of infection by increased WBC

RS ESR repeatedly

Plural effusion Blood picture- large plt

Abd PT/INR- to exclude bleeding disorders

Tenderness BT,APTT (prolong in VWD)

Hepatosplnomegaly(Leukemia) CT

Page | 296
FMAS, RUSL
Under the guidance of Dr.Anuruddha Padeniya
Done by Nalaka Rathnayaka 07/08,2nd Edition by Lilani 08/09,3rd Edition by Chinthani 09/10
…………………………………………………………………………………………………………………………………………………………..
bone marrow aspiration- Indications Prednisolone 1-4 mg/kg/24hr continue (2-3 wk),until the
plt count become (>20x109)
- Unexplained anemia
Before start prednisolone do BM aspiration to rule out
- Abnormal WBC count in FBC ALL
- Findings of Hx and examination suggestive of bone
marrow failure or malignancy
If there is evidence of major bleeding manifestations -Other Tx
-If child is going to be treated with steroids for the Modalities
thrombocytopenic purpura
IV immunoglobulin (0.8-1g/kg/d for 1-2 d)
Comb test-(direct)–to exclude- Evan’s Xn ( unexplained ane-
mia and Thrombocytopenia) Rapid rise in platelet count (>20x109) within 48 hrs

ANA- to exclude SLE Disadvantages- expensive, time consuming to


administer, aseptic meningitis
Management
IV anti D therapy for Rh ( + )pt,( 50-75) micro g/kg for
Mild to moderate cases
3 days, rise in platelet count (>20x109)within( 48-
Educate, counseling of family and patient 72)hrs,

no Rx is needed In ICH-plt transfusion ,IV immunoglobulin, high dose


prednisolone , Ref neurosurgical unit
But in our setup prednisolone start in moderate symptoms
and put neurosurgical observation chart ,avoids drugs that in- Chronic ITP-Supportive Rx mainly ,can give rituximab
crease bleeding, monitor BP PR, Bed rest and thrombopoetic growth factors

Prednisolone, corticosteroids for acute/chronic ITP Splenectomy -indication

-older child (>4y)

Page | 297
FMAS, RUSL
Under the guidance of Dr.Anuruddha Padeniya
Done by Nalaka Rathnayaka 07/08,2nd Edition by Lilani 08/09,3rd Edition by Chinthani 09/10
…………………………………………………………………………………………………………………………………………………………..
-with sever ITP lasted >1y, 04. What are the treatment modalities?

-symptoms not control with Rx As mentioned above in the management part

-life threatening Hemorrhage which poorly respond 04. What is chronic ITP?
to Rx

Target questions

01. What is thrombocytopenia?


Thrombocytopenia is reduce platelet count less than 150 x109 /L
02. What is the investigation done before starting steroids for ITP
and why it is done?
BM aspiration biopsy, because steroid treatment temporally
mask the diagnosis of ALL, to exclude that BM biopsy is done prior
to the steroid treatment
03. What are the indications for BM biopsy in ITP?
- Unexplained anemia

- Abnormal WBC count in FBC

- Findings of Hx and examination suggestive of bone


marrow failure or malignancy

-If child is going to be treated with steroids for the


thrombocytopenic purpura

Page | 298
FMAS, RUSL
UNDER THE GUIDANCE OF DR. Anurudhdha Padeniya
Done by Nalaka Rathnayaka 07/08,2nd Edition by Lilani 08/09,3rd Edition by Chinthani 09/10

PLT count remains low 6 months after the treatments for episode 08. How about the outcome of the disease?
of ITP
Severe bleeding is rare, less than 3%
70-80% -spontaneous resolution with acute attack
05.When do you consider splenectomy in the case of ITP?
Less than- 1% develop ICH
Splenectomy –indicationS
Less than 20% go to chronic ITP
-older child (>4y
Therapy does not appeared to be affect natural history of disease
-with sever ITP lasted >1y,
Younger the age of onset, better the prognosis
-symptoms not control with Rx

-life threatening Hemorrhage which poorly respond


to Rx
06.What are the main consideration prior to splenetomy?
Vaccinations against encapsulated organisms-pneumococcal ,
meningococcal ,H- influenza
Prophylaxis with penicillin before 2 weeks
07. What are the advices we have to give?
Regarding clinic follow up
Close observation
Inform parents and teachers
Reduce contact sports

Page | 299
FMAS, RUSL
UNDER THE GUIDANCE OF DR. Anurudhdha Padeniya
Done by Shanika 07/08,2nd edition by Nayomi 08/09 and 3rd edition by pathum 09/10

Leukaemia
Other organ infiltration-
Presentation

Most common childhood neoplasm (31%) CNS-headache, vomiting, nerve palsies ,fit

Types Testis-enlargement

Acute lymphoblastic leukemia- ALL(77%) DD’s

AML 1.bleeding disorders-platelet, coagulation, vascular disorders

CML 2.AML-more in adolescents, features of marrow failure


Juvenile myelomonoclonic leukemia Subcutaneous nodules(blueberry muffin),infiltration of
General feature- malaise, anorexia ,irritability, intermittent low gingiva-uncommon ALL fx
grade fever
3.neuroblastoma-bone pain ,abdominal mass
Bone marrow infiltration features
4.Rhabdomyosarcoma
Anemia-lethargy, tiredness, exertional dyspnea
5.eving’s sarcoma
Neutropenia-recurrent infection, fever
6.myelofibrosis
Thrombocytopenia-gum bleeding, purpura,
7.IMN- acute onset of fever, intermittent high fever spikes ,throat
bruising, petechial,
hyperemia ,palatal petechial, rash appear following antibiotics.
Bone or joint pain or swelling
8.rheumatoid arthritis - bone pain and fever, with joint swelling
Reticuloendothelial infiltration- hepatospleenomegally (symmetrical and small joint in hand and feet)
,lymphadenopathy
History
Page | 300
FMAS, RUSL
UNDER THE GUIDANCE OF DR. Anurudhdha Padeniya
Done by Shanika 07/08,2nd edition by Nayomi 08/09 and 3rd edition by pathum 09/10
p/c P/M/hx

usually nonspecific clinical features for very short duration similar episodes in history, any other medical condition

anorexia, fatigue, malaise, irritability or fever birth Hx


lymphadenopathy BW,Antinatal,perinatal,postnatal complications
HO/P/C
Imm/Hx
describe symptoms eg-fever
Family Hx
usually symptoms appear after upper
respiratory tract infection in proceed- S/HX
ing 1-2 months
Economy, educational level, family support, other siblings
low grade intermittent fever, bone pain is severe can wake status
the patient night
Transport facility
progressing disease- feature of bone marrow failure
Accessibility to a specialized center
pallor, fatigue, exercise intolerance,
bruising, epistaxis knowledge regarding the disease

respiratory distress due to anemia or obstructive airway due further fertility wishes
to large anterior mediastina mass Examination
predisposing factors
General-pallor ,purpuric and petechial lesions, mucus membrane
environmental-exposure to ioniz- hemorrhages, mouth ulcers, lymphadenopathy
ing radiation, alkylating agents,
drugs CVS -features of hyper dynamic circulation-tachycardia, high
volume pulse, flow murmurs
genetic condition-down syndrome
,fanconi anemia ,turner’s, ataxia RS-evidence of distress
telangiectasia
Page | 301
FMAS, RUSL
UNDER THE GUIDANCE OF DR. Anurudhdha Padeniya
Done by Shanika 07/08,2nd edition by Nayomi 08/09 and 3rd edition by pathum 09/10
Abdomen-distention, bleeding manifestations Radiography-lytic bone lesions, mediastinal
,hepatospleenmegally, testicular swelling mass

CNS-signs of increased ICP (rarely),papilledema ,retinal USS abdomen-hepatospleenomegally


hemorrhage ,cranial nerve palsy CT,MRI as necessary
Musculoskeletal-joint swelling or effusion, bone tenderness MANAGEMENT;
INVESTIGATIONS Without effective therapy disease is fatal
1. FBC-Hb low ,plt low ,WBC low or high or normal General Mx
Median WBC count at presentation is 30000,but 75% it is 1) Counseling & psychological support
less than 20000 indicates bone marrow failure.eg- 2) Analgesics for pain relief
anemia, thrombocytopenia 3) Establish good fluid and electrolyte balance
4) Nutritional support
2. BLOOD PICTURE-patient with WBC less than 10000,leu- 5) Control of infections – broad spectrum AB
kemic cells initially appear as atypical lymphocytes but cells Prophylaxis for pneumocystis carinii – co-trimoxazole
more than 20% 6) Antiemetic for nausea and vomiting (metochlopramide, phenothi-
azine)
3. ESR,CRP-high
7) Anemia – RC concentrate Tx thrombocytopenia – Plt. Tx
4. Bone marrow aspiration-may sufficient alone for diagnosis 8) Manage tumour lysis Xn (↑ urate) – Allopurinol, IV fluids
9) Manage coagulopathy if present – Vit K and FFP
5. Bone marrow biopsy-sometimes require to provide adequate
Specific Mx
tissue to exclude other possible causes eg- myelofibrosis,
Chemotherapy given in 3 phases
aplastic anemia
• Remission induction (4-6 weeks)
6. Other investigation- - Kill most tumour cells
- Prednisolone, Vincristine
CSF-high leukocytes
- 98% →4 weeks →if poor response another 2 weeks
Biochemical-LDH high, serum uric acid high

Page | 302
FMAS, RUSL
UNDER THE GUIDANCE OF DR. Anurudhdha Padeniya
Done by Shanika 07/08,2nd edition by Nayomi 08/09 and 3rd edition by pathum 09/10
- After treatment → 5% blast cells in BM Bx, normal Allogenic stem cell transplantation – take peripheral
peripheral blood film blood of Pt. →filter stem cells and transfer
• Intensification / consolidation
- High dose multi drug chemotherapy →↓tumour burden
to very low levels 3) BM transplant –small number of patients with poor prognosis eg-
- Rx – cyclophosphamide, daunorubicin, cytosine philedelphia chromosome
- ↑ Complications – tumour lysis Xn HLA matched, ideally monozygotic twin/ other sibling → deposit
- Temporary withholding of drugs until WBC >5000; if harvested BM cells
not WBC <2000 →DEATH Complications – graft rejection/ failure
- If low Hb →Tx, AB, anti-fungals
TARGET QUESTIONS
• Maintenance
- ♀ & adults – 2yrs young ♂ - 3yrs (to reduce testicles) 1)What are the important poor prognostic factors of disease
- O. mercaptopurine – daily
1. Age of patient at the time of diagnosis-≤1yr,≥10yr
- O. methotrexate – weekly
- IV vincristine 2. Sex –male
- Prednisolone for 5days monthly/ 3 monthly
3. WBC-≥15000
- High risk of varicella and measles → prophylactic Igs
on exposure 4. Speed of response to treatment
1-10 year and WBC ≤50000-average risk
• Cranial prophylaxis for CNS disease
- IV/ intrathecal methotrexate ≥10yr or initial count ≥50000 high risk
- Cranial irradiation →avoided
Time to clear blast cells from ciculation≥1
2) Treatment of relapses
week
High dose chemotheraphy
Total bodt irradiation- Before irradiation → sperms Time to remission ≥4week
conserved – sperm bank
CNS disease and presentation

Page | 303
FMAS, RUSL
UNDER THE GUIDANCE OF DR. Anurudhdha Padeniya
Done by Shanika 07/08,2nd edition by Nayomi 08/09 and 3rd edition by pathum 09/10
2)why you do LP

Look for CNS infiltration

3)what specific therapy you do after LP

Give intrathecal chemotheraphy to prevent CNS


relapses

4)what is blast crisis

It occur in chronic myeloid leukemia,and after 3-4


yr of onset,patient condition get worsen,blood count largely
increased,pt get hyperuricemia,neurological symptoms,CNS
perfusion reduced due to increased blood viscosity

5)What is the specific drug treatment for CML and what is the
action of that

IMATINIB MESYLATE

Inhibit thyrosin kinese

Page | 304
FMAS, RUSL
UNDER THE GUIDANCE OF DR. Anurudhdha Padeniya
Done by Madushani and Dimantha 2nd edition Nayana 08/09, 3rd edition Prathiba 09/10
-abrupt and disturb daily activity of the child and family
Sickle cell disease -Preventive measures, are they practice at home

Presentation – Eg. Cover with blanket, relaxation techniques, massaging

-Jaundice with anemic symptoms (lethargy, poorexercise tolerance,sob, Features of anemia, yellowishdiscoloration of body
drowsy)
-Recurrent episodes of infection and hospitalization
-features of painful crises (pain occurs in any part of the body mainly
-History of silent infarction (headache, seizures,loss of consciousness)
abdomen chest and extremities)
history of priapism
-features of hand feet syndrome (swelling of the fingers or feet)
-History of fever,chest pain and respiratory distress
-infection
-History ofhematuria,frothyuria, decrease urinary out put
-leg ulcers
-History ofdactylitis(symmetrical unilateral swelling and pain of the
-priapism fingers or feet

-features of hemolysis -History of leg ulcer

History of presenting complain -History of gallstone disease (left hypochondrialcolickypain,


frequentflatulence, features of obstructive jaundice)
-Previously well or diagnose patient for sickle cell anemia
-History of blood transfusion
-Pain full crises
Past medical history-renal disease,hypertension,epilepsy,arthritis,cardiac
-unremittent discomfort occur in any part of the body common in the diseases
chest, abdomen and extremities.
Past surgical history-splenectomy
-Pain is precipitated by physical stress infection dehydration hypoxia
exposure to the cold weather and swimming for prolong time. Allergic history –drug, food or plasters

-pain is respond to analgesics Drug history –penicillinprophylactically

Page | 305
FMAS, RUSL
UNDER THE GUIDANCE OF DR. Anurudhdha Padeniya
Done by Madushani and Dimantha 2nd edition Nayana 08/09, 3rd edition Prathiba 09/10
Birth history-body weight,mode of delivery Features of anemia

Immunization history-vaccination for influenza, Features of dehydration


pneumococcal,H.influenza,Hepb
Dactylitis
Development-school performance and cognitive function
Leg ulcers
Family history-sickle cell disease, otherhematological manifestation and
consanguinity Ankleedema

Social history-parentaleducation, occupation and monthly income CVS

Knowledge regarding Identification of pain full crises and avoiding of -Pulse


precipitation -BP
Factors -features of cardiac failure
How disease affect child day to day life and family burden, Nearest RS
hospital
-rate
Examination
-VB/Added sound
Weight
ABDOMEN
Height
-distension
BMI
-tenderness, guarding
General appearance - ill looking
-hepatospleenomegaly
Febrile or not
-hyposplenism
On pain or not
Throat examination
Features of respiratory distress

Page | 306
FMAS, RUSL
UNDER THE GUIDANCE OF DR. Anurudhdha Padeniya
Done by Madushani and Dimantha 2nd edition Nayana 08/09, 3rd edition Prathiba 09/10
-tonsils for enlargement -Done in neonates

-in the first week of life

CNS Examination -using Guthrie test-done ondried blood spots

-Consciousness

-Orientation Management

Muscular skeletal examination Prophylaxis

-Ulcers features of osteomyelitis Daily oral Penicillin throughout childhood

-GAIT

Investigations Immunization for

Diagnosis -encapsulated organisms

FBC Eg-Streptococcus pneumonia

Retic count -Haemophilus influenza type B

Blood picture Folic acid

Blood culture -oral

UFR -once daily

Chest x-ray Minimizing vasoocclusive crises-

HPLC for confirm the diagnosis -avoiding exposure to cold,dehydration,excessive


exercise,undue stess,hypoxia
Screening

Page | 307
FMAS, RUSL
UNDER THE GUIDANCE OF DR. Anurudhdha Padeniya
Done by Madushani and Dimantha 2nd edition Nayana 08/09, 3rd edition Prathiba 09/10
-using practical measures such as dressing children -used for most severly affected
warmly,giving drinks,esp.before exercise children who do not respond to hydroxy
urea

-can only be safely carried out if


Treatment of acute crisis the child has an HLA identical sibling
Painful crisis-oral/IV analgesics according to need(may -cure rate 90%
require opiates) with good hydration
-5% risk of fatal transplant related
Infections-Antibiotics complications
Oxygen-if the oxygen saturation is reduced

Exchange transfusion-indicated for acute chest


syndrome,stroke & priapism

Treatment of cronic problems

Hydroxyurea-a drug which increases HbF production & help against


further crises

-benefits in children who have recurrent hospital admission for painful


vaso-occlusive crises /acute chest Xn

-requires monitoring for side effects

Esp.WBC suppression

Bone marrow transplantation-the only cure sickle cell disease

Page | 308
FMAS, RUSL
UNDER THE GUIDANCE OF DR. Anurudhdha Padeniya
Done by Rasika Harshani Premarathna 07/08, 2nd Edition by Gayan 08/09. 3rd Edition by Iresha 09/10
Connective tissue- JIA, SLE, dermatomyosities,MCTD
JIA
Presentations:

Arthritis for > 6 weeks

LOA, LOW, malaise,fever with high spikes,rash,arthral-


gia,myalgia
Reiter ’s syndrome
Limping
History:
Morning stiffness relived by rest or gelling phenomenon
P/C→
Cant stand in the crib in the morning or after naps
o Arthritis with duration
DD’s:

Infection- septic arthritis Hx/P/C→

Reactive arthritis o Regarding arthritis

Rheumatic arthritis - No of joints involved

Inflammatory bowel disease(IBD/UC) -distribution ;large joints


Vasculitis- HSP Small joints
Kawasaki’s dx
Spine/TMJ
Haematological- haemophilia, sickle cell
-symmetrical/asymmetrical
Malignant - leukaemia,Neuroblastoma
-symptoms;
Page | 309
FMAS, RUSL
UNDER THE GUIDANCE OF DR. Anurudhdha Padeniya
Done by Rasika Harshani Premarathna 07/08, 2nd Edition by Gayan 08/09. 3rd Edition by Iresha 09/10
Swelling,warmth,redness,pain Uveitis(red eye,blurredvision,pain)

Morning stiffness & gelling after inactivity Associated enthesitis(pain in heel,soles)

Severity Low back pain ( sacroilitis)

Funtional status: Features of systemic onset JIA

limping Exclde DD

Slowness on walking -Rheumatic fever:( hx of sore throat/

Reduse range of movements Murmur/exertionaldyspnoea)prophylaxis of


peniciline,abnormal movements
Worse at early morning/ end of day
-Hx of rubella, mumps infection
-Pattern ;migatory/fliting
-IBD: (blood & mucus diarrhea)
Continuous
-Reactive arthritis(hx of fever & diarrhea)
Recurrent
-HSP(purpura in buttocks& extensors/
Destructive
Ab pain/maleana/haemturia)
-progression; mild & self limiting- viral
-Kawasaki’s(high spiking remittent fever/
Mild & chronic – JIA
Swollen tongue& lips/peeling of skin in hand)
Acute & rapid – septic
-SLE;(oral ulcer /photosensitive rash,alopesia,photophobia,)
-Exrta-articular features;
-Reiters(urethritis/uveitis/arthritis)
Page | 310
FMAS, RUSL
UNDER THE GUIDANCE OF DR. Anurudhdha Padeniya
Done by Rasika Harshani Premarathna 07/08, 2nd Edition by Gayan 08/09. 3rd Edition by Iresha 09/10
-Leukamia(bone pain/gum bleeding/epistaxis) -haemophilia

-Haemophilia& Sickle cell dx -connective tissue dx

-Hx of trauma -IBD

-Septic arthritis(any inflammatory foci) -Dental extraction

What treatments was given -heart dx

Drugs & SE of drugs, compliances P/S/H;

Any referrals Correction of limb deformities

Surgery & splints Joints prosthesis

Complication ;

Joints contractrures/deformities Family Hx

Chronic anterior uveitis Large joint arthritis

Growth retardation Psoriasis in 1st degree relatives

Amyloidosis-protinuria/CRF

Involment of cervical spine;UL/LL weakness Haematological malignancy

Bleeding disorders

P/M/H; Connective tissue dx

Page | 311
FMAS, RUSL
UNDER THE GUIDANCE OF DR. Anurudhdha Padeniya
Done by Rasika Harshani Premarathna 07/08, 2nd Edition by Gayan 08/09. 3rd Edition by Iresha 09/10
IBD Examination

General;

Social Hx -Ht,Wt(growth retardation)

-parents education ,financialstatus,knowledge about dx -ill/well

-how dx affects childs day today activity, -pain,depressed

Schooling,playing or whether child is -wasting

Immobile -hydration

-how does it affect family,missing job, -aloplecia(SE of MTX)

Expenses for travelling,drugs,hospital stay -cushinoid (steroids)

Oter children affected or not -febrile or not

-housing; steps/toilets/water/floor -Eyes;

-nearest hospital & transport Pallor(chronic dx,HSP,IBD)

-physiotherapy Icterus(JIA,SLE)

Conjunctivitis(reactive,Kawasaki)

Scleritis(SLE,HSP)

Anterior uveitis(JIA)

Page | 312
FMAS, RUSL
UNDER THE GUIDANCE OF DR. Anurudhdha Padeniya
Done by Rasika Harshani Premarathna 07/08, 2nd Edition by Gayan 08/09. 3rd Edition by Iresha 09/10
-mouth; -evidence of inflammation

Oral ulcers(SLE,Kawasaki) (swelling,redness,warmth,tender

Strawberry tounge(Kawasaki) ,reduce movements)

Cracked lips(Kawasaki) -evidence of truma

Gum bleeding/epistaxis(leukemia) -deformities,contractures

-skin; -muscle wasting

Salmon pink,macular rash(SOJIA) -ex of other joints

Purpura in buttocks&extensors(HSP) CVS

Peeling of skin in hand&feet(Kawasaki Features of pericarditis

Erythema nodosum (IBD) RS

Subcutaneous nodules (RF) Features of pleuraleffusion(SOJIA)

Echymoticpatches(haematological) Abdomen

Hepatosplenomegaly(SOJIA)

-face; photosensitive rash(SLE) Target Quections

-lymphadenopathy(SOJIA,malignancy) 1 How are you going to Ix patient with arthritis

Examination of joints 1. Lab Ix – ESR (high)

Page | 313
FMAS, RUSL
UNDER THE GUIDANCE OF DR. Anurudhdha Padeniya
Done by Rasika Harshani Premarathna 07/08, 2nd Edition by Gayan 08/09. 3rd Edition by Iresha 09/10
FBC (Low Hb ,highneutropils, platelets Can visualize both inflammatory &

In SOJIA) Distructive manifestation.

Blood picture 5. CT/MRI

2. Synovial fluid ex 6.Radionucleideinaging

Septic arthritis- turbid More sensitive in septic arthritis &

Low viscosity osteomyelitis

High neutropils 7. Special Ix

Culture positive Coagulation profile

Inflamatoryarthropathy; ANA

Turbid, yellow green Rheumatoid factor

Iow viscosity ASOT

High neutropils Blood culture

3.X-ray Bone marrow biopsy

Not sensitive in detecting early bone dx 2 How are you going to manage patient

Or soft tisse manifestation. With JIA?

4. USS Multidiscipinary approach

Page | 314
FMAS, RUSL
UNDER THE GUIDANCE OF DR. Anurudhdha Padeniya
Done by Rasika Harshani Premarathna 07/08, 2nd Edition by Gayan 08/09. 3rd Edition by Iresha 09/10
1. Pharmacological Splinting to prevent contactures

-NSAIDS; all pt with active arthritis to control 3. Supportive Rx;

Pain & suppress inflammation. -diet (balanced diet , high ca&fe)

-dental care (early dental referral & jaw exercise)

Ibuprofen,piroxicam,naproxen -eye care (regular ophthalmological screening

-if not responding to NSAIDS intra articular Using a slit lamp for all patients)

Corticosteroid therapy. -physiotherapy;

-IV methyl prednisolone(active arthritis/ Daily exercise

Systemic arthritis wth pericarditis) Important to encourage mobility & maintain a

-DMARDS Full range of movements & muscle strength to

-Methotrexate; effective 70% of children with polyarthritis. Prevent contractures

Hydrotherapy

2. Surgical Rx; -occupational therapy

-replacement of joint Make the child independent

-orthopaedic Rx; Dressing, washing, eating, toilet training

Shoe lift on shoter side to prevent scoliosis -psychological support

Page | 315
FMAS, RUSL
UNDER THE GUIDANCE OF DR. Anurudhdha Padeniya
Done by Rasika Harshani Premarathna 07/08, 2nd Edition by Gayan 08/09. 3rd Edition by Iresha 09/10
-social service support

-discharge

Tell what the pt has

Importance of drug compliance, follow up, drug SE

Page | 316
FMAS, RUSL
UNDER THE GUIDANCE OF DR. Anurudhdha Padeniya
Done by F.Rafeek 07/08, 2nd edition by Rfhan 08/09, 3rd edition by Darshani 09/10
History:
Hypothyroidism
P/C→
Introduction:
Prolongation of physiological jaundice
Congenital or acquired
Failure to thrive
Congenital-
Developmental delay
Commonly due to thyroid dysgenesis

Some are familial


Hx/P/C→
May associated with goiter
Feeding difficulty
1 in 3000 infant affected in world wide
Lack of interest
Acquired-
Somnolence
most common cause is chronic lymphocytic thyroidi-
tis Chocking spell during nursing
Most are asymptomatic at birth
Respiratory difficulty- Apnea attack, noisy respira-
Fetal thyroid predominantly produces rT3 which is largely tion, nasal obstruction
inactive.
Cry little, sleep much, poor appetite
After birth there is a surge in the level of TSH and therefore
marked rise in T4 and T3. Constipation does not respond to treatment

Then the TSH level decline to the normal adult range within Lethargy
a week.
Developmental delay
Page | 317
FMAS, RUSL
UNDER THE GUIDANCE OF DR. Anurudhdha Padeniya
Done by F.Rafeek 07/08, 2nd edition by Rfhan 08/09, 3rd edition by Darshani 09/10
Late in learn to sit and stand P/S/H

Hoarse voice, does not learn to talk Thyroidectomy for thyrotoxicosis or carcinoma

Delayed growth Irradiation for head & neck malignancy, Hodgkin


lymphoma
Weight gain
Drugs- iodides, amiodarone, antithyroid drugs,
Cold intolerance valproate, lithium carbonate

Muscle weakness, cramps Birth Hx→

Delayed puberty

Menometrorrhagia in females Imm Hx→

Pseudo puberty- breast development in female & Diet Hx


macro-orchidism in male
Social Hx→
Headache, vision problem
Examination:
Mental retardation
General→
Macrocytic anemia refractory to hematinics o Stunted
P/M/Hx→ o Short extremities
DM, Celiac disease, Down syndrome, Turner syn- o Dry, scaly & cold skin
drome, Klinfelter syndrome, Addison disease, Perni-
cious anemia, Chronic hepatitis C infection, Large o Skin shows generalized pallor
hemangioma of the liver
Page | 318
FMAS, RUSL
UNDER THE GUIDANCE OF DR. Anurudhdha Padeniya
Done by F.Rafeek 07/08, 2nd edition by Rfhan 08/09, 3rd edition by Darshani 09/10
o Myxedema of eyelid & back of hand o Deposition of the fat above clavicle& between neck
& shoulders
o Head size- normal or increase
o Broad hand
o Scalp is thickened
o Short finger
o Anterior &posterior fontanel- widely open
o Goiter – non tender, firm, rubbery
o Coarse, brittle & scanty hair
CVS→
o Hair line reaches far down on forehead
o Bradycardia
o Coarse & ugly face
o Slow pulse
o Carotenemia- yellowish discoloration of eyes but
sclera remain pale o Cardiomegaly

o Eyes appear apart o Murmurs

o Swollen eyelid o Pericardial effusion- asymptomatic

o Narrow palpabral fissure RS→

o Depressed bridge of broad nose o

o Mouth kept open Abd→

o Large abdomen
o Protruded broad, thick tongue
o Umbilical hernia
o Delayed dentition
o Edema of genitalia
o Short & thick neck
Page | 319
FMAS, RUSL
UNDER THE GUIDANCE OF DR. Anurudhdha Padeniya
Done by F.Rafeek 07/08, 2nd edition by Rfhan 08/09, 3rd edition by Darshani 09/10
Radiograph – Retardation of osseous development , epiphy-
sis have multiple foci ossification, deformity (beaking) of
CNS→ 12th thoracic or 1st/2nd lumbar vertebra
o Hypotonic X ray knee & ankle (one film)- absent distal femoral epiphy-
o Ataxia sis, absent cuboids

o Deep reflex exaggerated & slow relax X ray hip – epiphyseal dysgenesis (pathognomonic)

Investigations: Roentgenogram of skull - large fontanels & wide sutures

Neonatal screening ECG – large voltage P & T wave with diminished amplitude
QRS complex ( poor ventricular function)
o Blood from heel prick
ECHO (pericardial effusion)
o Place on filter paper card
>2years – serum cholesterol ↑
o 2-5 day
SE – hyponatremia
o T4 ↓, TSH ↑
↑ CPK
Serum T4 ↓
Blood picture – macrocytic anemia
Free T4 ↓
Bone age X ray
T3 may be normal & not help in diagnosis
Antithyroglobulin & antiperoxidase
1ry hypothyroidism TSH >100mU/L

Serum thyroglobulin - ↓ in thyroid agen-


esis, defect in thyroglobulin synthesis or secretion ↑ in
ectopic gland, inborn error of thyroxin synthesis
Management:

Page | 320
FMAS, RUSL
UNDER THE GUIDANCE OF DR. Anurudhdha Padeniya
Done by F.Rafeek 07/08, 2nd edition by Rfhan 08/09, 3rd edition by Darshani 09/10
Levothyroxine Poor sleeping habits Restlessness Be-
havioral problems Deterioration of school work
o Treatment of choice Short attention span
o Oral Constipation – bisarcodyl, lactulose
o Single dose Correct anemia
o Empty stomach Heart failure – frusemide
o Should not mixed with soy protein formula, concen-
trated Fe/Ca (inhibit absorption)
Target Questions
o Neonates – 10-15 mg/kg/day (totally 37.5-50
mg/day) 1-3 years-- 4-6 mg/kg/day 3-10 48) Complications in management of complicated hypothyroid-
years - 3-5mg/kg/day 10-16 years- 2-4mg/kg ism
/day

o Dose gradually decrease with age Heart failure; Impaired ventricular systolic and diastolic
functions and increased peripheral vascular
o Monitor S.T4, free T4 & TSH – monthly in 1st 6 resistance
months & every 2-3 months in 6months to 2 years
&6 weeks after change in dose Ventilatory failure ; Blunted hypercapneic and hypoxic
ventilatory
o Over treatment cause craniosynostosis tempera- Drives
ment problem advanced epiphysis fusion
Hyponatremia; Impaired renal free water excretion and
o In lifelong treatment titrate dose to maintain the nor- syndrome of inappropriate antidiuretic
mal growth
Ileus; Bowel hypomotility
o Advice mother to child may be got following symp- Medication sensitivity; Reduced clearance rate and increased
toms within 1 year of treatment & it is transient sensitivity to sedative, analgesic, and
anesthetic agents
Page | 321
FMAS, RUSL
UNDER THE GUIDANCE OF DR. Anurudhdha Padeniya
Done by F.Rafeek 07/08, 2nd edition by Rfhan 08/09, 3rd edition by Darshani 09/10
Congenital Hypothyroidism
Hypothermia and lack of
Febrile response to sepsis; Decreased calorigenesis PRIMARY HYPOTHYROIDISM

Delirium, dementia, seizure, Defect of fetal thyroid development (dysgenesis)


stupor, and coma; Decreased central nervous system thyroid • Aplasia
hormone actions, and encephalopathy • Hypoplasia
due to • Ectopia
hyponatremia and hypercapnia Defect in thyroid hormone synthesis (dyshormonogenesis)
• Iodide transport defect: mutation in sodium-iodide symporter gene
Adrenal insufficiency; Associated intrinsic adrenal or pituitary • Thyroid organification, or coupling defect: mutation in thyroid
disease, or reversible impairment of peroxidase gene
hypothalamic-pituitary-adrenal stress • Defects in H2O2 generation: mutations in DUOXA2 maturation
response factor or DUOX2
gene
• Thyroglobulin synthesis defect: mutation in thyroglobulin gene
• Deiodination defect: mutation in DEHAL1 gene
49) follow up TSH unresponsiveness

1) features of hypothyroidism • Mutation in TSH receptor


Defect in thyroid hormone transport: mutation in monocarboxylate
2) Hr, dyspnoea, liver transporter 8
(MCT8) gene
3) Linear growth, OFC Iodine deficiency (endemic goiter)
Maternal antibodies: thyrotropin receptor–blocking antibody
4) Serial photograph (TRBAb, also termed
thyrotropin-binding inhibitor immunoglobulin)
5) T4, TSH Maternal medications
• Iodides, amiodarone
6) Bone age X ray annually • Propylthiouracil, methimazole
• Radioiodine

Page | 322
FMAS, RUSL
UNDER THE GUIDANCE OF DR. Anurudhdha Padeniya
Done by F.Rafeek 07/08, 2nd edition by Rfhan 08/09, 3rd edition by Darshani 09/10
CENTRAL (HYPOPITUITARY) HYPOTHYROIDISM Systemic disease
PIT-1 mutations • Cystinosis
• Deficiency of TSH • Langerhans cell histiocytosis
• Deficiency of growth hormone Hemangiomas (large) of the liver (type 3 iodothyronine deiodinase)
• Deficiency of prolactin Hypothalamic-pituitary disease
PROP-1 mutations
• Deficiency of TSH
• Deficiency of growth hormone
• Deficiency of prolactin Clinical features
• Deficiency of LH
• Deficiency of FSH Congenital

Usually asymptomatic and picked up on screening.


TSH
hypothyroidism with elevated TSH level) Otherwise: Failure to thrive
Multiple pituitary deficiencies (e.g., craniopharyngioma) Feeding problems
TRH deficiency Prolonged jaundice
• Isolated Constipation
• Multiple hypothalamic deficiencies (e.g., septo-optic dysplasia) Pale, cold, mottled dry skin
TRH unresponsiveness Coarse facies
Mutations in TRH receptor Large tongue
Teeaeaee59-3 ETIOLOGIC CLASSIFICATION OF ACQUIRED Hoarse cry
Acquired hypothyroidismYROIDISM Goitre (occasionally)
Autoimmune Umbilical hernia
• Hashimoto thyroiditis Delayed development
• Polyglandular autoimmune syndrome, types I and II
Iatrogenic . Acquired
• Propylthiouracil, methimazole, iodides, lithium, amiodarone
• Irradiation Otherwise: Failure to thrive
• Radioiodine Feeding problems
• Thyroidectomy Females > ma Short stature/growth failur Cold intolerance
Dry skin
Cold peripheries
Page | 323
FMAS, RUSL
UNDER THE GUIDANCE OF DR. Anurudhdha Padeniya
Done by F.Rafeek 07/08, 2nd edition by Rfhan 08/09, 3rd edition by Darshani 09/10
Bradycardia
Thin, dry hair
Pale, puffy eyes with loss of eyebrows
Goitre
Slow-relaxing reflexes
Constipation
Delayed puberty
Obesity
Slipped upper femoral epiphysis
Deterioration in school work
Learning difficulties

COUNSELLING

reassurance, make aware that the early treatment of congenital


hypothyroidism is essential to prevent learning difficulties.
Treatment is lifelong with oral replacement of thyroxin. There fore
councel on importance of compliance and clinic followup

Page | 324
FMAS, RUSL
UNDER THE GUIDANCE OF DR. Anurudhdha Padeniya
Done by Chathuranga 07/08, Seegiri 08/09, Amandha and Dhanushka 09/10
Hyperglycemia
Diabetes Mellitus ( type 1) Cerebral edema
Presentation
DKA
Acute presentation (within hours)
Cerebral infections
Nausea / vomiting
(For sub-acute presentation)
Severe abdominal pain
Diabetes mellitus
Hyperventilation (heavy rapid breathing) Diabetes insipidus
Ketotic breath (fruity smell breath) Primary polydipsia
Confusion, convulsion & coma CRF
Sub-acute presentation ( within days or weeks) Diuretics
Polyuria (>2l/m2/hr) & nocturia History:
Polydypsia P/C: Acute presentation/ Sub acute or Sub acute followed by
acute
Dry mouth and throat
Hx/P/C:(Presentation may be 1st episode of DM or later
Increased appetite presentation of already diagnosed child with DM due to poor
insulin compliance)
Weight loss
In acute presentation
Mild fever/ fatigue/irritability/ unusual behavior
Describe the symptoms in chronological order with durations
Nocturnal enuresis
How the symptoms were progressed
DD’s: (for acute presentation)
Shock

Page | 325
FMAS, RUSL
UNDER THE GUIDANCE OF DR. Anurudhdha Padeniya
Done by Chathuranga 07/08, Seegiri 08/09, Amandha and Dhanushka 09/10
What has been done up to now at the GP’s, peripheral hospi- Ask about complications (rare)
tals or ETU,(RBS, ABG, ECG, IV, cannulation, insulin, IV
fluids urine tests) Recurrent infections(UTI, oral/genital candidiasis,
skin sepsis)
Improvement of symptoms
Visual impairments
Ask about precipitating cause for DKA
CKD- frothy urea
Infection
Tingling sensation of hands & feet
Dehydration
Surgery/trauma
P/M H→
Poor insulin compliance
in already diagnosed child with DM
Then, if there sub-acute symptoms, describe them with du-
rations. Initial presentation- how worse

Exclude the DDs, Diagnosis- how, at what age


Shock-Severe vomiting & diarrhea leading to Initial treatments & changes
hypovoalemia
Insulin- (type, dose, frequency, time, sites, technique,
Cerebral edema- headache ,restlessness, drowsiness storage, SI, compliance )
& incontinence
Glycemic control-(diet, glucometer, FBS monthly,
Cerebral infection-high fever, photophobia, neck
HbA1c values, diary maintenance, clinic follow up)
stiffness, altered behavior & fits

Diabetes insipidus past episodes of DKA , HYPOGLYCEMIA, and pre-


vious hospital admissions
CRF – frothy urea, body swelling
Ask for the complications and what has been done for
UTI – dysuria, hematuria their mx (growth delay, eye- retinopathy, cataracts,
renal diseases ,foot ulcers)
Page | 326
FMAS, RUSL
UNDER THE GUIDANCE OF DR. Anurudhdha Padeniya
Done by Chathuranga 07/08, Seegiri 08/09, Amandha and Dhanushka 09/10
Renal diseases Gyn hx- Achieved menarche/not (achieving puberty delayed as
a complication)
Young hypertension

Associated Autoimmune diseases-(thyroiditis, vitiligo,


pernicious anemia, coeliac disease, rheumatoid arthritis, Diet Hx
Addisons disease)
Breakfast, lunch & dinner
Thalassemia
Short eats, fast foods & sweets.
Mumps/pancreatitis
Triggers of getting DM - cow’s milk
Cushing's Xn, chromosomal Xn- down's, turner's Xn
Manipulate food intake with insulin to achieve good gly-
cemic control
Triggers of getting DM - viral infections

P.S Hx→
family Hx→
I & D of abscess
DM in 1st degree relatives
Drug Hx→
Autoimmune disorders
Diuretics

Long term steroid


Social Hx→
Birth hx- congenital rubella Xn, syndromic baby (Down’s Xn,
Turner's Xn) Parental educational state

(May be associated with them) Family income & support

Child’s school performance & sports , school abstinence


due to recurrent hospital admissions

Page | 327
FMAS, RUSL
UNDER THE GUIDANCE OF DR. Anurudhdha Padeniya
Done by Chathuranga 07/08, Seegiri 08/09, Amandha and Dhanushka 09/10
Interference of disease with normal life Level of consciousness/GCS

Knowledge regarding the disease, complications, insulin Ill looking/drowsy/irritability


& prognosis
Dyspnea/having oxygen
(Insulin-storage, technique, complications)
Fruity odor breathing
knowledge regarding home blood glucose moni-
toring, testing urine sugar In skin -signs of insulin resistance; acanthosis nigricans-
velvety dark skin on neck or arm pits
Eye sight of the person who inject insulin
Dehydration - dry mucus membranes, reduced skin tur-
compliance for drugs and clinic follow up gor, sunken eyes.

How to identify a DKA attack & what should do imme- No of cannulas on limbs
diately
Receiving IV fluids/inulin
Whether school teachers aware of it
Febrile to touch
With whom child go to school and come
pallor or cyanosis
Impact on family and child
lymphadenopathy
How to identify hypoglycemic attacks and what to do
cold peripheries
If adolescence -hx of smoking, alcohol.
Inspect feet for non-heeling ulcers
Examination:

General→
RS→
Height/weight/OFC
RR, kussmaul breathing
BMI/emaciated
Page | 328
FMAS, RUSL
UNDER THE GUIDANCE OF DR. Anurudhdha Padeniya
Done by Chathuranga 07/08, Seegiri 08/09, Amandha and Dhanushka 09/10
Recessions& accessory muscle use Cranial nerves
Chest expansion Check eyes for features of retinopathy and cataracts.
Air entry , breath sound, added sounds Investigations:
CVS→ (Aim to diagnose, monitor the therapy & find an infection)
CRFT
PR, Volume, rhythm, character, peripheral pulse Capillary blood sugar/RBS> 11.1mmol/l (200mg/dl) +
BP symptoms(&glycosuria and ketonuria)
Apex, heart sounds murmurs
FBS >7 mmol/l (126mg/dl)
Abd→
HbA1c -previous 3 months control
Distention
tenderness TSH (as an associated condition)

hepatosplenomegaly Microalbumin, BU, SCr- to monitor renal functions

bladder, kidneys FBC & ABG

free fluids SE, BU &S.Cr

CNS→ Urine ketone bodies & UFR

Higher functions ECG & CXR

Neck stiffness blood & urine cultures

Kerning sign Management:

Motor system(tone, power, reflexes) DKA emergency management


general resuscitation - ABC
Sensory confirm the diagnosis- DKA (clinical & inv)
assess the degree of dehydration
Cerebellar signs
Page | 329
FMAS, RUSL
UNDER THE GUIDANCE OF DR. Anurudhdha Padeniya
Done by Chathuranga 07/08, Seegiri 08/09, Amandha and Dhanushka 09/10
monitoring – BP, HR, RR, IPOP, ECG, U.ketones, RBS, Long term management of type 1 DM
weight, ABG, SE After clinical improvement child can be discharged on sub-
start IV fluids(Management of dehydration) – cutaneous insulin. (mixtard)
o bolus if shocked – 10ml/kg - to be given via syringe, pen device, S/C pump
o deficit according to dehydration -sites- upper arm, ant. and lat. aspects of thigh, buttocks,
o maintenance according to weight abdomen(with rotation)
o dehydration corrected over 48 hrs -dose - 2/3 before breakfast, 1/3 before evening meal
o Gradually correct it - if not cerebral edema Parents and the child were counseled and reassured that dia-
o Monitor - IP/OP, SE ,SCr, neurological state betics can have a near normal life with a good glycemic con-
potassium – trol.
o add 20mmol(10ml) KCL to each 500ml of fluid bag they are educated on;
o check SE 2 or 4 hrly Nature of the disease and its long term care.
o titrate the K dose according to SE Importance of good glycemic control.
o Cardiac monitoring for any changes in ECG with Possible complications.
hypokalemia Symptoms of hypoglycemic attacks and treatment.
Symptoms-hunger, sweatiness, feeling of faint, dizziness,
seizure, coma, in young- pallor, irritability
insulin – -predict after severe exercise or missed meal.
o make a syringe pump by adding 50units (0.5ml) S.in- Technique of insulin injection (with insulin pen) and dos-
sulin to 50ml of N/S age.
o rate - 0.1ml/Kg/hr (0.05-0.1) Home glucose monitoring with a glucometer and optimal
o check RBS and titrate the ins. Dose range for blood glucose. HbA1c -keep < 7%, RBS- 4-10
o Convert to subcutaneous insulin when RBS mmol/l
<12mmol/l (Don’t stop infusion before starting S/C Testing urine for sugar at home.
insulin) Dietary control (For the adjustment of diet they were re-
Correction of acidosis- self correct with insulin, Consider Bi- ferred to the dietician.)
carbonate – rarely necessary & only use if severely acidotic -diet -3 main meals and snacks
(pH<6.9) & shocked. - reduce refined CHO
Antibiotics- To tx of intercurrent infections -increase fiber in diet
-Fat intake no more than 30%
-matching food intake with insulin
- 'sick day rules' during illness to prevent DKA

Page | 330
FMAS, RUSL
UNDER THE GUIDANCE OF DR. Anurudhdha Padeniya
Done by Chathuranga 07/08, Seegiri 08/09, Amandha and Dhanushka 09/10
Clinic follow up Intubation & ventilation
Maintain PCO2 >3.5KPa
Assessment of growth Exclude the other DDs by CT scan (thrombosis, hem-
Screened for complications orrhage, infarction)
Physical activities should not be restricted due to disease Repeat manitol after 2 hrs if no improvement
His teachers and friends must be aware of this condition Document all events

4. What are the complications of diabetes?


Target Questions: Immediate
1. What are the other complications of DKA? Hypoglycaemia
Cerebral edema
Acute renal failure DKA
Aspiration pneumonia
Long term
2. How do you identify cerebral edema? Damages to eyes, kidneys, heart, nerves and feet.
Headache
Restlessness, irritability, increased drowsiness, inconti- These are more likely if glycemic control is inade-
nence. quate during puberty, adolescent and thereafter
Cranial nerve palsies, abnormal posture, rising BP, low
pulse Screening for complications
Convulsions, papilledema, respiratory arrest Begin at 5 Years after the diagnosis/after 12 years of
age
3. How do you manage cerebral edema?
Inform senior staff immediately Then annually do screening for eyes and kidney.
Transfer baby to PICU
Exclude hypoglycemia 5. Advantages and disadvantages of HbA1c?
Give manitol 1g/Kg (5ml/Kg of 20% manitol) over
Advantages
20 minutes as soon as possible
Restrict IV fluid to 2/3 maintenance and replace defi- Assess glycemic control for 3 month.
cit over 72 hrs rather than 48hrs

Page | 331
FMAS, RUSL
UNDER THE GUIDANCE OF DR. Anurudhdha Padeniya
Done by Chathuranga 07/08, Seegiri 08/09, Amandha and Dhanushka 09/10
* SHOULD DO AT LEAST THREE TIMES PER YEAR Do not cut the ends of toe nails
IF POSSIBLE
Inspect feet for any change every day
Disadvantages
Costly
8. What is the sick day management?
Hba1c will be misleading if ,
Never stop Insulin
RBC life span is reduce ex; sickle
cell trait Ensure adequate nutritional intake. If not dangerous
changes of blood sugar level can occur.
Abnormal hemoglobin ex; thalasse-
mia If uncertain admit the child to a hospital.

* IN THESE SITUATIONS FRUCTOSAMINE LEVEL


CAN BE MEASURED (It reflects previous 2 to 3 weeks
9. How do you calculate calorie requirement of the child per
glycemic control)
day?
Calculate for current weight of the child,
6. If there is no glucometer what signifies that child blood first 10 kg 110 kcal/Kg
glucose levels are high?
Second 10 kg 70 kcal/ Kg
Polyuria / polydipsia /weight loss in spite of good appetite.
Thereafter 30 kcal/Kg

7. How to protect child’s feet?


Always wear slippers or shoes.
Whenever the feet are wash they should be wiped dry
Apply a cream on feet once a day sparing the web

Page | 332
FMAS, RUSL
UNDER THE GUIDANCE OF DR. Anurudhdha Padeniya
Done by Sajee 07/08, 2nd Edition by Nishadi 08/09, 3rd Edition by Prabath & Danukshi 09/10
History:
Dengue fever
P/C
Presentations:
o Fever for 3 days
Acute fever x duration

Ad. As requested by a GP
Hx/P/C
Features of pre-shock or shock
o Describe the fever fully
DDs:
o Associated symptoms(mentioned above)
Dengue fever/DHF - headache, periorbital pain, arthral-
o Complications - bleeding manifestations
gia, myalgia, peticheal rash, abdominal pain
- Features of shock: sweating,
Leptospirosis – contact hx of muddy water, myalgia,
abd pain, altered conscious level
low UOP, icterus
- postural dizziness, anurea
UTI – urinary symptoms
- Features of pulmonary effusion:
RTI – cough, cold, sore throat, sputum production
SOB
Meningitis – child well in between fever, photophobia,
- Convulsions
phonophobia

Hepatitis – RH pain, icterus, pruritus, pale stools, dark


colour urine P/M/H

o DM, CHD, CLD, CRF – very important as they


may complicate the condition

Page | 333
FMAS, RUSL
UNDER THE GUIDANCE OF DR. Anurudhdha Padeniya
Done by Sajee 07/08, 2nd Edition by Nishadi 08/09, 3rd Edition by Prabath & Danukshi 09/10
Travel Hx o Features of shock – CRFT> 2 sec, peripheral
coldness, restlessness
o Travel to endemic areas
o Hess test
Imm. Hx

Diet Hx
CVS

o Pulse
Social Hx
o BP – narrow pulse pressure
o Contact hx of dengue

o Home environment
RS
o School environment
o RR

o Pulmonary effusion
Examination:

General
Abdomen
o Weight, height
o Tenderness
o Febrile/not
o Ascites
o Pale, icterus, lymph nodes
o Hepatosplenomegaly
o Features of dehydration

o Rashes – bruises, petechial, convalescent rash


CNS

Page | 334
FMAS, RUSL
UNDER THE GUIDANCE OF DR. Anurudhdha Padeniya
Done by Sajee 07/08, 2nd Edition by Nishadi 08/09, 3rd Edition by Prabath & Danukshi 09/10
o GCS b)Hemorrhagic manifestations(at least single positive
tourniquet test)
o Signs of meningeal irritation
c)Thrombocytopenia <100 ,000cell/mm
o Features of encephalopathy
d)Objective evidence of leaky capillaries

3) What are 3 phases in the natural history of DHF?

a)Febrile phasea
Investigations:
b)Critical phase
Discussed under target questions
c)Convalescent phase

4)What are the initial investigation findings to suspect


dengue illness in a child with acute febrile illness?
Management: a)Platelet count - <150 000/mm
Discussed under target questions b)Leucopenia <5 000/mm

c)Rising HCT - 5-10%


Target questions 5)What are the warning signs in suspected DF/DHF
1) What are possible differential diagnosis/What is your di- warrant hospital admission & intense monitoring?
agnosis? dengue fever,DHF,Other viral fever a) abdominal pain or tenderness
2) How you are going to define a case of DHF? b)Persistent vomiting
Following criteria are necessary for the case definition c)Cold extremities & signs of shock
a)High fever or recent history of acute fever d)Clinical fluid accumulation-pleural effusion,ascites
Page | 335
FMAS, RUSL
UNDER THE GUIDANCE OF DR. Anurudhdha Padeniya
Done by Sajee 07/08, 2nd Edition by Nishadi 08/09, 3rd Edition by Prabath & Danukshi 09/10
d)Mucosal bleeding

e)Lethargy,restlessness & drowsiness Adequate physical rest.

f)Liver enlargement-> 2cm Paracetamol 10-15mg/Kg/dose dose

G)Laboratory- Avoid all NSAIDS & steroids

o Increase HCT>10% Monitor patient using monitoring chart

o Decrease in platelet count <100 000/mm Monitoring during febrile phase involves followings

o Elevated SGOT well above SGPT Temperature four hourly

Note- tourniquet test in dengue- Meassure the BP using a cuff Vital parameters-pulse,blood pres-
of a appropriate size for each patient.Raise the preassure to sure(both systolic & diastolic),respira-
midway between systolic & diastolic Bp for 5 tory rate,CRFT-3 hourly
minutes.Release the preassure cuff & wait for another 1
minute before reading the result.Test considered as positive Input/output chart
when there are > 10 petechiae per square inch.(negative test FBC daily(even twice daily if platelet
not exclude possibility of dengue) count drops below <150 000)
6) What is your initial Mx plan at Febrile phase[in ward] HCT – once/twice daily
Ensure adequate oral fluid intake-If unable to tolerate
oral fluids go for iv fluids.Total fluid requirement
(oral+iv) will depend on the degree of dehydra- 7) When you suspect that patient entering critical phase?
tion.The rate of fluid administration reduced soon af-
ter correction of dehydration. Because of DHF pa-
Often after 3 days(Usually occurs between 4 -
tient entering critical phase since 3r day onwards
5th day of illness
need caution on fluid administration.

Page | 336
FMAS, RUSL
UNDER THE GUIDANCE OF DR. Anurudhdha Padeniya
Done by Sajee 07/08, 2nd Edition by Nishadi 08/09, 3rd Edition by Prabath & Danukshi 09/10
Platelet count drops <100 000/mm –most b)Hemorrhagic manifestations(at least single positive
useful & earliest indicator of entering into tourniquet test)
critical phase
c)Thrombocytopenia <100 ,000cell/mm
Progressive rising HCT towards 20% or Ris-
ing HCT > 20% d)Objective evidence of leaky capillaries

Objective evidence of fluid leakage- Pleural 3) What are 3 phases in the natural history of DHF?
effusion on CXR USS or Ascites on USS ab- a)Febrile phase
domen.
b)Critical phase
When in doubt-Biochemical parameters useful
c)Convalescent phase
Serum albumin-<3.5g/dl or if albumin
has dropped <o.5g/dl 4)What are the initial investigation findings to suspect
dengue illness in a child with acute febrile illness?
Serum cholesterol-<100mg/dl or if
cholesterol has dropped by 20mg/dl( a)Platelet count -<150 000/mm
non fasting)
b)Leucopenia <5 000/mm

c)Rising HCT - 5-10%


Target questions
5)What are the warning signs in suspected DF/DHF
3) What are possible differential diagnosis/What is your di- warrant hospital admission & intense monitoring?
agnosis? dengue fever,DHF,Other viral fever
a) abdominal pain or tenderness
4) How you are going to define a case of DHF?
b)Persistent vomiting
Following criteria are necessary for the case definition
c)Cold extremities & signs of shock
a)High fever or recent history of acute fever
d)Clinical fluid accumulation-pleural effusion,ascites
Page | 337
FMAS, RUSL
UNDER THE GUIDANCE OF DR. Anurudhdha Padeniya
Done by Sajee 07/08, 2nd Edition by Nishadi 08/09, 3rd Edition by Prabath & Danukshi 09/10
d)Mucosal bleeding Monitoring during febrile phase involves followings

e)Lethargy,restlessness & drowsiness Temperature four hourly

f)Liver enlargement-> 2cm Vital parameters-pulse,blood pres-


sure(both systolic & diastolic),respira-
G)Laboratory- tory rate,CRFT-3 hourly
o Increase HCT>10%
Input/output chart
o Decrease in platelet count <100 000/mm
FBC daily(even twice daily if platelet
o Elevated SGOT well above SGPT count drops below <150 000)

6) What is your initial Mx plan at Febrile phase [in HCT – once/twice daily
ward]
7)When you suspect that patient entering critical phase?
Ensure adequate oral fluid intake-If unable to toler-
ate oral fluids go for iv fluids.Total fluid require-
Often after 3 days(Usually occurs between 4 -
ment (oral+iv) will depend on the degree of dehy-
5th day of illness
dration.The rate of fluid administration redused
soon after correction of dehydration.Because of Platelet count drops <100 000/mm –most
DHF patient entering critical phase since 3rd day useful & earliest indicator of entering into
onwards need caution on fluid administration. critical phase

Adequate physical rest. Progessive rising HCT towards 20% or Rising


HCT > 20%
Paracetamol 10-15mg/Kg/dose dose
Objective evidence of fluid leakage- Pleural
Avoid all NSAIDS & steroids
effusion on CXR USS or Ascites on USS ab-
Monitor patient using monitoring chart domen.

Page | 338
FMAS, RUSL
UNDER THE GUIDANCE OF DR. Anurudhdha Padeniya
Done by Sajee 07/08, 2nd Edition by Nishadi 08/09, 3rd Edition by Prabath & Danukshi 09/10
When in doubt-Biochemical parameters useful Evidence of overt bleeding & quantifi-
cation
Serum albumin-<3.5g/dl or if albumin
has dropped <o.5g/dl Frequency of monitoring-If the patient haemodynamically
stable-do hourly, If haemodynamically unstable (leaking
Serum cholesterol-<100mg/dl or if
rapidly or while in shock until stable do monitoring every 15
cholesterol has dropped by 20mg/dl(
minutes
non fasting)
9) What are the concerns you take in fluid mx during the
8)How are you monitoring patient during the critical
critical phase?
phase?
Best method is calculation of fluid quota using ideal body
It is very important to monitor the patient very
weight
carefully & frequently during critical
phase,monitoring involves Maximum amount of fluid recommended during entire
critical phase irrespective of it’s lenth is
Total fluid administrated(oral+iv)
Maintenance+5% of body weight (50/ml)
Pulse,blood pressure
Assuming a boy of 16kg (height 102 cm ) & calculate his
Pulse pressure-Target to maintain fluid quota for the entire critical phase
pulse pressure >30mmHg during en-
tire critical phase Maintenance-100(10)+6(50)=1300 ml

CRFT 5% deficit-50(16) = 800ml

Warmth/coldness of the peripheries So total for 48hrs =2100ml

Respiratory rate So hourly fluid quota =2100/48= 43.75ml

Urine output(Aim is to maintain UOP So should provide around 45ml of fluid/Hr


between 0.5-1.0ml/kg/hr
Page | 339
FMAS, RUSL
UNDER THE GUIDANCE OF DR. Anurudhdha Padeniya
Done by Sajee 07/08, 2nd Edition by Nishadi 08/09, 3rd Edition by Prabath & Danukshi 09/10
All patient entering the critical phase to start on IV normal Mnemonic-ABCS
saline or hartman’s solution through IV cannula, in
addition to oral fluid A-Acidosis,B-Bleeding,C-Calcium,S-Sugar

Initial fluid requirement is -1.5ml/kg/hr.Those who can drin 12)What are the complications of DHF?
give IV fluids as 0.5ml/kg/hr to “keep vein open” & the Critical phase
balance as oral.
Fluid overload
Choice of fluid-For those < 6 months-Use N/2+5% dextrose
Prolonged shock
->6 months when not taking orally for
prolong duration –give N saline in 5% Concealed bleeding
dextrose
Acidosis
Subsequent rate of infusion will depend on rate of
Hypocalcaemia
leak(highly vary from patient to patient & even in the same
patient from time to time)juged by pulse,bp,CRFT,HCT & Hyponatraemia
UOP
Hypoglycemia
Hourly urine output is the best guide to decide the rate of
infusion.To maintain renal functions normally in the critical Encephalopathy
phase sufficient UOP is- 0.5-1.0ml/kg/hr
Convalescence
10)How you are going to Mx the shock/
Fluid overload
Refer the National guideline-alogrithm on management of
Hypokalaemia
shock in DHF
Nosocomial infections
11)What should be consider when there is no improvement
despite of adequate fluid therapy 13)What are the indicators that the patient has reached
convalescent phase

Page | 340
FMAS, RUSL
UNDER THE GUIDANCE OF DR. Anurudhdha Padeniya
Done by Sajee 07/08, 2nd Edition by Nishadi 08/09, 3rd Edition by Prabath & Danukshi 09/10
Improved general well being & improved appetite 17)As a House officer what will you look on daily ward round?
,appearance of convalescent rash,generalized
itching,haemodynamic Clinically- in the history-appetite,RUQ tendereness,DIB or
stability,bradycardia,diuresis,stabilization of HCT,rise of any other emerging or resolving complications
white blood cell count followed by platelet count On examination-fever chart,IP/OP
Chart,PR,BP,RR,CRFT,extrimities,any bleeding
14)What are the available laboratory tests available?
manifestations,Evidence for hepatomegaly,ascites,pleural
NS-1 antigen-Which can be done during first 5 days of effusion
fever
Investigations-FBC(Hb,PLTs,HCT),Renal & other
Detection of dengue IgM,IgG or both is performed on blood haematological investigations if necessary.According to
samples collected after 5 days of illness-Highly suggestive senior opinion-USS abdomen(chest),CXR
for dengue

During the first 3 days of the illness,PCR for dengue is


usually positive-used at research level

Isolation of virus in blood can be at 4-5 days –Also at


research level

15)how do you Mx DHF with hepatic encephalopathy?

Refer Nationa guidelines for Mx of HE & complications


during critical phas

16)Who should be transferred to tertiary care?

a) DHF Patient –Entering to critical phase

b) Those who are on prolonged shock state-Need to be


manage at the ICU
Page | 341
FMAS, RUSL
UNDER THE GUIDANCE OF DR. Anurudhdha Padeniya
Done by Hasini 07\08, 2nd Edition by Pavithra & Uthpala 08/09, 3rd Edition by Udara & Pramitha 09/10

■ How did the child’s activity get affected?


Failure to Thrive ■ What do the parents attribute the problem to?
The term failure to thrive is used to describe suboptimal, weight ■ Feeding during illness?
gain in infants and toddlers.

History:
GIT
o Introduction- age of the pt, informant
Appetite (Does the child demand food?)
o P/C→
Sucking, swallowing with ease, recurrent aspirations?
o Failure to gain weight/
Attachment and positioning corrected
o Features of causative disease
Irritable during food ( GORD )
(eg- UTI, RTI)
Vomiting
o H/P/C→ 1) In relation to meal
When the problem was first detected? 2) How often?
Any identifiable reasons around that period? 3) Content
(eg- Beginning of complementary feeding, any illnesses) 4) Projectile (pyloric stenosis )
■ If due to an illness, for how long? 5) Bile stained (duodenal atresia)
■ Was it treated properly? Investigations? Results? 6) Mucus / Altered blood
■ Was it progressive or not? How severe? (Hiatalhernia)

Page | 342
FMAS, RUSL
UNDER THE GUIDANCE OF DR. Anurudhdha Padeniya
Done by Hasini 07\08, 2nd Edition by Pavithra & Uthpala 08/09, 3rd Edition by Udara & Pramitha 09/10

Diarrhea Cyanotic spells

Watery ( viral gastroenteritis) CLD -Icterus, pale stools, dark urine

Steatorrhoea ( biliary atresia , Chron’s disease) GUT

Greesy, frothy, foul smelling Recurrent UTI or features of UTI(Crying during maturation,
increased frequency)
(Malabsorption)
Past history of haematuria , facial swelling
Constipation- Hirsprung's disease
ENT - Ear ache, ear discharges (OM)
Respiratory
MSS -Joint pains, rashes
Cough
Endocrine - Polyuria,polydypsia (DM)
Frequently with wheezing (asthma )
-School performances, appetite (thyroid)Haematological
Persistent loose cough with purulent sputum (cystic fi-
brosis ) Exertionaldyspnea ( anemia )

With fever ( RTI ) Fatigability

Contact history of TB Malignancy – Fractures, bone pain, bleeding manifestations,


anemic features
Recurrent RTI
o P/M/Hx→
CVS
Previous hospital admissions – Cause , duration , &
Whether a known patient with congenital heart disease or outcome
not? Any investigations(echo)?, Any correction surgeries
Recurrent attacks
Poor feeding , Head sweating ( HF)

Page | 343
FMAS, RUSL
UNDER THE GUIDANCE OF DR. Anurudhdha Padeniya
Done by Hasini 07\08, 2nd Edition by Pavithra & Uthpala 08/09, 3rd Edition by Udara & Pramitha 09/10

Feeding during illness o Fever with rash or contact history of such

( exclude all congenital & acquired causes ) o Any medical complications – GDM, HTN, anemia

o Drugs

o Smoking / alcohol / elicit drugs

o Exposure to irradiation

o P/S/Hx→

o Abdominal, bowel surgeries Birth Hx→


(Hirshprung’s, pyloric stenosis, kasai procedure) o Term / preterm

o diagnostic surgical procedures ( biopsy , UGIE , LGIE) o Birth weight

Drug Hx→ o MOD

o Any long term medications( thyroxin ) o Birth asphyxia

o Congenital anomalies detected at birth

AllergyHx→

o Known allergy to drugs, foods ( lactose , cow’s milk pro- Postnatal Hx→
tein intolerance , meat )
o Cried , sucked , handled well

o Time of first feeding


Antenatal Hx→
o Cyanosis with crying

Page | 344
FMAS, RUSL
UNDER THE GUIDANCE OF DR. Anurudhdha Padeniya
Done by Hasini 07\08, 2nd Edition by Pavithra & Uthpala 08/09, 3rd Edition by Udara & Pramitha 09/10

o PBU admissions o BCG scar present or not?

o Neonatal jaundice / convulsions / projectile vomiting


(PS)
o Diet Hx→
o Meconium passed when?
Breast feeding –
o Feeding difficulties (Cleft palate )
o EBF for how long?
o Diarrhea after feeding ( lactose intolerance )
o Duration of one feeding

o Continuously/ stops in between/ scalp sweating/


Development Hx→ tachynpnoea

Milestone achievements with age o Behavior after feeding – sleep / cry / ask more

o Let down relax

o Positioning / attachment
Growth Hx→
Formula milk-
Weight gain in CHDR
o When started / why / who adviced
Gomez classification for weight
o Exclusive / supplement
Waterlow classification for height
o What type / amount / frequency
o ImmHx→
o Concentration( how many tsp per bottle)
o Age appropriate or not?
o Hygiene in preparation FM, diarrheal illnesses?
o Delay due to illnesses?
o baby refused BF after commencement of FM
Page | 345
FMAS, RUSL
UNDER THE GUIDANCE OF DR. Anurudhdha Padeniya
Done by Hasini 07\08, 2nd Edition by Pavithra & Uthpala 08/09, 3rd Edition by Udara & Pramitha 09/10

Compelmentary feeding- o Whether they are using any family planning method? Type,
time duration
o When started?
o Planned pregnancy , unwanted child
o Who advised?
o Illness or growth problems in other children
o What was started?
o Any family illness , mental illness
o Progressive introduction of new items

o Quantity / frequency / consistency/hygiene

o Who prepare? , who feeds?

o Demand feeding doing/food preferences? Social Hx→

o Practice of responsive feeding? o Education of parents

o 24hour dietary recall o Occupation of parents

o Feeding during & recovery of illness? o Income

o Already receiving supplements/ “Threposha”? o Marital disharmony , divorce , family conflicts , loss of par-
ents

o Battered baby
Family Hx→
o Occupants in house, caregivers ,over crowded
o Consanguinity ( high risk of congenital defects & inborn er-
o Stressors
rors of metabolism )

o Family size and ages of other children Economic

o Birth spacing , multiple pregnancy Intrafamilial

Page | 346
FMAS, RUSL
UNDER THE GUIDANCE OF DR. Anurudhdha Padeniya
Done by Hasini 07\08, 2nd Edition by Pavithra & Uthpala 08/09, 3rd Edition by Udara & Pramitha 09/10

Major life events o Skin – Hydration, Hygiene, Rashes, Neurocutaneous mani-


festations, Signs of trauma ( bruises , burns , scars ), Pyoder-
o Housing, toilet, water, refuse disposal magangrenosum, Erythema nodosum, Dermatitisherpati-
o Alcohol, smoking formis, Flaky paint dermatitis, acrodermatitiseuteropathica

o Myths o Head - Quality of hair( flag sign, dry, thin, silky, depig-
mented), Alopecia, Frontanelle size, Frontal bossing, Su-
o School performance tures, Shape, Microcephally

o Parent’s attitudes and expectations regarding the child o Eyes – Ptosis, Strabismas, Palpebral fissures, Conjunctival
pallor, Fundoscopy, Micropthalmia, Bitot’s spots
Examination:
o Ears – low set ears
General→
o Mouth , nose , throat - Thinness of lip, philtrum, Dental
health, Glossitis, Cheilosis, Gum bleeding, Protruding
o Weight , height, OFC – compare with the charts tongue, Cleft lip and cleft palate

o Weight for height - acute malnutrition o Neck- Hairline, Masses, Lymphadenopathy

o Height for age – chronic malnutrition o Hands- koilonychia, leukonychia, finger clubbing, palmar er-
ythema
o MUAC- mid upper arm circumference
CVS→

o Pulse, tachycardia, murmurs


o General appearance- Activity, Posture, Dysmorphism, gross
wasting, Oedema RS→

o Wheezing , Crepts, Acidotic breathing

Abd→
Page | 347
FMAS, RUSL
UNDER THE GUIDANCE OF DR. Anurudhdha Padeniya
Done by Hasini 07\08, 2nd Edition by Pavithra & Uthpala 08/09, 3rd Edition by Udara & Pramitha 09/10

o Protuberance Thyroid function tests

o Hepatospleenomegally 2D Echo - Congenital heart disease

o Masses Stool microscopy & culture

o Distented abdomen with thin buttocks Parasitic infections, GE

CNS→ Urine microscopy & culture – UTI, Renal diseases

o Cranial nerves CRP & ESR – TB, IBD

o Involuntary movements Iron studies -Fe deficiency anemia

o Tone CXR & Sweat test – cystic fibrosis

o power Immunoglobulin -Immune deficiency

o reflexes Chromosomal analysis – chromosomal disorders

o retention of primitive reflexes Antibody profile -Coeliac disease

Investigations:

FBC / DC - Anemia , Infection, Inflammation, Immune defi- Management:


ciency
Multi-disciplinary approach
Plasma proteins - Nutritional level
-dietician
BU / S.Cr / SE - Renal failure, RTA, Metabolic abnormality
-physician
LFT ( S. albumin, PT / INR )
-speech therapist
Liver disease,malabsorption, metabolic diseases

Page | 348
FMAS, RUSL
UNDER THE GUIDANCE OF DR. Anurudhdha Padeniya
Done by Hasini 07\08, 2nd Edition by Pavithra & Uthpala 08/09, 3rd Edition by Udara & Pramitha 09/10

-perants 2) Iron and vitamin supplements

1) Identification and correction of any underlying cause 3) Development

2) High energy intake-150% of their daily energy requirement to 4) Worm treatment


catch up weight and multi vitamins
5) Immunization
3) Dietary and behavioral modification (advice on BF / CF)
Target Questions
4) Monitoring of growth
1) Classify the causes for failure to thrive?
5) If severe degree – inward management with TPN + fluid
electrolyte correction Inadequate intake-

6) Address for family conflicts / physiological support (appropriate Organic


feeding atmosphere at home is important for treatment) –impaired suck/ swallow
Can Mx as an out pt and hospitalization rarely required _ Chronic illness leading to anoerexia

Non organic
Indication for hospital admission _inadequate availability of food
Severe malnutrition _ Psychological deprivation
Failure of outpatient management _neglect or child abuse
Diagnostic purposes Inadequate retention
Follow up -vomiting
1) Weight gain -severe GORD

Page | 349
FMAS, RUSL
UNDER THE GUIDANCE OF DR. Anurudhdha Padeniya
Done by Hasini 07\08, 2nd Edition by Pavithra & Uthpala 08/09, 3rd Edition by Udara & Pramitha 09/10

Malabsorption 3)So you investigate this baby & all investigations found to be
normal. You suspect a feeding problem. How are you going to
-ceoliac disease assess the adequacy of BF?
-cystic fibrosis 4) So you decide to observe BF technique & found it to be wrong.
-NEC How do you counsel?

Failure to utilize nutrients 5) If growth faltering is still persisting what can you do?

-syndromes o Can give breast milk fortifiers

-chromosomal disorders o Can do top up feeding with expressed breast milk

-congenital infection o Can do top up feeding with formula milk

-metebolic diseases o After completion of 4th month, can initiate complementary


feeding early
Increased requrements
6) If the baby is having cleft lip, how are you going to manage?
-thyrotoxicosis
o Multi-disciplinary approach
-malignancy
o Some can still be breast fed successfully
-chronic infection
o Can give expressed breast milk in sitting position with a
-CRF spoon or cup

-congenital heart diseases o If not successful can use special teats, feeding devices and
dental prosthesis
2)You are the HO of paed ward. A 4 month old baby reffered to you
by MOH due to inadequate weight gain. What are the possibilities? 7)If all Ix are normal, how would you manage?

Use weight for heart charts


Page | 350
FMAS, RUSL
UNDER THE GUIDANCE OF DR. Anurudhdha Padeniya
Done by Hasini 07\08, 2nd Edition by Pavithra & Uthpala 08/09, 3rd Edition by Udara & Pramitha 09/10

-1SD to +2SD - Normal 10)what is the classification of malnutrition?

-1SD to -2SD - Dietary advices m o d e r a t e s e v e r e


W/H >-3SD to < -2SD ( 70-79%) Wasting W/H < -3SD ( <70%)
-2SD to -3SD - Dietary advices +Thriposha MUAC < 115 mm
H/Age >-3SD to <-2SD ( 85-89%) Stunting H/ Age <-3SD ( < 85%)
Below -3SD - Dietary advices+BP100

8)How do you advice the mother feeding during & recovery of an 1. Severe complicated- LOA, LRTI,high fever, sverepallor,de-
illness? hydration,not alert, any other
Continue breast feeding. 2. medical condition
Increase the frequency, quality of food in a more palatable 3. Severe uncomplicated- good appetite, clinically well & alert,
manner. no medical conditions
Maintainhygiene. 11)How to manage acute severe malnutrition?
Continue as this at least 2wks after gaining the normal Correct hypoglycaemia, hypothermia,ehydratiion,
growth potential. electrolytes urgently
9) What are the differences between marasmus & kwashiorkor?

M a r a s m u s K w a s h i o r k o r Give antibiotics, vitamins A,B complex, micronutrients


W/H < -3 SD (<70%) Generalized oedeme
Wasted & wizened Hyperkeratosis & desquamation Initiate feeds
N o o e d e m a Distended abdomen & enlarged liver
MUAC/ skin fold thickness markedly reduced Spare &depigmented hair Small volumes,frequently by NG,cup,spoon
Withdrawn &apathetic children Angular stomatitis
Initially formula 75 (75 – 80 Kcal/kg/day)
Brad ycardia, hypotensio n
Low plasma albumin, K+ ,Mg, glucose Icrease gradually if tolerate

Page | 351
FMAS, RUSL
UNDER THE GUIDANCE OF DR. Anurudhdha Padeniya
Done by Hasini 07\08, 2nd Edition by Pavithra & Uthpala 08/09, 3rd Edition by Udara & Pramitha 09/10

If appetite gained give formula 100 (150 – 220 Kcal/kg/day)


6times/day
Gomez Classification (1956)
Gradually switched to Ready To Use TheraputicFood(
RUTF) Based on WEIGHT forAGE

Peanut based products(plumpy nut) Actual wt wt deficit Grade

Solid buiscuits (BP 100)1tab = 150kcal below ref

Target evaluation 90-75% 10-25% I

*Good weight gain >10/kg/day. 74-60% 26-40% II


*Moderate<5-10g/kg/day*Poor<5g/kg/day <60% >40% III

(K,M,M-K)
On discharge refer to nutrition rehabilitation programme (
NRP) Waterlow’s Classification (1972)
Thriposha HEIGHT for AGE >90% Normal
WEIGHT forHt >80%
Corn soya blend

Discharge criteria- good appetite HEIGHT for AGE <90% Stunting


WEIGHT forHt >80% (NL) (chronic)
Minimal oedema
HEIGHT for AGE >90% (NL) Wasting
Medical problems settled
WEIGHT forHt <80% (acute)
Wt/ht> -2SD
HEIGHT for AGE <90% Wasted &
WEIGHT forHt <80% Stunted Page | 352
FMAS, RUSL
UNDER THE GUIDANCE OF DR. Anurudhdha Padeniya
Done by Hasini 07\08, 2nd Edition by Pavithra & Uthpala 08/09, 3rd Edition by Udara & Pramitha 09/10

Improvement of method and skill in feeding

2.Psychological and social aspect

3. Buildup of mother’s confidence

12)How do you counsel the mother with FTT baby?

- Explain the child’s condition

-Advice on

1. Nutrition

Selection of nutritional rich foods

Eating behavior

Quantity and composition of foods intake

Increasing of energy intake

Page | 353
FMAS, RUSL
UNDER THE GUIDANCE OF DR. Anurudhdha Padeniya
Done by Manoj 07/08, 2nd Edition by Gihan and Mayumi 08/09, 3rd Edition By Dileep and Bahagya 09/10

exact duration
PUO any preceding symptoms / events
Presentations: fever pattern with diurnal variation
Fever more than 1wk chills and rigors
DD’s: documentation
Infective -(localize) RTI, GIT ,UTI, bones and joints,
response to antipyretics, dose, adequacy of
Abcesses
dose
Infective-(generalize) IE , IMN , TB , Typhoid , malaria in between fever spikes (active, alert, feeding)
Inflammatory- JIA , SLE ,Kawasaki

Neoplastic – hematological and other malignancies


Associated constitutional symptoms LOA, LOW,
Other-drugs , factitious fever Generalized malaise, fatigability

History:

P/C→ Exclude causes for fever

01 Localized infections
o Fever with duration
o RESPIRATOTY
Hx/P/C→
cough, sputum (if present, describe it )
o Describe fever
sore throat, wheezing, SOB, difficulty in
onset
swallowing

Page | 354
FMAS, RUSL
UNDER THE GUIDANCE OF DR. Anurudhdha Padeniya
Done by Manoj 07/08, 2nd Edition by Gihan and Mayumi 08/09, 3rd Edition By Dileep and Bahagya 09/10

Recurrent aspirations or regurgitations, pleu- 02 Systemic infections


ritic chest pain
o INFECTIVE ENDOCARDITIS
o GIT
past history of Rheumatic Heart disease con-
localized or colicky abdominal pain, bowel genital heart disease ,dental caries, Dental
habbits(constipation, diarrhea), PR bleeding, surgeru, Necrotic skin lesions ,prosthetic heart
malena, tenesmus, Abdominal distension, valves, a murmur detected by GP
vomiting, nausea
o IMN
o GUT
fever
dysuria, urgency, frequency, haematuria, malaise
frothyuria Sore throat (often severe, limiting oral inges-
tion of fluids and food; rarely, breathing may
o Bones and joints be compromised)
jaundice.
bone pain, swelling, limping

Osteomyelitis :- bone fractures, pus draining o TB


sinuses
Chronic cough, evening pyrexia, night sweat-
In SOJIA fever patten is one to two times per ing, contact Hx of TB or pt with prolong
day some times same times in day. But some cough , past Hx of TB
times temp: goes below the normal.

o HEPATITIS

right hypochondrial pain, LOA ,vomiting


,jaundice, Risk exposure o TYPHOID

Page | 355
FMAS, RUSL
UNDER THE GUIDANCE OF DR. Anurudhdha Padeniya
Done by Manoj 07/08, 2nd Edition by Gihan and Mayumi 08/09, 3rd Edition By Dileep and Bahagya 09/10

Exposure to unhygienic food, recent food tak- early morning stiffness, joint involvement,
ing from outside, Salmon pink maculopapular rash, back pain,
red eye
Slow rising fever,dry cough, vomiting, tenes-
mus ,constipation followed by diarrhea o SLE
1st week – headache, malaise
joint pain, photosensitive rashes ,oral ulcers
nd
2 week – high fever, abdominal distension, ,hair loss
pea soup diarrhoea, constipation, Rose spots
over abdomen o KAWASAKI DX

3rd week – intestinal perforation red & oedematous palms & soles, reddish oral
mucosa,cracked lips, peeling of peripheries,
o MALARIA redness of eyes(B/L non purulent conjunctivi-
tis),erythematous skin rash, oedema of limbs
recent travel to endemic area, chills,rigors,
fever pattern (relapsing fever), prophylaxis or o HAEMATOLOGICAL MALIGNANCY
treatments taken
features of anaemia, recurrent infections
o ZOONOTIC ,bleeding manifestations,

animal contact, expose to insect bites Bone fractures & pain specially at night

Toxoplasmosis – neck stiffness, myalgia. Ar- CNS involvement :- headache ,vomiting .sei-
thralgia, maculopapular rash spare palms & zures ,numbness ,body weakness
soles
P/M/Hx→
03 Inflammatory causes
o TB, Congenital heart dx, Rheumatic fever, any other
o JIA medical disorders

Page | 356
FMAS, RUSL
UNDER THE GUIDANCE OF DR. Anurudhdha Padeniya
Done by Manoj 07/08, 2nd Edition by Gihan and Mayumi 08/09, 3rd Edition By Dileep and Bahagya 09/10

Drug Hx → steroids

Birth Hx→ as usual Examination:

Developmental Hx - weight loss or FTT General→

Imm Hx→ BCG Anthropometry - Growth- weight, height , wt/ht with


centile
Diet Hx→
General appearance-ill/well looking
o Allergies
Hydration
o How about protein & calorie in the diet . Because
they tent to lose the weight Skin

Social Hx→ o Malar rash -SLE

o Salmon pink rash -JIA


o General introduction to the family
o Petechial rash, janeway lesions, oslers nodes,
o mother :- age, education level, Employment
splinter haemorrhage, necrotic leisons -IE
o Father :- age, educational level, Employment
o Peripheral purpuric rashes - Vasculitis
o Impact on child (day to day activities ,schooling ) and
o Erythema nodosum –TB, IBD, SLE
family members
o Eschar +/- erythematous rash- Typhus
o Psychological state of parent.
o Rose spots (chest , abdomen) -Typhoid
o Food preparing , water supply, hygienic practices
o Erythematous rash and desquamation –Kawa-
o Nearest hospital, transport facilities
saki disease
Page | 357
FMAS, RUSL
UNDER THE GUIDANCE OF DR. Anurudhdha Padeniya
Done by Manoj 07/08, 2nd Edition by Gihan and Mayumi 08/09, 3rd Edition By Dileep and Bahagya 09/10

o Any bruising o Clubbing with peripheral stigmata of infective endo-


carditis – Osler’s nodes, Janeway lesions, Splincter
o Bleading menifestations – Haematological haemorrhages
malignancy
o Type of joint involvement –which joint , crepitus ,features of
opetechiae on the soft palate, Ampicilin rash-, inflammation(redness , warmth ,tenderness , swelling , loss
maculopapular rash (5%) in IMN
o Head and neck of function)

o Eyes o Lymphadenopathy

Generalized – IMN, SLE , JIA TB hematolog-


pale - anaemia
ical malignancies
conjunctivitis(B/L non purulent)-Ka-
Asymmetrical cervical lymphadenopathy-Ka-
wasaki disease
wasaki
jaundice – haemolytic anaemia, Liver
Evidence of nutritional deficiencies
disease

Fundi - Roth’s spots leukaemia,


vasculitis, IE CVS→
- Retinopaty – Toxoplasmosis o Pulse rate -______bpm (tachycardia?)
- Milliary tubercles - TB o Active precordium, Apex ,Thrill ,Murmurs (? IE)
o Mouth-Reddened lips, Strawberry tongue -
Kawasaki
RS→
o Throat –pharyngitis tonsillitis hyperemic
throats (IMN) palatal petechial oral ulcers

Page | 358
FMAS, RUSL
UNDER THE GUIDANCE OF DR. Anurudhdha Padeniya
Done by Manoj 07/08, 2nd Edition by Gihan and Mayumi 08/09, 3rd Edition By Dileep and Bahagya 09/10

o BCG scar ,RR ,Dyspnoea ,Effusion, Cavitation ,Con-


solidation, Bronchial breathing ,Added sounds
Investigations:
Abd→
Basic Investigations
o Abdominal distention
FBC,ESR,CRP,UFR, Urine culture/ABST, Blood
o Bruising , striae ,rashes ,(rose spots, eschar) culture, Blood picture

o Hepatomegaly - hepatitis, FBC – WBC : infection

o Splenomegaly -SLE ,IE ,malaria Hb : anaemia- chronic infection/ inflammation,


connetive tissue disorder
o Hepatospelenomegaly- IMN , Haematological ma-
lignancies ,Typhoid ,JIA RBC - anaemia

(site ,size, tender or not, margins PLT - Kawasaki (2nd wk)


,consistency)
Pancytopenia BM suppression in leukaemia
Other palpable masses in the abdomen
CRP/ESR - inflammation
Paraaortic LN – lymphoma, Wilm’s tumor
UFR – pus cells, red cells – UTI
CNS→
Albumin – ? renal pathology
o GCS ,Confusion, Restless ,Focal neurological signs
Urine culture – UTI
Musculoskeletal→
Blood picture – Anaemia
bone tenderness – osteomyelitis, leukaemia
Atypical lymphocytes : IMN

Haemaological malignancy
Page | 359
FMAS, RUSL
UNDER THE GUIDANCE OF DR. Anurudhdha Padeniya
Done by Manoj 07/08, 2nd Edition by Gihan and Mayumi 08/09, 3rd Edition By Dileep and Bahagya 09/10

Blood culture - ? bacteriaemia Rheumatoid factor, ANA (may be positive I


JIA, SLE),USS Abdomen (hepatosplenomeg-
CXR – cavitations : TB aly)
Consolidation : Pneumonia o Malignancy

bone marrow biopsy & aspiration


Further investigations according to suspected differential di- o SLE
agnosis.
ANA,Ds DNA, Rheumatoid factor
o IMN
o Malaria
Paul-Bunnel test & mono spot test(Ig G), Se-
cific EBV Ig M Ab test, cold agglutination malaria strip test, thin and thick films
test
o Kawasaki dx
o Infective endocarditis
neutrophil leukocytosis, ESR,CRP
Blood culture, Renal function test, 2D echo
o Osteomyelitis
o Typhoid
USS bone, Aspiration culture/ABST
SAT
o Hepatitis
o TB
Hep B profile
CXR ,Mantoux test, Sputem culture
Management:
o JIA
There is 4 stages of management

1. Hx & Ex
Page | 360
FMAS, RUSL
UNDER THE GUIDANCE OF DR. Anurudhdha Padeniya
Done by Manoj 07/08, 2nd Edition by Gihan and Mayumi 08/09, 3rd Edition By Dileep and Bahagya 09/10

2. Review hx & repeat Ex, Specific Ix Monitor for complications – Severe valvular dysfunction,
Heart failure, Myocardial abscess, systemic emboli
3. Invasive Ix
3. Typhoid
4. Therapeutic trial Correct dehydration, Antibiotics (3rd generation
cephalosporins, Amoxicillin, Chloramphenicol,
Azithromycin, Co trimoxazole)
Acute side bed Monitor for complications – GI perforation, Myocarditis,
Hepatitis, Nephritis
QHT – frequently monitor temperature
4. TB
Ix Anti TB drugs according to the TB regime

Antipyretics
5. Malignancy
Antibiotics Radiotherapy, chemotherapy
6. Kawasaki dx :- Immunoglobulin IV, Aspirin
7. JIA
1. IMN steroids
8. SLE
Usually supportive, if air way severely compromised – Immunosuppretion
corticosteroids, tonsils associated (strep A) Avoid 9. Osteomyelitis / Abscess
Ampicillin & Amoxicillin IV antibiotics according to ABST
2. Infective endocarditis
Target Questions
High dose of empirical antibiotics for 4- wks 1. What is the definition of PUO?

IV Benzyl Penicillin, Gentamicin A temperature greater than 38.3°C on several occasions.


This should be accompanied by more than 3 weeks of illness.

Page | 361
FMAS, RUSL
UNDER THE GUIDANCE OF DR. Anurudhdha Padeniya
Done by Manoj 07/08, 2nd Edition by Gihan and Mayumi 08/09, 3rd Edition By Dileep and Bahagya 09/10

There should also be failure to reach a diagnosis, after 1 Appendisectomy


week of inpatient investigation.
ENT surgery
2. Why do we give high dose antibiotics in IE?
o Tooth extraction
o Infective organisms are lodge in vegetations together
with fibrin and platelets, this high dose need to penetrate o Bronchoscopy
in to vegetations. o GIT ( sclerotherapy, ERCP, endoscopy ,biliary tract sur-
gery)

3. What are the things that should be considered in morn-


5. Fever patterns with examples.
ing ward round?
Intermittent
o General condition of child
o Typhus
o Fever chart and response to antibiotics
o IMN
o Changes in murmur intensity and characteristic
o Hepatitis(other viral infections )
o features of heart failure
o Infective endocarditis

o SOJIA

Remittent
4. Occasions which need prophylactic antibiotic therapy.
o Pyelonephritis
o GUT (cystoscopy,catheterization )
o Cholecystitis
o Surgeries with high risk of bacteraemia

Page | 362
FMAS, RUSL
UNDER THE GUIDANCE OF DR. Anurudhdha Padeniya
Done by Manoj 07/08, 2nd Edition by Gihan and Mayumi 08/09, 3rd Edition By Dileep and Bahagya 09/10

o Meningitis IV immunoglobulin 2g/kg over 10-12 hrs and

o Kawasaki disease High dose of Aspirin 80-100mg/kg/day divided every


hourly orally until pt is afebrile for atleast 48hrs
Continuous
(As soon as diagnosis & within 10 days of disease onset )
o Typhoid,
Convalescent stage (after the pt has been afebrile for 48 hrs)
o typhus
Aspirin 3-5mg/kg once daily orally until -8 weeks after dis-
Tertian/Quitrent
ease onset
o malaria
Long term therapy for pt with coronary abnormalities
Fever patterns
Aspirin 3-5mg/kg once daily orally

Clopidogrel 1mg/kg/day (maximum 75mg/day)

Most experts add Warfarrin or LMWH for pt at high risk of


thrombosis

Acute coronary thrombosis

Prompt fibrinolytic therapy with tissue plasminogen activa-


tor or other thrombolytic agent under supervision of paediat-
rics cardiologist

7. How do you manage Typhoid?


6 .How do you manage Kawasaki disease?
Adequate rest
Acute stage

Page | 363
FMAS, RUSL
UNDER THE GUIDANCE OF DR. Anurudhdha Padeniya
Done by Manoj 07/08, 2nd Edition by Gihan and Mayumi 08/09, 3rd Edition By Dileep and Bahagya 09/10

Hydration Tapering the dose over 7 days

Close observation Indications - incipient airway obstruction, thrombocytopenia


with hemorrhaging, autoimmune hemolytic anemia, seizures,
Correct fluid & electrolyte imbalance meningitis
Antipyretics – Paracitamol 10-15mg/kg 4- hrly

Diet – soft 7 easily digestible (unless pt has abdominal dis-


tension or ileus)

Antibiotics to minimize the complications

8. How do you manage IMN?

No specific Rx

Symptomatic Mx

Bed rest (if pt has debilitating fatigue)

After symptomatic improvement allow normal activities

Avoid contact sports during 1st 2-3 weeks of illness (risk of


splenic rupture)

High dose Acyclovir +/- corticosteroids (corticosteroids re-


duces complications)

Predinesone 1mg/kg/day (maximum 60mg/day) or


equivalent for 7 days

Page | 364
FMAS, RUSL

You might also like